Sie sind auf Seite 1von 214

April Paper 1

Topic: poverty and developmental issues

1) The fight against extreme poverty must also focus on the challenge of ensuring environmental
sustainability. Critically comment. (200 Words)

Live mint

Poverty is multi-dimensional concept and till poverty is eradicated on all counts, poverty eradication will
never be sustainable. One such dimension of poverty relates to environment and its sustainability.

Environment Sustainability

Environmental sustainability allows for the needs of man to be met without jeopardizing the ability of
future generations to meet their needs. In poverty eradication discourse it is often thought that poverty
eradication schemes introduce wasteful practices and are danger to environmental suitability. However
the recent MDGs and SDGs have simultaneously targeted poverty and environmental sustainability,
depicting their dependence on each other.

Actions for sustainability

what individuals can do:

1. engage personally-keep surrounding clean

2. Own less

3. Reduce waste

what business can do

4. Integrate sustainability into core business culture

5. Provide accurate product labelling

6. Lead globally with innovative design to reduce GHG emissions

7. Create zero carbon landscapes

8. Design for climate change

9. Reduce disaster risks

what governments can do:


10. Teach environmental ethics

11. Tax non-renewable and luxury

12. Go renewable
13. Promote the transport pyramid

14. Restore ecosystems

15. Plan for population increase

16. Involve the public as a knowledge base.

Why it is needed?

Fifteen years on from the historic Millennium Declaration, indicators show that our failure to protect our
environmental systems is undermining much of the progress that has been made in helping the worlds
poorest communities. The stories from around the globe are all too familiar.

Small-holder farmers in Tanzania have been suffering smaller yields as a result of soil
degradation;
Communities in Bangladesh are struggling to cope with increasingly erratic weather conditions as
a result of climate change;
Indigenous peoples in Latin America and South-East Asia are searching for alternative livelihoods
where high levels of deforestation have robbed them of their principal economic assets.
Unmindful subsidies in agriculture have resulted lowering of water table, soil pollution, wastage of
energy in India.
IPPC report has indicated that climate change will have most adverse effect on marginal section
of society. Number of increased extreme events like episodes of drought and flash rains throws
back marginal section below poverty line.

Thus Post-2015 agenda must tackle the relationship between poverty and sustainability if it is to bring
about long-lasting change.

Action Plan

Multilateral Environmental Agreements and Poverty


Multilateral environmental agreements such as the Convention on Biological Diversity, Convention to
Combat Desertification and the Framework Convention on Climate Change contain mechanisms to
protect the environment that can also be harnessed to reduce poverty. These mechanisms can be linked
to poverty alleviation efforts through local-level natural resources management.

Connecting Poverty and Ecosystems


for e.g. International Institute of sustainable development developed a framework that connects people's
well-being to ecosystem services, which was then used to provide a preliminary overview of the state of
poverty and ecosystems in seven sub-Saharan countries. Same can be applied elsewhere too.

Participatory Techniques and Capabilities


Although the poor are becoming increasingly involved in the various stages of development, questions
remain as to whether their inclusion constitutes genuine participation and whether people's capabilities
have been increased in such a manner as to enable them to chart the course of their destinies in
collaboration with the government, NGOs and the international community

Policy Coherence and Governance


the poverty-ecosystem nexus is governed by a complex system of institutions, organizations and policies
at international scales right down to the local community. Need is to find ways that policies, organizations
and institutions can work together to achieve linked poverty alleviation and environmental management
goals.

Poverty and Ecosystems Report


many reports are produced that assess various aspects of the state of human poverty and the
environment, but none fully convey an assessment of essential ecosystem-poverty links. Need is to
develop such reports and to inspect the wide-ranging elements found in these reports for a clearer picture
on integrated poverty-ecosystem framework.

Sustainable Livelihoods
Livelihood is the capabilities, assets (stores, resources, claims and access) and activities required for a
means of living. A livelihood is sustainable if it can cope with and recover from stress and shocks,
maintain and enhance its capabilities and assets, and provide sustainable livelihood opportunities for the
next generation. Need is to see how climate change and natural disasters are impacting livelihoods and
for ways to make livelihoods more sustainable.

Vulnerability and Adaptation


many people are vulnerable to natural hazards and disasters because of where they live or because they
lack the necessary capabilities and community support to survive and thrive when disaster strikes. Need
is to identify policies that promote local resilience and adaptive management so that vulnerability is
reduced.

Poverty reduction can also be green and fair.

Costa Rica provides cash payments for environmental services on the premise that those who maintain
natural resources should be compensated for doing so. Forests now cover more than 50% of Costa
Rica's land, compared with 21% in 1980.

The CBD under Nagoya protocol provide incentive to local population to preserve biodiversity as they will
be beneficiary of economic gains out of biodiversity.

In India, there is a clear awareness of the complex two way relationship between poverty and
environment, as indicated in National Environment Policy -2006.

The forest Act-2006 provide right to collect minor product for tribal people in forest.
The most effective actions for reducing poverty and fostering human development are those that integrate
the sustainable management of ecosystem services, the promotion of economic development and
ensuring social equity within a single comprehensive framework. Implementing these actions requires the
use of a wide range of toolsfrom public participation techniques to integrated assessment toolsto
understand the complex nexus among constituents of well-being, ecosystem services and institutions.

2) What do you understand by invasive species? How do they alter local ecology? Illustrate with
suitable examples.

The New York Times

Invasive species maybe termed as species, who either


naturally or by anthropogenic activities, invade habitats which they do not
originally belong to Invasive species hold a considerable threat to transitioning or weak ecosystems that
are considerably at risk from natural or man-made effects. The invasive characteristic of the species is
further defined by how it acclimatizes to the system, sometimes replacing components of a tropic system
to manipulate it to its own needs. Therefore, with a successful invasion local ecology can be altered to the
point where salient features are changed and the ecological nature of the system is altered.

IMPACTS:

1 Due to their high reproductive capabilities, most invasive species have the capabilities to displace local
counterparts and disrupt ecosystems.

2 They also disturb the ecological balance, by wiping out prey species which do not see them as a threat,
like the pythons in Florida are doing to wood rats and other species.

3 They compete for resources against the local species. Alligators and pythons have been competing for
food and space in the everglades recently.

4 Invasive species sometimes adapt by preying on their local cousins, like Asian lionfish when introduced
into American markets preys on the local species.

5 Sometimes, invasive species like snakes and other venomous reptiles can be dangerous to human
lives as well.

6 Transformation of physiological conditions. For example: Eutrophication by blue green algae.

7. Such species could interbreed with native species resulting in changes to the genetic makeup. It will
have adverse negative consequences which might result into creation of a more successful invader or
susceptible to certain pests.

Some invasive species, like the Himalayan blackberry have positive effects as well in the form of edible
fruits eaten by both humans and animals. But most invasive species have detrimental and insidious
effects on the local populations and have to be controlled, and in some cases eradicated.
Topic: Indian culture will cover the salient aspects of Art Forms, Literature and Architecture from ancient
to modern times.

3) Write a note on the form and style of Vijayanagar empire paintings.

Intro:-

With the decline of power of the Chola dynasty in the thirteenth century, the Vijayanagara Dynasty
captured and brought under its control the region from Hampi to Trichy with Hampi serving as its capital. -
Many paintings survive in a number of temples.
-The paintings of the Vijayanagar represent the great revival of Hindu religion and art in South India.
During the Vijayanagar era, mural paintings made a comeback after the chalukya's intense interest in
mural paintings at Badami.

Vijayanagara Style:-
The paintings at Tiruparakunram, near Trichy, done in the fourteenth century represent the early phase of
the Vijayanagara style.

- In Hampi, the Virupaksha temple has paintings on the ceiling of its mandapanarrating events from
dynastic history and episodes from the Ramayana and the Mahabharata. Among the important panels are
the ones which show Vidyaranya, the spiritual teacher of Bukkaraya Harsha, being carried in a palanquin
in a procession and the incarnations of Vishnu.
- The faces of the figures are shown in profile, with large frontal eyes.
-The figures have narrow waists.
- In Lepakshi, near Hindupur, in present Andhra Pradesh, there are glorious examples of Vijayanagara
paintings on the walls of the Shiva temple.
- In keeping with the tradition, the Vijayanagara painters evolved a pictorial language wherein the faces
are shown in profile and figures and objects two-dimensionally.
- Figures stand with a slight slant with both feet pointing in the same direction.
--Lines become still but fluid, compositions appear in rectilinear compartments.
- All these paintings are seen mainly on the ceilings of the Mandapas and in the corridors of the temple.
- The themes of them are generally religious
- There is simplicity and vigor in the style of the paintings with a sense of movement and energy caught in
the figures.
- These paintings show a rhythmic forward movement and do not look overcrowded. They also show the
episodes from the life of King Manunitikanda Chola, who granted justice to a cow whose calf was run over
by his son's chariot.
- Most of these Vijayanagar paintings depict the mythological legends of Siva and stories from the epics
but they also represent the life and customs of the Vijayanagar Empire.
-These stylistic conventions of the preceding centuries were adopted by artists in various centers in South
India as can be seen in the paintings of the Nayaka Period.

Nayaka paintings were more or less an extension of the Vijayanagara style with minor regional
modifications and incorporations.

NCERT, Class XI, Fine Art (Introduction to Indian Art), Chapter 5

Topic: Indian culture will cover the salient aspects of Art Forms, Literature and Architecture from ancient
to modern times.

4) How do temples in Uttar Pradesh, Rajasthan and Madhya Pradesh differ from temples
found in Assam, West Bengal and Odisha in form and artistic style? Discuss.

Intro:-

THE BASIC FORM OF THE HINDU TEMPLE


The basic form of the Hindu temple comprises the following:
(i) A cave-like sanctum (garbhagriha literally womb-house), which, in the early temples, was a
small cubicle with a single entrance and grew into a larger chamber in time. The
garbhagrihais made to house the main icon which is itself the focus of much ritual attention;
(ii) the entrance to the temple which may be a portico or colonnaded hall that incorporates space
for a large number of worshippers and is known as a mandapa;
(iii) from the fifth century CE onwards, freestanding temples tend to have a mountain like spire,
which can take the shape of a curving shikhar in North India and a pyramidal tower, called a
vimana, in South India;
(iv) The vahan, i.e., the mount or vehicle of the temples main deity along with a standard pillar or
dhvajis placed axially before the sanctum.
Two broad orders of temples in the country are known Nagara in the north and Dravida in the
south. At times, the Vesara style of temples as an independent style created through the selective
mixing of the Nagara and Dravida orders is mentioned by some scholars.

Every region and period produced its own distinct style.

Central India: Temples of Uttar Pradesh, Madhya Pradesh and Rajasthan


-classic nagara style of temples
-made of sandstone
-Some of the oldest surviving structural temples from the Gupta Period are in Madhya Pradesh. These
are relatively modest-looking shrines each having four pillars that support a small mandapa which looks
like a simple square porch-like extension before an equally small room that served as the garbhagriha.
-panchayatana style of architecture where the main shrine is built on a rectangular plinth with four smaller
subsidiary shrines at the four corners (making it a total number of five shrines, hence the name,
panchayatana).
-tall and curvilinear shikhara
-The crowning elements: amalakand kalash, are to be found on all nagara temples of this period.
-Khajurahos temples are also known for their extensive erotic sculptures; the erotic expression is given
equal importance in human experience as spiritual pursuit, and it is seen as part of a larger cosmic whole.
-Many Hindu temples therefore feature mithun (embracing couple) sculptures, considered auspicious.
-Usually, they are placed at the entrance of the temple or on an exterior wall, or they may also be place
on the walls between the mandapa and the main shrine.

East India:
Eastern Indian temples include those found in the North-East, Bengal and Odisha. Each of these three
areas produced distinct types of temples.
-The history of architecture in the North-East and Bengal is hard to study because a number of ancient
buildings in those regions were renovated, and what survives now are later brick or concrete temples at
those sites.
-It appears that terracotta was the main medium of construction, and also for molding plaques which
depicted Buddhist and Hindu deities in Bengal until the seventh century.
-A large number of sculptures have been found in Assam and Bengal which shows the development of
important regional schools in those regions.
Assam:
Post-Gupta style continued in the region well into the tenth century.
-However, by the twelfth to fourteenth centuries, a distinct regional style developed in Assam. The style
that came with the migration of the Tais from Upper Burma mixed with the dominant Pala style of Bengal
and led to the creation of what was later known as the Ahom style in and around Guwahati.
-Kamakhya temple, a Shakti Peeth, is dedicated to Goddess Kamakhya and was built in the seventeenth
century.
Bengal:
The style of the sculptures during the period between the ninth and eleventh centuries in Bengal
(including Bangladesh) and Bihar is known as the Pala Style.
-While the style of those of the mid-eleventh to mid-thirteenth centuries is named after the Sena kings.
While the Palas are celebrated as patrons of many Buddhist monastic sites, the temples from that region
are known to express the local Vanga style.
-The ninth century Siddheshvara Mahadeva temple in Barakar in Burdwan District, for example, shows a
tall curving shikhara crowned by a large Amalakaand is an example of the early Pala style. It is similar to
contemporaneous temples of Odisha.
-The black to grey basalt and chlorite stone pillars and arched niches of these temples heavily influenced
the earliest Bengal sultanate buildings at Gaur and Pandua.
-Many local vernacular building traditions of Bengal also influenced the style of temples in that region.
Most prominent of these was the shape of the curving or sloping side of the bamboo roof of a Bengali hut.
-This feature was eventually even adopted in Mughal buildings, and is known across North India as the
Bangla roof.

Odisha:
The main architectural features of Odisha temples are classified in three orders, i.e., rekhapida, pidhadeul
and khakra. Most of the main temple sites are located in ancient Kalingamodern Puri District, including
Bhubaneswar or ancient Tribhuvanesvara, Puri and Konark.
-The temples of Odisha constitute a distinct substyle within the nagara order.
-In general, here the shikhara, called deul in Odisha, is vertical almost until the top when it suddenly
curves sharply inwards. Deuls are preceded, as usual, by Mandapas called jagamohana in Odisha. The
-ground plan of the main temple is almost always square, which, in the upper reaches of its
superstructure becomes circular in the crowning mastaka. This makes the spire nearly cylindrical in
appearance in its length.
-Compartments and niches are generally square, the exterior of the temples are lavishly carved, their
interiors generally quite bare. Odisha temples usually have boundary walls.

NCERT, Class XI, Fine Art (Introduction to Indian Art), Chapter 6

Topic: Indian culture will cover the salient aspects of Art Forms, Literature and Architecture from ancient
to modern times.

5) Write a note on the significance of the statue of dancing girl of Mohenjo-Daro to Indias art
and culture.

'Indus dancing girl', represents a stylistically poised female figure performing a dance. The
bronze statue is hardly four inches high yet speaks ample of the superb craftsmanship and of
the caster's skills.

Significance-
1)SOCIAL OUTLOOK
- The forward thrust of the left leg and backwards tilted right, the gesture of the hands, demeanour of the
face and uplifted head, all speak of absorption in dance, perhaps one of those early styles that combined
drama with dance, and dialogue with body-gestures.

- It was a well-developed society, Indus people had innovated dance and other performing arts as modes
of entertainment. Her elaborately coiled hair, her bangles and necklaces speaks of social life of India at
that time. Dancing figurine indicates that dance was the part of their culture.

2) SCIENTIFIC & TECHNICAL OUTLOOK- But its most interesting aspect is that in casting it the
Harappan metallurgists used an advanced technique known as the lost-wax (cire perdue) process. The
Indus artists knew metal blending and casting and perhaps other technical aspects of metallurgy.

3) SKILL OF ART & HANDICRAFT - The figure has been cast as wearing on her
breasts a necklace with four 'phalis' like shaped pendants.

4) DRESS, HAIR STYLES & ORNAMENTS - Women loved jewellery and wore heavy bangles in
profusion, large necklaces, and earrings, possibly indicates different ethnicity or status

5) WOMENS STATUS- with hand on hip, and the expression of self-assurance on her face, it represents
a respected & confident woman of harrapan society

6) NON SHYING SOCIETY - This small figure confirms that the Harappan people were neither shy of
nakedness nor of explicit sensuality. The artificats such as "Dancing girl" of Mohenjo-Daro helps us to
analyze and reconstruct our past. So efforts should be made to excavate and preserve such artifacts
which have immense significance.

Topic: Indian culture will cover the salient aspects of Art Forms, Literature and Architecture from ancient
to modern times.

6) Examine the influence of Buddhism and its concepts on the art of Mauryan empire with
suitable examples.

Asoka made a dramatic conversion to Buddhism after witnessing the carnage that resulted from his
conquest of Kalinga. He adopted the teachings of the Buddha known as the Four Noble Truths, referred
to as the dhamma. We can see following influence of Buddhism and its concepts on the art of Mauryan
empire.

influence of Buddhism and its concepts on the art of Mauryan empire -:

1. pillars: The physical appearance of the pillars underscores the Buddhist doctrine. Most of the
pillars were topped by sculptures of animals. They carry messages of benevolence, kindness and
mercy inspired by Buddhism.

2. The Lotus symbol of purity and the unspoiled Nature of Buddha each pillar is also topped by
an inverted lotus flower, which is the most pervasive symbol of Buddhism (a lotus flower rises
from the muddy water to bloom unblemished on the surfacethus the lotus became an analogy
for the Buddhist practitioner as he or she, living with the challenges of everyday life and the
endless cycle of birth and rebirth, was able to achieve Enlightenment).

3. This flower, and the animal that surmount it, form the capital, the topmost part of a column. Most
pillars are topped with a single lion or a bull in either seated or standing positions. The Buddha
was born into the Shakya or lion clan. The lion, in many cultures, also indicates royalty or
leadership. The animals are always in the round and carved from a single piece of stone.

4. Stupas : these are the holy places constructed over the relics of Buddha, and eventually
chaityas, viharas, were constituted for prayer halls and resting places for the followers of Budha.
They were also erected at pilgrimage sites such as at Bodh Gaya, the place of Buddhas
Enlightenment, and Sarnath, the site of his First Sermon and Sanchi, where the Mahastupa, the
Great Stupa of Sanchi, is located. Stupa(became objects of cult worship) is a burial mound for an
esteemed person. When the Buddha died, he was cremated and his ashes were divided and
buried in several stupas. These stupas became pilgrimage sites for Buddhist practitioners.

5. Torranas : these gateways to stupas , which had beautiful carvings of jatakas representing the
previous lives of Buddha and also the various stages from birth to mahaparinibbana(death) , also
we find the carvings of a Big tree ( the Bodhi tree) where the Buddha attained Bodhi/
enlightenment

6. The Wheel of law (dhammachakra), The wheel (chakra) is a symbol of samsara, both the
endless circle of birth and rebirth, and the dhamma, the Four Noble Truths

7. The Buddha's footprint to represent the teachings of Buddha.

Topic: Indian culture will cover the salient aspects of Art Forms, Literature and Architecture from
ancient to modern times.

7) Gupta sculpture is the logical outcome of the early classical sculpture of Amravati and
Mathura. Discuss with examples.

The Gupta sculptures not only remained models of Indian art for all time to come but they also served
as ideals for the Indian colonies in the Far East. The finished mastery in execution and the majestic
serenity of expression of the image of Buddha came to be adopted and locally modified by Siam,
Cambodia, Burma, Java, Central Asia, China and Japan, etc. In the Gupta period all the trends and
tendencies of the artistic pursuits of the proceeding phases reached
their culmination in a unified plastic tradition of supreme importance in Indian History.

a) Importance to Sensitiveness of Art form


1. The art of the Gupta is considered as the pinnacle of Indian Buddhist art. Hellenistic elements are still
clearly visible in the purity of the statuary and the folds of the clothing, but are improved upon with a very
delicate rendering of the draping and a sort of radiance reinforced by the usage of pink sandstone. Artistic
details tend to be less realistic, as seen in the symbolic shell-like curls used to render the hairstyle of the
Buddha. Stone and bronze images of Buddha have also been found at Mathura and other places.

2. The magnificent red sandstone image of the Buddha from Mathura is a great example of Gupta
workmanship datable to the 5th century A.D.

b) Aesthetic and Spiritual Sense of Art:

Standing Buddha from saranath and seated Buddha from Mathura are exhibiting great sense of Spiritual
Serenity (Spiritual enlightenment) is matured art which reached its climax from previous art of Mathura
(giving importance to sensual sense) and ornamentation of Amaravthi
The image of the standing Lord Buddha is an excellent example of Gupta sculpture. The molded figure
has its right hand in the attitude of assuring protection. Wheel of Law is one of
the masterly creations of Gupta classical sculpture. The image is carved in Chunar sandstone and has a
surface texture of smoothness. The decorative prabhas are characteristic also of Mathura images.

c) Importance to temple Sculpture

During Gupta regime revival of Hinduism assumed paramount importance. Temples at Udayagiri and
Ajanta with beautiful ornaments and decoration of Lord Vishnu and lord Shiva and Ganges and
Yamuna River Goddess might have played major role in temple architecture

A representation of Vishnu belongs to the Gupta period, 5th century A.D., and comes from Mathura. The
typical gown, the vanamala; the string of pearls twirled round the neck, the long and elegant yagnopavita
are characteristics of early Gupta sculpture.

So we can say Gupta sculpture is the logical outcome of the early classical sculpture of Amravati and
Mathura. Its grace is derived from that of Mathura and its elegance from that of Amravati. Gupta sculpture
seems to belong to a sphere that is entirely different.

Topic: changes in critical geographical features (including waterbodies and ice-caps) and in flora and
fauna and the effects of such changes; Paper-3; Conservation

8) Critically discuss the effects of deforestation on Indian economy

Indian Economy derives its raw material, land and water resource from Forests. Industries
like pulp and paper, textiles, rubber, SMEs, mining, steel and other metallurgical process
directly or indirectly dependent on forest areas. Hence, effects of degrading the forest
resource have bad effects on Indian economy too.
Recent study which observed that Deforestation in Northern mid attitudes has caused a decrease
of 1/5th of Monsoon in India though raised moderately, the rain in Southern Hemisphere and
Australia and South Africa.

It is evident that Deforestation has an impact on the Indian economy, which still largely depends
on Agriculture and Monsoon. Some of effects of Deforestation are
~ Greater Pollution or carbon emission with no carbon capture increased the Health burden of the
country
~Greater amount of funds will be diverted for the climate change , resilient crops which otherwise
could have helped in raising the incomes
~Monsoon plays key role in Monetary policy of RBI, thus tight and dear economy may harm the
market sentiments Like after ELNINO ,ENSO
~Deforestation caused increased land pressure as more tribal becomes share croppers thus
increasing of lower strata in the economy
~Deforestation and Encroachment to natural resources like Wetlands etc also causes disasters
like Deluge in Uttaranchal etc
~Forest, tress preserve and withhold the nutrients in the soil. Absence causes greater use of
fertilizer, which is generally subsidized in India ~ 70k cr subsidy
~Tourism and Aviation are also affected as, Eco tourism is affected, also Leads to job loss
unemployment etc As cited in recent AAI report
~It also causes Change in rainfall, crop failure, pushing farmer s to migrate already stressed over
burdened cities, thereby causing slums, poor hygiene crime, prostitution, social inequalities etc

In sum depleting the Natures wealth also decreased the Human and Economy Health.

Topic: urbanization, their problems and their remedies.

9) Recently, the National Green Tribunal banned all diesel vehicles over ten years old from
plying in Delhi and the National Capital Region. Examine why only diesel vehicles are
banned but not petrol ones.

Intro:-
The recent move by the NGT to ban all diesel vehicles in Delhi (ranked as the most polluted city in the
world.), which are older than a decade, is an initiative to reduce the air pollution in the city.
-the recent reports have shown the increasing level of pollution growing in the city and certain immediate
measures were required.

The move to ban old diesel cars only, and not the petrol cars, can be justified by the following points:
-Since long, diesel has been cheaper than petrol, which has led to the diesel cars being bought more than
petrol cars. Banning old diesel cars can be seen as an initiative to bring number of both types of cars on
par with each other, as government has now decided to equalize price of both in 5 years, in phases.
-Catalytic converters in petrol cars can convert the CO into CO2, which is lesser harmful. Such devices
are not present in most diesel cars.
-Diesel cars have much higher emission of nitrous oxides and particulate matter, having a more serious
impact on the exposed persons, which can both damage the respiratory system, due to production of
ozone on ground level. Both these (nitrous oxide and ozone) cause respiratory disorders, asthma,
choking, irritation to eyes, nose, etc. and the children and the elderly are most vulnerable. Diesel has
been shown to be a carcinogen. According to recent estimates, daily 78 people die out of asthma and
related air pollution effects in Delhi region.
[Ground level ozone pollution, formed when nitrogen oxides and hydrocarbon emissions combine in the
presence of sunlight]
-Diesel vehicles are cause of emission of carcinogenic compounds and heavy metals such as arsenic,
selenium, cadmium and zinc.
-Ultrafine particulates, which are small enough to penetrate the cells of the lungs, make up 80-95% of
diesel soot pollution. Particulate matter irritates the eyes, nose, throat, and lungs, contributing to
respiratory and cardiovascular illnesses and even premature death. Although everyone is susceptible to
diesel soot pollution, children, the elderly, and individuals with preexisting respiratory conditions are the
most vulnerable
-Diesel is more polluting than petrol because combustion capacity of diesel is less.
-Those vehicles which are decade older or so do not have any compulsion to follow Bharat Stage 4. Older
engines were made without any environmental concerns and degrade further with time and cost less than
petrol.
-Even the fuel price of diesel is less than petrol. While the public transport fleet of Delhi is on CNG,
growth of diesel use is a big threat.
-Diesel vehicles are primary reason for soot, SOx and NOx emission compared to a petrol car. A typical
new diesel car on the road emits 10 times more nitrogen oxides.
-New engine standards for diesel cars, trucks and heavy equipment have traditionally lagged far behind
those for petrol powered vehicles which need to bring on par with petrol vehicles.
-The diesel in India contains more sulphur which reduces the benefits of better diesel engines
-Diesel exhaust particles and gases are suspended in the air, so possibility of exposure stands high,
especially when diesel-powered engines have large presence on Indian roads. People spending more
time on roads, truck loading and unloading operations, operating diesel-powered machinery or working
near diesel equipment face exposure to higher levels of diesel exhaust and face higher health risks.

Conclusion:-
Keeping the global concern on diesel use and citizens welfare, the National Green Tribunal has exercised
its authority well.
-Stringent regulatory measures, new diesel technology, vehicles equipped with advanced diesel
emissions controls are urgently needed
-Retrofitting the old diesel vehicles and equipment with advanced emission control devices can effectively
reduce harmful emissions. It will be important to ensure that emission levels are maintained throughout
the life of the vehicle through periodic testing.
The Hindu

Reference

Reference-2

Topic: urbanization, their problems and their remedies.

10) India recently launched National Air Quality Index to provide real time data on air quality in
some of Indias bigger cities. Write a critical note on this policy initiative and examine if
anything more needs to be done to tackle air pollution in Indian cities.
Intro:-

Indian cities with 13 in the Top 20 most Air Polluted cities according to WHO report are extremely
unhealthy for the population and reducing the life expectancy of its citizens by more than 3 years a
person. The Indian Govt. has recently launched the Air Quality Index in 10 cities which will display the
status of PM 2.5, harmful gases and PM 10 in the Air.
Positive as aspects:-
- With the launch of the National AQI in 10 cities for now India has become among the countries that
monitors the air pollution data in real time. This policy initiative certainly is good and progressive move as
it will help the inhabitants with the real time situation of the air quality level in cities, with Delhi and other
12 Indian cities being reported in the 20 world's most polluted cities, having prior information about the air
quality outside will help to avoid going out by children and old age persons who are most vulnerable to
get affected by poor air quality.
-Monitor real time data on air quality and display the hazard level using color coding. Green for safe levels
and progressively reddish as limits are breached.
-Pollutants like particulate matter 2.5 and 10, nitrogen and sulfur oxides, ozone, carbon monoxide are
included
-The plan is to extend this initiative to all big cities.
-Making citizens more informed
-Give an opportunity to citizens to enforce their right to live in a health environment. Now citizens will be
able to move to courts with evidence in their hands
-Air quality will become an important pillar for public policy
-Tries to monitor a number of pollutants
-Will cover entire India within 5 years
However, in spite of all the good things that it offers, there are still some loopholes like:
-No separate index on residential and industrial areas
-Benzene and some other deadly pollutants have not been included in the index
-No prescription for dealing, if the air quality is poor
- Quality of data from some cities remains weak as monitoring stations are not fully equipped.
- Standards set for pollutants fall short of WHO recommendations, values for respirable particulates far
exceed even a lax threshold from health point of view.
- for hazardous pollutants such as PM10, WHO limits are 10 mcg/m3 but in our NAQI it is set at "50
mcg/m3" for good rating. India's tolerance level for 2.5 PM is 50 mcg per cubic meter whereas the
international standard is only 40 mcg per cubic meter.
- do not have a mechanism in place to bring down peak pollution levels.
-Standardization of various monitoring instruments still needs to be done
National Air Quality index with "One number-One color-One description" property will make the common
man aware of his surroundings but at the same time will not relieve him if surroundings have poor air
quality.

Though the NAQI can be termed as a good move but reactive steps also needs to be taken for dealing
with poor air quality that persists in some of the cities of India. For example in China if pollution levels
cross a threshold, primary schools are shut down. Also recently in Paris, 50% of the cars were taken off
road when pollution levels were breached.
Further steps needed:-
- banning the old vehicles from plying on the road, encouraging the use of clean energy, public transport
(metros etc.) , getting the vehicle tested for pollution regularly, encouraging tree plantation on the
dividers, encouraging people working in same office for carpooling etc.
- imposing a congestion charge on vehicles, which could be used to fund investment in public transport
system
-Dedicated corridors for cyclists
-Sensitizing people to use public transport
-Increased taxes on fuels and other carbon taxes
-Improvement in emission norms by including Euro 4 and higher standards
Conclusion:-
Air pollution and water pollution can significantly blunt life enhancement opportunities of the affected. But
a sustainable like micro power grids etc. and equitable one say. Polluter pays yields quality which
enhances productivity and longevity of citizens.
-Apart from all these, there is a great need in change of lifestyle of we people also. The condition of our
environment is basically in our hand only. So we must avoid any step which is harmful to our environment
directly or indirectly.
The Hindu

Topic: urbanization, their problems and their remedies.

11) In your opinion, what are the effective and scientific ways of garbage management? Discuss.
(200 Words)

Globalization which is the leading cause of Urbanization, has evolved to solve many basic problem of the
society such as poverty, unemployment , education & health facilities but also producing by-product such
as Garbage which has now become the basic problem.

Waste Management: Waste management is the collection, transport processing or disposal of the waste
materials in an effort to reduce their effect on human health or services.

Potential hazards from waste/ poor waste management:


-Surface water and soil contamination
-Can acts as epicenters for spreading of diseases
-Can undergo fire accidents and
-Can block the canals there by causing flooding
-Improper disposal may leads to release of harmful heavy metals in the food chain

Ways of Garbage Management:

Using incinerators to burn the garbage and usage of components like Catalytic converters to convert
gases like CO to CO2 etc for low harmful gases emission.

Proper segregation of types of garbage like domestic organic waste, plastic, electronic items, chemical
compounds, metals etc. This helps in reusing or recycling along with proper disposal techniques for few
types.

Use of aerobic treatment to reduce bio waste

Usage of organic waste especially in rural areas for preparing compost which is rich in nutrients can be
used in organic farming.

Setting up of recycling and sewerage treatment plants at industrial waste outlets, rivers etc for effective
management of waste from the industries and also treating sewerage for eliminating garbage.

Plastic recycling must be done as laying plastic roads, preparing pipes etc for better usage and re-usage.
New small scale industries to segregate parts within electronic items which need careful separation of
harmful elements for reusage and also using new technology for recycling.

Biological treatment: solid sanitizers, microbes for water conversion and microbial culture for accelerated
decomposition.

Anaerobic digestion or Incineration: generate energy using the best technology for incineration that can
replace the fossil fuels which are blamed for GHG emissions.

Discouraging people for using plastic by providing alternatives, encouraging them to reuse plastic; and to
minimize the waste.

Additional information:
The developed world has overcome this problem and is now sharing their experiences in helping
developing world to deal with it.
Specially the one like India where average urban density is 8000 & largely unregulated.
It is $400 billion dollar industry. In developing countries, 64% of the population lives in urban
development and next 35 year it will be far greater.
Urban Development Ministry has recently come up with idea of 3Rs Reduction, Recycling and
Reuse of Solid Waste and Sewage to deal with it.
In reducing measures such as: Segregation of household garbage at source; Use of incinerators &
catalyst convertors; Restricting use of polymer to specific purpose only; Use of jute bags; Organic
composts can be converted in to manures; Proper dumping sites which can later be converted into parks
e.g.. Ghazipur - Delhi dumping site by Reliance.
In recycling: Recycling polymer, glass & paper products spreading awareness for reuse. Cotton, Jute,
Toys products can be reused by poor peoples. Sewage treatment plants for segregating solid waste
before dumping water into rivers.
Reuse: Kitchen water can be reused with small modifications; rain water harvesting, Community
participation etc.
Incentives to be provided to start-ups in waste management to make the move more economical. With
4 Ps model i.e. People, Private, Public Partnership we can solve our new basic problem.

Topic: Distribution of key natural resources across the world (including South Asia and the Indian
subcontinent);

12) Examine why smuggling of Red Sanders trees is rampant in some parts of India. (150 Words)

Information about RED SANDERS:

Fragrance less variety of the Sandalwood, having botanical name Pterocarpus santalinus.

Luxury wood is in great demand in the world, especially in UAE and Japan and neighboring countries, as
they are associated with the status symbols and also used for producing best quality musical instruments.

Red sandalwood grows mainly in deciduous forests of the central Deccan, between 500 ft and 3000 ft. in
Andhra in Palakonda and Seshachalam hills in the districts of Kadapa and Chittoor, in some contiguous
areas of Anantapur district, in the Nallamala forests in Kurnool and Prakasam, and in parts of Nellore
district. However, majority of red sander smuggling takes place in Chittoor and Kadapa.
Red Sanders is a protected species under the Convention on International Trade in Endangered Species
(CITES) of Wild Fauna and Flora

The wood is very costly and receives about 30 lakh a tonne in the international market, but the trees
require at least 25 years to mature and the woods be useful.

Also, the area in which they grow is thorny deciduous region limited to only ~5000 sq kms.

It is not frost tolerant, being killed by temperatures of 1 C.

It grows even in barren, degraded and hilly soil with lesser water in soil.

The wood has historically been valued in China, particularly during the Qing Dynasty periods, and is
referred to in Chinese as zitan

Red sandalwood grown on the shale sub-soils, at height between 500-3500fts altitude. Such semi-arid
climatic conditions give a distinctive wavy grain margin. Lumber pieces with the wavy grain margin are
graded as A grade. Red sandalwood with wavy grain margins sells at higher prices than the standard
wood.

So, there is ban to save them from extinction and they are also listed in the Convention on International
Trade in Endangered Species (CITES) of Wild Fauna and Flora.

Reasons for smuggling red sanders:

Red sanders, or red sandalwood, has huge demand abroad, especially in China and Japan, and
commands good price.

High economic value complemented by medicinal


use and also used as flavoring in alcohol beverages

The southern states where it is mostly grown


have inadequate checkpoints at the borders facilitating easy movement

Members of a police task force that was supposed to protect forest officials were alleged to be taking
bribes from smugglers and allowing them to enter the forest.

Lack of employment opportunities and lure of money by mafias are acting as basic drives among the
village people.

Though the State governments has formed Anti Red Sanders Smuggling Task Force(ARSSTF) but it is
not effective as they were recently only give rights to fire for defense, otherwise they used to be mute
spectator and smugglers used to attack them with sharp objects and stone pelting.

Lack of commitment in implementation of Andhra Pradesh Sandalwood and Red Sander Wood Transit
Rules, 1969 and Andhra Pradesh Red Sander Wood Possession Rules, 1989.

Suggestions:

Efforts should be made to check smuggling of red sanders and nab the kingpins of the smuggling rackets.

The personnel should be asked to take strict measures when the smugglers resort to violence.
The forest check posts manned by forest guards should be strengthened with deployment of armed
contingent from the police department and these measures apart, villagers should also encouraged on
the entry points (of forests) to apprehend the smugglers.

The southern states where it is mostly grown have inadequate checkpoints at the borders facilitating easy
movement

Topic: Locational factors of resources and issues related to them

13) Write a note on the locational factors and performance of paper industry in India.

INTRODUCTION

India continues to rein one of the fastest growing paper markets in the world. There are about 759 units
which manufacture pulp, paper board and newsprint.

LOCATIONAL FACTORS OF PAPER INDUSTRY

1. Lots of electricity - So paper industry is present in the Himachal hills as the Himalayan Rivers
are a source of cheap and reliable hydroelectric power. Saharanpur is the epicenter of paper
industry in North India
2. Proximity to forests where lumbering is possible. So mostly temperate forests where single
species grow, not much dense as compared to very dense, non-replenish able and mixed species
tropical forests.
3. Transport: The transport cost of newsprint and paper is slightly higher than the transport cost of
its raw material. Paper mills are usually situated near the coast, where the rivers on which the log
are floated reach the sea, and where paper or pulp can be loaded on ships for export. Ex.
Sweden and Britain Mills.,
4. Closeness to water which is used for transportation of logs, in bleaching and automating the
manufacturing process. For example in Canada, lumbers place logs on frozen rivers which are
carried downstream. Also here wood behaves as a weight losing raw material as roughly 50% is
used for pulp, rest is discarded.
5. Capital: Paper industry is a capital-intensive industry; therefore, large capital is required for
sophisticated machinery and other works. The larger the mill the greater is the initial costs. The
large paper mills are also associated with the printing and publishing business and often own
their own forests.
6. Besides, nowadays specially in India, there is a trend of recycling waste paper, using bamboo or
grasses and using bagasse form sugar cane mills to manufacture pulp which is environmentally
positive. Saharnapur is a centre for paper industry in India.

PERFORMANCE OF PAPER INDUSTRY

1. Indian paper industry has exhibited resilience and has shaped up well in the face of increasing
competition from the overseas players.
2. Total operating capacity of Indian paper industry is around 11 million tons. Presently FDI up to
100% is allowed on the automatic route for the pulp and paper sector. Still there is need to
work on some aspects.
3. Indian paper industry is in fragmented structure. Certain specialty is required as security papers
and check papers, etc. special papers are being imported in the country.
4. Our demand for paper has grown but is slowing down due to proliferation of web based
technologies and computer. Offices are encourage are to go paper-free. Schemes like Sarva
Siksha Abhiyaan, etc push up the demand.
5. But we have to face tough competition from imports from China and SE Asia.

Topic: Secularism

14) The idea of secularism as expressed in our Constitution and as articulated over the years
through the political process is embedded in the concepts of equality and democracy rather than
in the Western concept of secularism which denies religion any space in the public sphere. In the
light of the statement, critically analyze how Indias version of Secularism is different from that of
Wests.

INTRODUCTION

Secularism is one of the core principles of Indian constitution and is highlighted by


Articles 25 to 28.

Secularism is followed in two ways in any country

1. State should respect all religion equally or

2. State should totally dissociate itself from religion.

FEATURE OF WESTERN SECULARISM

1. In the West, secularism has always had a narrow connotation. It mainly meant that the state
does not follow any religion. In other words, the separation of church and state has been the main
emphasis by Western secularism: we can call this negative secularism.

2. The Western secular concept, instead of integrating the world, has achieved the exact opposite. It
has divided people along racial, religious, national, and cultural lines. Western secularism pits
science against religion and spirituality.

3. Church dont try to politically motivate people

4. Religious minority is given freedom of faith

5. In the western model, the religion is private matter, not a matter of state policy or law. This model
interprets freedom and equality in an individualist manner. There is little scope for community
based rights or minority rights.
6. Capitalism needs the exact opposite value to be successful: extravagance, arrogance,
selfishness and greed in order to make people good consumer.

FEATURE OF INDIAN SECULARISM

1. There is no state religion in India. No religion has been given special rights and no religion is
given any special help by the state.
2. The Constitution grants religious freedom to all the people. Every individual can pursue religion of
their choice
3. Indian Secularism over a period has demonstrated equality and democracy as part of secularism
and is based on equal treatment to all religions as 'Sarva Dharma Samabhava'.
4. Indian concept of secularism is of Principled distance between State and religion: State can
interfere with religion where it is discriminatory, intra-religious domination
5. Politically motivated religious ideology, religion becomes public matter in some circumstances
6. Unlike the Western concept of secularism which envisions a separation of religion and state, the
concept of secularism in India envisions acceptance of religious laws as binding on the state, and
equal participation of state in different religions.
7. There is given an adequate amount of freedom to manage internal affairs of people. (There is no
universal civil code still).

Thus, we can see wider and clear difference in western and Indian Secularism, in terms of:
1.Inclusiveness and uniqueness (roots of Indian culture are secular)
2. Recognition and preservation (equality before law and equal opportunities)
3.Powerful but vigourless (cant challenge national spirits and aspirations)
4.Loyalty (faith teaches to respects and to be loyal towards nation)
5. Understanding (people don't disrupts propagation of each others faith)

CONCLUSION

So, from Indias points of view, secularism is evolved through religious reforms movements of
19th century, freedom struggle, and partition of nation on Two nation Theory. Indian secularism
provides respect to all religions. This character suits better to reality of deep and multifaceted
religious identity of Indian people.

Western secularism is more lining towards creation of modern nation state, where religious
intervention must not slower the progress of economic and political systems.

Topic: Locational factors of resources and issues related to them

15) Critically analyze problems faced by farmers and consequences of these problems in the
cotton growing belt of the Vidarbha region.

INTRODUCTION

Vidharbha is home for approx. 3.4 million cotton farmers and 95% of these are struggling with the
massive debt. Most of the villages in Vidharbha are badly in need of basic social infrastructure.
PROBLEM FACED BY FARMERS

1. Climate change- unseasonal monsoons and unseasonal winter rainfall as well as droughts have
affected Vidharba, which practices rainfed agriculture almost entirely.

2. The minimum support price which the government provides has been coming down, from Rs
7000 in 2011-12 to Rs 4050 this year. This has led to indebtedness amongst the farmers.

3. Lack of irrigation and other facilities in the region has led to crop failures when monsoons are
irregular.

4. Economic reforms and lack of proper supply chain and marketing or APMC facilities, lower MSPs
force these cotton growers to resort to distress sales to private money lenders making it hard for
them to recover their production costs too.

5. Short-sightedness of the state government in providing cash relief and loan waivers only gives
them relief for that particular period.
6. Rising costs of inputs, poor yields and slowdown in export markets to USA and China has led to
farmers being left in dire distress.
7. Uncertainty of agricultural enterprise in the region, employment problem of the farmers' children,
decreasing interest of the young generation in farming, rapid urbanization, etc.
8. Absence of adequate social support infrastructure at the level of the village and district

CONSEQUENCE OF SUCH PROBLEMS

1. Mounting distress on farming community takes toll; as can be seen from rising number of
suicide cases among famers of the region.

2. All these issues has led to farmers being pushed to indebtedness and poverty, this also leads to
land fragmentation

3. Ten years back, international price of cotton lint was $1.10 a pound ($2.42 a kilo) but now it is 52
cents. The retail price of cotton then was Rs.40 a meter, and it is now Rs.80. Retail prices have
doubled but farmers are forced to sell their produce at half the price.

4. The cotton yielded this year is of very low quality, as a result of which the government refuses to
buy them since they do not match the grade that has been prescribed at the MSP. There might
be some private players, who will then buy them for a much lower price than MSP.

Way ahead

solution lies not in announcing successive loan waiver scheme as announced by GoI but in
ensuring better infrastructure in the region in form of irrigation & electricity;

The government should act quickly to remedy this situation and also introduce its crop insurance
and other schemes in this area on a priority basis.
Topic: Indian culture will cover the salient aspects of Art Forms, Literature and Architecture from
ancient to modern times.

16) EXAMINE THE EVOLUTION AND SALIENT FEATURES OF BENGAL SCHOOL OF ART.

INTRODUCTION

Bengal school of art was an Indian style of painting that originated in Bengal and flourished through India
during the British Raj in early 20th century. it was associated with Indian nationalism and led by
Abanindranath Tagore; promoted & supported by British art administrators like Haveli

EVOLUTION

1. During Indian nationalist movement, especially after the partition of Bengal in 1905, painting
became subject matter.
2. It was served as a counter to the Western styles taught in the art colleges of the time, revived and
modernised the old Mughal, Rajasthani and Ajanta styles.
3. In 1906, the year after the partition, the iconic painting of Bharat Mata, was the gentle yet
vulnerable and subjugated figure became a symbol of the nationalist movement.
4. Bengal school arose as an "avant-garde"; experimental or innovative work in Indian art.
5. academic art styles promoted during British raj by both early British art schools and Indian artists
such as Raja Ravi Verma were imitative and westernized; lacked Indian values and was
unsuitable for depicting rich culture of India.

SALIENT FEATURE

Abindranath Tagore's painting was stylistically unique and displayed his immense creative range.
The paintings are so evocative and powerful that they can draw a viewer right into the stories that
they tell. The expressions of paintings are kind and gentle, but at the same time, there is a sense
of sadness and nostalgia. It has a misty quality to his works, which became one of his
trademarks.
While rejecting colonial techniques, the Bengal School drew heavily from other Asian
traditions. For e.g. the Japanese wash technique was imbibed to lend the works a soft and misty
quality.
Diverse themes, ranging from Indian landscape and nature, Mughal and Rajput historical figures
to scenes, characters from Indian mythology and anecdotes, were employed. E.g. Passing of
Shah Jahan, Krishna series, Mangal Kavya.
It is indigenous yet modern style. The paintings are Indian in spirit and attitude;
each painting is stylistically unique and display immense creativity of the painter.

Thus, Bengal school created different subject and brought common people on oils, but this was
done by reviving old and medieval Indian style of paintings
Topic: Salient features of worlds physical geography.

17) Little-understood western disturbances have been blamed for most of the freak weather
events in India in the past decade. What do you understand by these Western Disturbances?
How do they originate and what effect they have on Indias economy? Discuss. (200 Words)
Down to Earth

Western Disturbance is the term used to describe an extra-tropical storm that brings sudden winter rain
and snow to the northwestern parts of the Indian subcontinent. This is a non-monsoonal precipitation
pattern driven by the Westerlies.

Origination:

Westerlies pick moisture over Caspian Sea.

Jet Streams move to south of Tibet during winters. They carry these moisture laden winds to India.

Origination:
WDs originate in the Caspian Sea or the Mediterranean Sea as extra-tropical cyclones. They gradually
travel across the middle-east from Iran, Afghanistan and Pakistan to enter the Indian sub-continent.
Western Disturbances the strongest in January and February over India:
Though WDs move across the Indian region throughout the year, they are in their peak during winter
months of January and February. Their effect is minimal during the monsoon months in India.
The induced systems and their effect:
Induced systems are secondary low pressure areas or cyclonic circulations induced by the primary WD.
Generally these are observed over central Pakistan and adjoining west Rajasthan region which gradually
shift eastwards, accentuating rain over Northwest India. They also lead to rise in temperatures, fall of
surface pressure, appearance of high, medium and low clouds. Normal pressure and wind patterns are
restored with the moving away of the disturbance.

Affects on Economy:
1. Western Disturbances are important to the development of the Rabi crop in the northern
subcontinent, which includes the locally important staple wheat.
2. Their effect sometime extends up to Gangetic plains and Northeast India, also causing heavy rains
and destroying crops.
3. But in the past few years western disturbances have been linked to disasters.
4. This years hailstorm and rains in Gujarat, MP, UP and Rajasthan has caused severe damage to
Cotton, Mango, Wheat and gram crops.
5. It has weakened economic conditions of farmers in these states.
6. Such erratic disturbances can lead to the failure of the crops, which makes the govt compensate the
farmers for their loss, and leads to further unplanned expenditure.
7. Lack of timely aid from the govt, worsens the condition of the farmers even further, which demotivates
them from continuing in the sector, leading to immigration.
8. Many hailstorms have devastated multiple crops like cotton, mango, wheat, etc.
9. Many incident like Leh flash flood, J&K flood, have been seen linked to the Western Disturbances
10. Distress among farmers, farmer suicide,
11. Food inflation may become high.
12. Unemployment increases, protest against govt increases, law and order problem.
13. The Western disturbance affects day-to-day weather of northwest India especially during winter
season.
14. It is usually associated with cloudy sky, higher night temperatures, unusual rain etc.
15. Over the Indo-Gangetic plains, it brings cold wave conditions and occasionally dense fog and cold
day conditions. These conditions remain stable until it is disturbed by another Western Disturbances.

Conclusion:
Though generally beneficial, western disturbances are causing problems in agriculture output of India,
due to climate change all over world. Effectively countering climate change will restrict its adverse impact.

Topic: Modern Indian history from about the middle of the eighteenth century until the present- significant
events, personalities, issues

18) Ambedkar was one of modern Indias first great economic thinkers, its constitutional
draftsman and its first law minister who ensured the codification of Hindu law. Elaborate
Intro
Dr. B. R. Ambedkar had a multifaceted personality. He was a jurist, economist, and politician,
constitutional expert and social reformer and played an important role in building modern India.
Following are his major achievements and contribution:

Contribution
1. Ambedkar was one of the first generation of professionally trained economists in India, who made a
name for himself as a monetary economist. His doctoral thesis was on the problems of the Indian
rupee, wherein he argued for the limited devaluation of the overvalued exchange system, which was
in place to help the British exporters, somewhere between the exchange rates that the 2 competing
groups were in favour of:
colonial government representing British business interests that wanted to maintain the existing
exchange rate
Congress speaking for Indian business interests that wanted a cheaper rupee;
2. Explained the effect of gold on exchange rate system. His ideas on monetary and credit control in the
Indian economy due to fluctuation in gold standards has been the foundation RBI's monetary policy;
his advocacy of price stability over exchange stability
3. As the chairperson of drafting committee, Dr. Ambedkar was the chief architect of our Constitution
based on values of democracy, liberty, equality and secularism (termed as the 'father of the Indian
Constitution'). It was his legal acumen and mastery of language, which makes the Indian constitution
one of the best constitutions the world over. He continues to inspire with his debates, which formed
part of the constitutional assembly debates.
4. He favoured industrialisation based economic model along with core socialistic values of Gandhian
economy. He favoured skill development, land reforms and technological upgradation in agriculture.
5. His targeting of centralization of financial powers as an ill for dev of masses provides inputs for
subsequent Finance Commissions in independent India and present stress on 'cooperative
federalism'
6. He worked relentlessly for the rights of the depressed class to end the discrimination faced by them,
made provisions for abolition of caste system and abolition of untouchability in the constitution. He
emerged as the leader of the underprivileged and even separately represented them at the round
table conferences in 1930s. It was after his signing of the Poona Pact with Gandhi that ensured
greater number of seats for the depressed class eventually becoming one of the most important
constituents of the Indian polity.
7. He inspired the modern Buddhist movement in India by converting to the Buddhism along with six
lakhs of his followers.
8. He was also an eminent educationist and founded institutions like the People Educations Society,
Milind College to advance the interests of the scheduled castes.
9. His disagreement with Marxian thought that only economy is not the sole criteria for exploitation in the
society, but also socio-religious factors in a society like India
10. As the first law minister of the country, he led the campaign for the codification of Hindu law that
worked as 'magna carta' for equitable status of Hindu women in society & provided for gender
equality in areas of marriage and inheritance overcoming opposition from conservative sections. He
could balance the rights of different sections of the Indian society with preservation of national interest
and central control on important matters
11. Reforming the agriculture and land relations in rural India through cooperative movement,
provisioning of input subsidy, fixing of minimum wage, shifting of labours from agriculture to industry
etc.
12. Played role in land reforms, income tax policy for poor i.e. Taxation according to the payer's capability
and not income. Progressive taxation holds significance in the Indian societies with growing
inequalities
13. Emphasis on family planning movement for economic development by Advising birth control
14. He Served as member in Viceroy council
15. He contributed towards modern India's economic planning
16. He was also the one to provide ideas and opinions on the formation of Reserve Bank of India to
Hilary Young.
17. He emphasized on placing the liberal values clubbed with freedom in the form of rights to its citizens.
He has to walk a tight rope of balancing the need to design a modern India that can compete globally
but at the same time secure its depressed section of citizens from the oppression and in -equalities
and discrimination

Conclusion
For all of these achievements, Baba Saheb has been awarded the Bharat Ratna in 1990. Though Dr.
Ambedkars role in constitution is often highlighted, his important contributions to the Indian economy
and socio-religious movements cannot be superseded. Thus, Ambedkar has contributed immensely
to bring 'independent India' onto the path of 'modernised India.'

The Hindu

The Hindu

Topic: Modern Indian history from about the middle of the eighteenth century until the present- significant
events, personalities, issues

19) Ambedkar stands apart for combining three attributes: modernity of outlook; bringing
scholarship and learning to political life; and pragmatism in public life. In the light of the
statement, critically examine Ambedkars contribution to India both during pre and post
Independence years. (200 Words)

Live mint
The Indian Express
Pre-independence:

Modernity of outlook:

1. Staunch believer in democracy. He said "Democracy is not a form of government, but a form of
social organization."

2. He was against coercive centralized institutional authority that Hobbesian Philosophy maintains.
For him political democracy is not an end in itself, but the most powerful means to achieve the
social and economic ideals in society.

3. Guided the passage of factories act, trade union act (factory working hours reduced to 8 hours
prevalent even today), minimum wage act etc. which were way forward to more humane working
conditions of workers and find their place in India even today.

4. His modern ideas on economics stated debates on money problem, monetary governance and
ways to industrialize India.

5. Was against Gandhis vision of village and said, What is a village but a sink of localism, a den of
ignorance, narrow mindedness and communalism. Argued villages are Oppressors for
downtrodden.

On Castes:

1) Insisted that a classless society is impossible without a casteless society and only then
individualistic principles would govern social mobility.

2) He called for annihilation of caste instead of uplifting depressed ones opposed to Gandhiji. He
said till there is caste, there will always be discrimination as we see in India today.

3) Poona Pact, established societies to mobilize people politically, urging people to go to towns etc.
were his pragmatic methods to fight caste discrimination.

4) He formed various organisations such as Bahishrit Hitkarini Sabha, Peoples Education


Society to uplift depressed classes and castes of society.
5) Published books like Mooknayak to give a voice to out casted people.

Education and Empowerment:

1) He supported western education and also absorption of new technologies for betterment of
economy such as large dams and multi-purpose projects.

2) His independent labour party protested against the Wardha system of education proposed by
Gandhi and some leading educationalists.

3) Fought for the education of masses without discrimination on any grounds. He founded People's
Education Society and started colleges at Bombay and Aurangabad.

4) He strongly advocated family planning measures, in 1942; he introduced Maternity Benefit


Bill during his tenure as Labour Minister in Governor Generals Executive council. His
introduction of Hindu Code Bill was a part of Social Engineering via law.

Economic Issues:

1) His PhD thesis the evolution of provincial finance in British India became the basis for
finance commission of India.

2) Highlighted the economic problems persisting in British India and wrote a book named the
problem of Rupee and its solutions. This book became a guide for RBI Act, 1934.

Pragmatic public life:

1) Fighter for human rights, not only for the most oppressed section of Dalits, but for all the Indian
caste-oppressed groups, for workers and farmers, and for women.
2) Organised protest against "Black Bill" and in 1938 ILP organised a huge march of 20,000
peasants and workers (largest pre-independence peasant mobilization).

Partition: Supported partition as that was a pragmatic solution of communalism and also
improving social and political status of Muslims at that time.

Post-Independence:

1) As Law Minister of Independent India, he worked for betterment of women by pushing Hindu
Code Bill which was defeated but eventually got passed in 1956.

2) Headed the Drafting committee of Constitution where his contribution as a framer of


constitution is living example of present modern society where he had taken into account of
various diversities present in our country.

3) Called Modern day Manu because of his important role in drafting a constitution based on
modern values of equality, liberty and freedom.

4) Provision of reservation was his pragmatic way to uplift depressed classes in modern India till
they attain self-reliance.
5) He helped in setting up central Technical power board later evolved as Central power grid
system and central irrigation and navigation commission which helped in various crucial dam
projects of independent India such as DVC.
6) In his work "Thoughts on linguistic states" he predicted the future he advocated for
organization of linguistic states in India.

Conclusion:
Dr. Ambedkar walked on the tightrope between securing a modern society for all Indians and
ensuring that a nation stabilised around a constitutional architecture of social change.
The pragmatic solutions of Dr. Ambedkar can be used to resolve societal problems of present
era. HOs ideas should be emulated to bring social equality in India.

Topic: The Freedom Struggle its various stages and important contributors /contributions from different
parts of the country

Topic: The Freedom Struggle its various stages and important contributors /contributions from different
parts of the country

20) Critically examine the role played by the British in partitioning India in 1947. (200 Words)

British Role:

- Increasing placation to fundamentalists

- Fulfilling demands of the religious groups discarding nationalists

- Various acts and orders, and finding security of the empire in these activities were the prime
reasons for the partition.

- The birth of communalism may be ascribed to British help to Sir Syed's viewpoint of Muslim social
enhancement, and their support to his view against congress. further acts such as separate
electorates granting through various acts in 1909 and later, active concessions given to Muslim
educational reforms that were separate from the whole educational support, communal award in
1932, utilizing the religious chasm between moneylenders and peasants to their purpose, giving
effective veto to the ML in Wavell discussions making them the sole representative of the
Muslims, backing out from the law and order and other administrative functions during riots of
1946.

- To stem nationalism, British had to divide people along whatever lines possible like religion,
caste, creed, etc. Communal lines were to prove most advantageous to colonialists and they
knew this. To prop up Muslims against numerically superior Hindus, it was necessary to support
them disproportionately. Hence we can see acts of British like communal electorates in Morley-
Minto reforms, stoking the issue of minorities time and again, not banning the League during Quit
India Movement and quid pro quo with League members, allowing Muslim League to veto
proposals for independence in the name of minority safeguards, etc. They also not checked
Hindu communal elements.

- When communal carnage prevailed in India, still British were responsible at the Centre and they
did not take any steps to check communalism. Rather they were deliberately more focussed on
immediately transferring power to Indians and leave with a face-saver and shed any responsibility
of communal carnage.

- Mountbatten deliberately delayed the Boundary Commission award and also the civil servicemen,
administrators and army were not put in place before division. This caused confusion and further
aggravated the situation.

- Having said this, the main reasons were also related to world problems of similar kinds. The
Yugoslavian question was referred by the Muslims to the British parliament for similar solution.
The liberals then demanded the protection of minorities in India. Probably, this was the reason for
granting veto for the ML. the social conditions were also backward and inadequate representation
in the higher administrative posts, all culminated in granting concessions that further fuelled the
demands.

Conclusion:

So British had initially sown the seeds of communalism for their own benefits but later realized that this
Frankenstein monster has grown beyond control and they rushed out before it degenerated into a civil
war.
However, the British did look sincere in avoiding partition especially during Cabinet Mission but the
celerity of Mountbatten gave little time for leaders to work out a solution over time.
But this also true that divided India did retain maximum unity but this was also an aim of the British for
Commonwealth interests.

Topic: Role of women and womens organization

21) This years (2015) UN report on sexual violence in conflict documents horrendous crimes
against women like those happening in conflicts around the world. Give a critical account of such
recent crimes and role of womens organizations in addressing such crimes. Also examine what
needs to be done at global level to stop such crimes. (200 Words)

Introduction:

- In 2014, 122 countries at the Global Summit to End Sexual Violence in Conflict in London made
renewed commitments to eradicate this human rights violation. The UN also released a report on
sexual violence in conflict that points out about 19 countries where such crimes have taken place
in 2014.

Some recent examples:


Boko Haram's abduction of 276 schoolgirls from their dorm in Nigeria and their abuse of women
and girls as a central belief of their doctrine.

Abduction of Yazidi women and girls by IS for sex slavery.


3. Several States themselves are to blame because they provide impunity to security forces who
commit these crimes in conflict or disturbed zones.

The use of conflict related sexual violence (CRSV), which includes various forms of sexual
violence including rape, abduction, sexual slavery, forced prostitution, forced pregnancy and
forced marriages, has been reported from various war zones including - Central African Republic,
Iraq, Somalia, Afghanistan and the Syrian Arab Republic.

Despite the political momentum and visibility gained in recent years, CRSV continues to remain
underreported because Governments have not been able to create an environment in which
survivors feel safe to report sexual violence.

Role of womens organizations in addressing these crimes:

(a) A country that does not respect women in peace time will not protect women in conflict. Hence,
these organizations should empower women to demand more representation, participation and
equality.

(b) Provide comprehensive assistance and care to survivors of sexual crimes.

(c) Coordinate with the governments in training, improving data collection, monitoring and
documentation of these crimes.

(d) Offering protected accommodation for women who are vulnerable to violence, thereby save them
from further abuse

(e) Helping victims of sexual violence to receive therapy and medical care.

(f) Giving women in war zones the opportunity to meet and share experiences, thus giving them the
courage to speak up.

(g) Providing legal aid and inform women about their rights.

(h) Helping create a conducive atmosphere for the rehabilitation and resettlement of the victims by
inculcating in them self-confidence.

However, WOs by themselves cannot tackle the issue of CRSV. What is needed is global coordination on
this issue in the following ways:

Formally recognising CRSV as a tactic of war and terror.


Employing all means available at the disposal of international organisations like the UN to
influence parties to conflict to comply with international law. This could also include referrals to
the International Criminal Court and sanctions.
Include commitments relating to prevention of CRSV in ceasefire and peace agreements signed
in such conflict zones
Encourage the deployment of more women troops for peacekeeping operations.
Recognise CRSV as a persecution for the purposes of granting refugee status.
Globally, the following actions if taken will ensure that these horrendous crimes are properly dealt
with:

strengthen institutional safeguards against impunity

sexual violence should be an element included in ceasefire and peace negotiations, and
perpetrators should be excluded from amnesty

The soft law of the declaration on ending sexual violence should be made into customary
international law that is more binding

Promoting women's full participation in all political, governance, and security structures, as well
as all decision-making processes.

Topic: The Freedom Struggle its various stages and important contributors /contributions from different
parts of the country

22) Why did the Indian National Congress accept Indias partition in 1947? Critically examine. (200
Words)

Communalism as a political phenomenon amenable to mass mobilization gain ground with rise of
extreme right parties like Hindu Mahasabha (1915), Unionist Party and the Muslim League (Save
nationalist phase in 1920s).

However, their acceptance at large for liberal communalism within the electorate remained law as
evidenced by the 1937 election seat tallies.

However, once a concerted mass politics phase with the cry Islam in Danger by the Muslim League
began along with implicit British support in the Wavell Plan, Cripps Mission and Cabinet Mission to
give the League a veto in constitutional matters.

Congress failed to draw Muslim masses into National Movement during Riots in Bihar, Noakhali,
Calcutta, Direct Action Day. MEANS, prolonged civil war if no partition.

The rift stemmed from several factors

Political ambitions of Jinnah, refusal by the Congress to be called a Hindu Party, lack of
consensus in the Nehru Report and 14 points which made compromise impossible.

Extreme communalism in the citizens

Hindu reaction by Jayakar and Golwalkar

And eventually the hasty Attlees declaration of 20th February, 1947.

Congress though succeeded in building up sufficient national consensus to exert pressure on the British
to quit India, but failed in integrating the nation.
INC had worked hard to secure freedom but it came at the cost of partition. The INC accepted partition
not because of its lust for immediate power but because of prevailing dynamics in India, which made
acceptance the only practical decision:

o Communal carnage raged in India and people were getting butchered in frenzy. The
British government was not doing anything to check communalism. INC leaders felt that
accepting Dominion status would at least give them some power to check further riots.

o The INC had to accept its failure in building a strong base amongst the Muslims despite it
being a secular party. Thus failure was most evident in growing popularity of Jinnah and
sweep of electoral polls in Muslim areas by the League.

o Failure of Cripps Mission, Shimla Conference, Cabinet Mission and the Interim
government convinced INC that Muslim League will not relent. Also British had given
virtual veto to the League at various stages of proposals.

o Even graver threat to a future India's unity and integrity was secession of princely states.
So it was more pragmatic to allow an obdurate leader have his way and focus more on
what could become a deep internal crisis in future India in form of Balkanisation. This is
vindicated by British stand that since INC had conceded its main demand-avoiding
partition- so its all other demands must be met and attempt to retain maximum unity.

o Moreover, Gandhiji was also not able to contain communal frenzy and Gandhi-Jinnah
talks had failed as well, so INC had to bow.

o Sections of INC also believed that once the British left, Indians could become united and
blur the illogical lines drawn on map.

o Sections of INC also believed that if Pakistan was formed once, there will be no more
issues to fight for and peace would prevail.

Conclusion:
League not ready to accept less than Pakistan, and success-failure dichotomy in the Congress,
ultimately led to partition, with consent of Indian National Congress.
Thus, we can see that INC was forced against its wishes to accept Partition of our land, against its
wishes. It had already granted many concessions to Muslim League before.

Topic: The Freedom Struggle its various stages and important contributors /contributions from different
parts of the country

23) In your opinion, which had more impact on Indian masses during the freedom struggle
Gandhijis philosophy of life or his political strategy as a political leader? Critically analyse. (200
Words)

Introduction:

Gandhiji, no doubt, is the mossy shining star in our struggle for freedom due to his technique of mass
mobilization and ability to take all along. His philosophy was essentially premised on truth, non-violence,
self-sacrifice and no to defeatism. He was phenomenally successful in blending the contents of his
philosophy into the fabric of national struggle and thus shaped his stature as a mass leader.

His unwavering belief on own philosophy did the groundwork for launching him to be a
successful leader and galvanize masses:

1. He devised the instrument of Satyagraha which meant non-violent response to British. This often
proved to be a moral fix for colonial government and they faced the dilemma of punishing peaceful
protestors or letting them continue. In any case it was a win-win situation for our struggle.

2. His methods of non-violence attracted masses on a grand scale, who would not have did so in case of
armed struggle and the same is the reason for large scale participation of women.

3. He also focussed heavily on social emancipation of the downtrodden and worked tirelessly for their
cause. This earned him acceptance and respect amongst them.

4. Finally, he believed that British might is enough to crush revolts so the best strategy is to follow
Struggle-Truce-Struggle tactics. This allowed him to wrest concessions from the British and allow
protestors gain some breathing time.

5. Gandhiji was the most prolific and accepted leader of the Indian mass movement. His conduct
both on and off the field of movement inspired many people of his and future generations.

6. His philosophy of life and religion and political strategies had deep impact on the masses that
were interconnected to one another.

7. His concept of truth and non-violence are a universal phenomena and hints to the civilized human
nature that ought to be followed for inner and outer peace. His performance of self control was
found to be immensely useful for idealizing in a person's life.

8. He followed religion and backed it by arguments, but he wasn't critical of other religions, in fact he
espoused that all religions gospelled same principles but only the methods were different. His
care for untouchability found support in the downtroddens.

9. He combined his philosophy into his political movements. He launched anti-untouchability


campaign, and fought for Hindu-Muslim unity.

10. He limited his struggles till the non-violent movements, as he knew the realities and limits of a
mass movement. His idea of penance for the fulfilment of aims and revolting against the unjust
laws, rules and regulations were a broader part of his philosophy.

11. He wanted self-reliance and promoted cottage industries. But this was not related to his political
ability and did not find much place in post 1947 India.

Conclusion:

- Thus it is difficult to isolate his philosophy from his political battles and then analyze the impact on
the masses. Rather, its the combined effect that affected the masses towards protesting against
the British.
- Thus, ultimately it was his ability as a political leader that impacted the Indian psyche and will
continue to do so.

Topic: The Freedom Struggle its various stages and important contributors /contributions from different
parts of the country

24) The national movement based itself on a clear-cut anti-colonial ideology and the vision of a
civil libertarian, democratic, secular and socially radical society. Elaborate. (200 Words)

Introduction:

A variety of themes characterised the Indian National Movement and played a monumental role in
enabling the movement acquire the character of a popular, people's movement.

These include:

(A) The INM arose out of the contradictions between colonialism and the interests of the Indian people.
The nationalist understood clearly that colonization would result in the degeneration of the Indian people
and economy. Accordingly, they pivoted the movement on an anti-colonial ideology, where under the twin
themes of drain of wealth theory and the use of India as a market for British goods formed the pith and
substance of their agitation.

(B) In face of British authoritarianism and despotism, the national movement committed itself to establish
a democracy, where civil liberties were respected and protected. The increased tampering by the British
with the freedoms of press and speech only strengthened the nationalists' resolve. While, the INC
internalised democracy by organised itself on democratic lines, the nationalist went to great lengths to
protect civil liberties. E.g. Lokmanya's declaration of liberty of press and freedom as being his birthright.

(C) In light of the recognition of the diversity of the Indian people, secularism became an integral part of
the INM from its very inception. Though communalism remained a challenge for the nationalists till the
very end, they fought hard to encourage Hindu-Muslim unity and eradicate caste oppression.

(D) Another theme that characterised the INM was its emphasis on pro-poor policies. In this regard, the
national movement was influenced by the Russian Revolution, in the aftermath of which socialism came
to be celebrated as a quality that the Indian Nationalists aspired to achieve. Nationalists demands of
compulsory education, lowering of taxation on poor, reduction of salt tax, land revenue and rent,
improvement in the position of women, the right to vote and abolition of untouchability stem from the
nationalists desire to establish a socially radical society.

(E)The national movement was born out of the anti colonial sentiment which was manifest in the form of
Economic critique of British rule in India and drain of wealth theory by Dadabhai Naroji. The national
leaders were convinced that British rule was the most important cause of
Indias backwardness.

(F)The national leaders were also against the racism of the British rulers.

(G)The national leaders were champions of democracy and were opposed to any form of imperialism,
colonialism or any other form of authoritarian rule over people. Therefore in 1929 at the Lahore session of
Congress Purna Swaraj resolution was passed which clearly stated the objective of complete
independence .They were strong supporters of the liberal democratic values like right to equality and
freedom of press, untouchabilty , women's emancipation etc.

(H)Secularism was a non-negotiable principle of the freedom movement and it was also enshrined in the
constitution of India when it gained independence. The attempts to communalize the country through the
policy of separate electorates for Muslims were opposed by the congress.

(I)The principles of socialism were also incorporated in the freedom struggle which meant not only
political freedom for the masses but also social and economic freedom. It ultimately led to the abolition of
Zamindari after Independence.

Therefore the national movement was anti-colonial, liberal, secular and socially radical also.

Topic: World Physical Geography

25) Describe any three experiments or observations which support the belief that the earth is
roughly a sphere. (150 Words)

The shape of earth has been a curious question which has had its fair share of debate. While it was
considered to be flat many decades ago, scientists have now concluded that it is roughly a sphere. There
are numerous experiments which were done by scientists to increase credibility about earth being
spherical. Some of them are as under:

Satellite Image: Picture taken by satellites orbiting the earth proves that earth is nearly
spherical.
Sighting of Ship: When we carefully observe an approaching ship at sea shore, we get to see
its mast first followed by other parts of ship. This is only possible if the earth is spherical.
Sun Rise and Sun Set: The sun rises in the east first and last in the west. If the earth was flat,
each and every place of the earth would have seen the sunrise and sunset at the same time.
Lunar Eclipse: The shadow casted by earth on moon during lunar eclipse is always circular.
This is only possible when the body i.e. earth is spherical.
Varying Time Zones: Sun rays dont fall with equal intensity over any place on earth, proves
that the earth is not flat. When two different places have different times of the day, with the
opposite hemispheres having night and day at the same time because of earth being roughly
spherical.
Varying Star Constellation: The farther you go from the equator, the farther the known
constellations go towards horizon, and are replaced by different stars. This would not have
happened if the world was flat.
Driving poles on level ground: If 3 poles of equal length are fixed at some distance to each
other, the tip of the poles are not in horizontal level. This is because of the curvature of the earth
proving that earth is not flat.

The circular horizon: The distant horizon viewed from the deck of a ship at sea, or from a cliff
on land is always and everywhere circular in shape. This circular horizon widens with increasing
altitude and could only be seen on a spherical body.

The above evidences prove that the earth indeed, is spherical.


Topic: World Physical Geography

26) Explain why daylight increases as we go polewards in summer in the northern hemisphere.
Also explain the effect of latitude on the length of the day and night. (150 Words)

The earth has two kinds of movements i.e. rotation around its own axis and revolution around sun. These
two phenomenons are reasons for different daylight experience throughout the year.

Increase of daylight towards pole in summer in Northern Hemisphere:

The earth revolves around the sun and its distance from the sun keeps changing due to its
elliptical path which further leads to change in intensity at different place.
The length of daylight depends on the sunrise and sunset timings. Due to earths inclined axis
and revolution, the sun apparently travels to and fro between tropic of cancer and tropic of
Capricorn. During summer in Northern Hemisphere, sun moves towards tropic of cancer.
The circle of illumination remains the same but due to inclined rotation and spherical shape of
earth, the northern hemisphere remains illuminated for a longer time. Sun rises earlier and sets
late at horizon. This explains summer solstice on June 21 in the northern hemisphere, especially
at the North Pole where there is day light the whole day.
At the same time, the South Pole is in complete darkness as no sunlight reaches there.

Effect of latitude on length of day and night

Latitudes play a vital role in varying the duration of days and nights.
Similar latitudes have similar day lengths whereas different latitudes have different day lengths.
Depending on the time of year, length of daylight increases or decreases.
Amount of solar radiation at any location depends on the latitude. Normally, lower latitudes
receive more solar energy and higher latitudes get less solar radiation.
At the equator, length of day and night are almost equal i.e. 12 hours day and 12 hours night.
As we move from equator to North Pole during summers, the length of day time increases. At
North Pole, it becomes 24 hours daylight for six months as it gets continuous insolation.
Exactly opposite pattern is seen in the southern hemisphere. As we move from equator to South
Pole during summer, day length decreases continuously with a complete darkness at South Pole
for six months continuously.

Topic: The Freedom Struggle its various stages and important contributors /contributions from different
parts of the country

27) Write a critical note on the contribution and legacy of Gopal Krishna Gokhale to Indias
national movement. (200 Words)

Introduction
Gopal Krishna Gokhale was one of the most influential leaders of pre-Gandhian phase of Indian national
movements.

He played a monumental role in Indian National Movement. His contribution towards the same was
immense especially in political awakening of the masses and simultaneous social work.

He gave voice to the aspirations of millions of Indians who were looking for freedom from British rule.

Gokhale was an indispensable link in the transformation of the freedom struggle to a more informed one
and for the commitment to bring Indians on parity with the British in terms of knowledge and
administrative skills and to prove that Indians too can govern and can govern more efficiently.

He was a moderate, appealing to logic, attempting to persuade the British with morality and reason.

Contribution

An Outstanding Parliamentarian: His main contribution lies in the way he used to put his
arguments based on study and analysis of situation in the legislature against the British's claims
of prosperity of India under British rule. Rather than rhetoric his emphasis was on actual data and
rigorous method of analysis of this data. Though the Imperial Legislative Council had no real
powers, Gokhale used this platform to shame the colonial government for its indifference to the
plight of the Indian masses. His fiery speeches are masterpieces for they highlighted the misery,
poverty and the drain of wealth from India. Such scholarly conduct by an Indian in the Imperial
Legislative Council inspired all the nationalists.

Mentor to Gandhi and Jinnah: He was acknowledged by Gandhiji and Jinnah as their political
mentor. Gokhale not only supported Gandhi in his battle against apartheid in South Africa, but
also was the one who encouraged Gandhiji to return to India and travel the countryside to
understand the problems and issues faced by the Indian masses.

Social Reform: A fierce advocate of compulsory education, especially women's education,


Gokhale established The Servants of India Society to work towards the upliftment of depressed
and poor. He believed in education for all and fought against other social evils like untouchability,
poverty, casteism, etc.

Economic critic of British Rule: Gokhale argued against free trade with India, while at the same
time promoted the idea of protectionist policies. He also questioned the theory of laissez faire and
advocated a more positive role of the state. Of special importance are his speeches criticizing the
British budgets, which lay bare the underlying intentions of draining Indian wealth to Britain. In a
clear logical style, his speeches used to educate the public about the British unfair and
discriminatory policies and had the British bureaucrats running for cover.

Mass Awakening: As the editor of the English language daily Sudharak, he took to the masses
ideas of social, economic and political reform through his articles.

Legacies
Free press: He was a champion of free press and stood for the freedom of Indian press in face of
severe oppression by the colonial rulers under Vernacular Press Act. Today it is because of his
legacy and other leaders like Bal Gangadhar Tilak that we have a vibrant and free press.
Mass Participation in Governance: He was persistently in favour of greater Indian
representation in elected assembly, judiciary and in academia. It was Gokhale who influenced
thousands of nationalists to participate in Indian freedom fight, including Gandhiji. He inspired
Hindus and Muslims alike with his sharp aptitude toward demand in constitutional way regarding
economic, administration and political power sharing to Indians

Criticism

However, Gokhale was also criticized for being too moderate and for not opposing the colonialists more
vehemently. As a moderate nationalist, he believed in reason and held faith in the prayer, petition method
of appealing to the British Government thereby bringing matters of importance to Indian masses to light.
However, this method of Moderates was not popular especially among youths who took to extremist
philosophies.

Conclusion

Times have changed. India has attained its freedom and its constitutional democracy has
withstood many tests over the last seven decades. Indias social and economic progress requires
continual reform.

For an independent nation with the constitution as its guiding light, Gokhales ideas take on
special meaning. Dedication to duty, a deep moral foundation from which to operate, distinction
between spirituality and religiosity, a strong belief in education as national salvation and an
openness to being persuaded by argument are ideas that will serve us well.

At a time when illiberalism frequently invades our political discourse, it will be wise for us to
remember Gokhales visceral tolerance and his commitment to free speech, constructive criticism
and a free press.

Gandhiji perfectly summed it up. For him G K Gokhale was As pure as crystal, as gentle as a
lamb, as brave as a lion, and the most perfect man in the political field.

Topic: World Physical Geography

28) In what ways sedimentary rock is different from metamorphic and igneous rocks? Describe
the various sources from which sedimentary rocks may be derived. (150 Words)

Rocks are different predominantly due to the way they are formed. Consequently, Sedimentary rock is
different from igneous and metamorphic primarily in how it is formed, and the sources that derive it.

Differences
Formation: Sedimentary rocks are formed from sediments accumulated over long periods.
Compared to this igneous rocks occur deep within the earths surface through dynamic means of
geological processes through either great heat or cooling of lava after an eruption. Also,
metamorphic rocks are formed when original rock is subjected to great heat or great pressure or
both usually in deeper depths of earth.

Stratification: Sedimentary Rocks are distinctly made of layers and are also called stratified
rocks. Igneous and metamorphic rocks do not have such a layered appearance.

Strength: Sedimentary rocks are weak as compared to igneous and metamorphic.

Content: Sedimentary rocks contain fossils of organic forms, or are turned into sedimentary
rocks by decomposing organic forms such as plants or animals. For e.g. chalkis formed from the
remains of corals or shellfish. Peat, lignite and coal are formed from the remains of prehistoric
trees.

Characteristic Features: Owing to the denudation factor, sedimentary rocks are not similar in
look. Some are coarse, others are soft, some powdery others hard. They are not crystalline; they
have sediments cemented together strongly and are much softer than igneous and metamorphic
rocks. Igneous rocks are hard and dark in colour and metamorphic rocks are generally crystalline
and hard in texture.

Various Sources of formation

Sedimentary rocks are most varied in their formation of all rocks. They are formed from sediments or
materials brought by streams, glaciers, wind or even animals and accumulated over long periods. Some
of the ways are as under:

Mechanical formation: This is when sediments are transported and deposited by wind, water,
glaciers and other natural forces of deposition. Overtime the sediments are deposited layer on
layer and get hardened due to mechanical pressure. In this type of formation, extreme variations
in texture, composition and colour are seen. Ex: Sandstone made from sand and quartz
fragments derived from granites.

Organic formation: These can be calcareous or carbonaceous rocks. Calcareous rocks are from
the fossils of animals, etc. When the flesh is decomposed, leaving behind the shells,
etc., this forms the material along with other sediments for calcareous rocks. Similarly,
carbonaceous rocks are from remains of vegetation. E.g.: Calcareous rocks like limestone and
Carbonaceous rocks from vegetative matter compressed under overlying sediments like peat,
lignite and coal.
Chemical formation: These are formed from the chemical precipitations of different solutions.
Ex: rock salt is from strata on sea beds. Gypsum is formed from evaporation of Dead Sea etc.,
which are very alkaline.

However, Sedimentary rocks can undergo structural changes and turn into metamorphic rock, but this
occurs due to larger geologic movements that can alter the conditions of these rocks.
Topic: World Physical Geography

29) The greatest concentration of volcanoes is located in the Circum Pacific region, known as
Pacific Ring of Fire. Examine why and also explain features of these volcanoes. (200 Words)

Introduction

The Ring of Fire refers to the stretch of volcanoes that almost surround the Pacific Ocean, hence called
as Circum-Pacific. The "Ring of Fire" sites are located on most of the Earth's subduction zones having
high seismic activity, or earthquakes, around the edges of the Pacific Ocean.

Roughly 90% of all earthquakes occur along the Ring of Fire. This region stretches from New Zealand,
northwards through Australia, Japan, Korea to the coasts of Alaska and southwards to the tip of South
America. It contains chain of highest number of active and dormant volcanoes on earth.

Reasons

The region is the subduction zone of intense tectonic plate activity.

Earths crust is formed by the interlocking of tectonic plates consisting of oceanic and continental
plates. These plates keep moving due to the forces internal to the earth.

A subduction zone is formed when the heavier ocean plate submerges under the lighter
continent plate. Subduction converts the dense mantle material to buoyant magma which rises on
to the surface and over a large span of time creates a series of volcanoes. That is why most of
the active volcanoes of the world are found in this region.

Some prominent examples are Mt Fuji in Japan, Mt. Saint Helens and Mt. Rainier in the American North
West, Krakatoa in Indonesia, Mauna Loa in Hawaii, Galeras in Colombia and Sangay in Ecuador...

Features

Violent Volcanoes: These are generally violent volcanoes as overriding of plates creates
blockage for molten lava leading to high intensity of eruption. They also give rise to many
occurrences of earth quakes and tsunamis.

Viscous Magma: As the plate overriding the magma is expected to travel larger distance within
the earth's crust. Hence, during the process it becomes highly viscous. Recent example being
Haiti Island.

Active Volcanoes: Some of these volcanoes are most active in the world making this a zone of
severe seismic activity.

Deep Ocean Trenches: Deep ocean trenches are a common feature of the Ring of Fire.

Conclusion
These volcanoes are well known and some of these could be very dangerous because of their
activeness, icy caps and highly populated areas around them with some famous tourist spots.

Topic: Social empowerment; Paper-3 Planning

30) Critically evaluate how five year plans have been able to empower scheduled castes and tribes
in India. (200 Words)

Five Year Plans (FYPs) have been a great way of bringing up the marginalized sections especially at
a time when the country is mired with innumerable problems post-Independence.

The First and Second plan focused on the Agriculture and Industry were a boost to the SCs and
STs where they could get employment and food security

5th Plan - Poverty alleviation programmes of 20 point programme have provided the inclusion
aspect in broad way.
8th Plan became the catalyst for the SC STs Inclusion in the Rural and Urban local
development using 73rd and 74th Amendments. This has been a huge contributor to the
upliftment of the sections as it led to inclusion of women and empowering them with the decision
making capability.

The Tenth Five Year Plan (200207) adopted a multipronged approach for the socio-economic
development of the SCs, including:
1. social empowerment through educational development;
2. economic empowerment through income and employment enhancing avenues;
3. protection through effective implementation of protective legislations and eradication of
occupations such as manual scavenging, etc.

11th Plan - the concept of "Inclusive growth" formally launched and coupled with the benefits of
economic reforms have provided much benefit to the SCs and STs in terms of education,
standard of living, health and nutrition.

12th plan -aims to reduce vulnerability of the marginal societies.

The National Scheduled Castes Finance and Development Corporation (NSFDC) established in
1989, provides financial and other support to beneficiaries for taking up various income generating
activities.

To help the STs in participating in the mainstream development, the government prepared a separate
Development Plan called Tribal Sub Plan for STs in 1976. It was followed by the Special Component
Plan for the SCs in 1978.

All the plans have included the welfare of SCs and STs in their provisions, like Skill India, Taking Urban
amenities to rural India, etc.

EVALUATION:-

1) The decadal literacy rate of SCs has increased from 10.3% in 1961 to 66% in 2011.whereas the
literacy rate of STs has increased from 8.53% in 1961 to 59% in 2011.But still the gap between the
literacy rate of SC and ST and those of general population still persist

2) The health and nutrition status if the SCs and STs continue to be one of the major concerns of the
govt. The infant mortality rate (IMR), child mortality rate and maternal mortality rate are comparatively
higher than that of general population.

3) Despite a reduction in the percentage of population living below poverty line, the poverty among SCs
and STs continue to be very high. The level of rural poverty among STs on all India basis is 47.4% which
is significantly higher than the average population.

No doubt various plans, policies and programmes have brought a perceptible improvement in the socio-
economic indicators, but still we have to go further to bring the backward communities to the mainstream.
Topic: Urbanization, their problems and their remedies.

31) Critically examine how different is Indias Smart City concept from that of Wests or US?
Also examine drawbacks of these concepts.

Source: http://bits.blogs.nytimes.com/2015/04/21/smart-city-technology-may-be-vulnerable-to-hackers

Additional Source 1: http://www.thehindu.com/features/homes-and-gardens/green-living/what-are-


smart-cities/article6321332.ece

Additional Source 2: http://smartcities.gov.in/

Introduction:

A 'smart city' is an urban region that is advanced in terms of overall infrastructure, sustainable
real estate, communications and market viability. It is a city where information technology is the
principal infrastructure and the basis for providing essential services to residents.
Smart Cities focus on their most pressing needs and on the greatest opportunities to improve
lives. They tap a range of approaches digital and information technologies, urban planning best
practices, public-private partnerships, and policy change to make a difference. And they always
put people first.
The conceptualisation of Smart City varies from city to city and country to country, depending on
the level of development, willingness to change and reform, resources and aspirations of the city
residents.
Indias Smart City is about developing an efficient and well planned city with energy efficient
usage of technologies in day-to-day life.
It is unlike Wests Developed nations where it includes redesigning urban infrastructures like in
case of Singapore outskirt cities under development.
The concept of Smart City in India arises because of the need to accommodate 70% of the
population of the country into urban areas by 2050, for which we right now do not have any
institutional as well as structural backing, but there is progress in devising and much work has
been anticipated in the coming decade.

Indias Smart Cities:

Our concept of smart cities is based on renovation rather than on innovation.


We are improving and restructuring our cities based upon new demands.
The support of foreign countries is being sought in developing our cities...
Our cities are not well planned in the past. And hence things become more complicated.
The Smart leadership, smart governance, smart technologies and smart people make smart city.
The resource challenge can be achieved by Public-Private-People Partnership (4Ps). Local
bodies should have courage to take quick and bold decisions and adopt best practices to attract
investments.
The concept include greater role on IT enable devices and better interface between government
and people.
In India, the concept include integrated multi-modal transits system, better e- health & e-literacy
facility, better governance via e-governance & e-panchayat models, IT enabled financial
inclusion, waste management, electricity etc.
Homogeneity in Smart City concept of India & West is that providing amenities in smarter way
than currently available. But as the current level of the amenities available differs the end is also
different.

Wests Smart Cities:

They are well planned since the beginning.


Their smart cities are surrounded by satellite cities they reduce stress on urbanization.
Grid pattern is well followed.
But is not similar to the one adopted by western world where smart cities refer to automation of all
basic services mechanism.
The system in western countries are more integrated and dynamic and user specific, thus more
prone to hackers.
West have a farsighted prediction of future problems so more emphasis was given on
municipality (most of European cites) planning.

Challenges in Indian Approach:

Our local governments have limited say in reshaping our cities.


Local problems may not be addressed effectively due to up-down approach.
We give less emphasis on making of satellite cities and PURA. This adds stress on urbanization
process.
We have unique challenges like large scale slums, limited finances and web like city planning.
There is limited scope for expanding our roads and bridges in the mid of our cities.
If not regulated properly, the amenities would be exploited & dominated by upper class of the
society further enhancing the inequality gradient and the core purpose of the project would be left
out.
Costly as the projects would be funded by profit oriented private sector.
Prone to hackers as many of the projects are IT enabled.
Consumption expenditure would increase.
High rural-urban migration therefore loss of social and cultural values which are better conserved
in self-sufficient villages.
Crime rate will increase.
Example: when after independence and till last decades of 20th century we were focused on
industry but forgot the sustainable sanitary practice and developed rural infrastructure which was
dreamed by not only 'Bapu ' but by many . Although the oldest sustainable smart cities were
found in India (harrapa, mohanjodaro etc) but lessen even learned from them.

Suggestions:

India should focus on:


o Solid Waste management
o Sewage/water management.
o Traffic lights, Eco-friendly building compliancing, Green corridor projects.
o Online facilities for purchasing, paying bill, knowing statuses of various facilities.
o Developing urban infrastructure of communication and interconnectivities.
Despite the apprehensions the concept of smart cities is a novel one given the challenges our
existing cities and challenges posed by climate change. Perhaps with their successful
implementation they would actually become growth poles in the region.
India must prioritize an overall, inclusive development rather than selective, for the latter is
what inordinate focus on smart cities engenders.

Topic: History of the world

32) Write a critical note on the causes and consequences of Vietnam War (1955 1975). Do you
think this War has any lessons for todays aspirants of Superpower status? Comment.

Source: http://www.business-standard.com/article/opinion/hearing-the-helicopters-115042101260_1.html

Introduction:

Preview: The Vietnam War is likely the most problematic of all the wars in American history. It
was a morally ambiguous conflict from the start, ostensibly a war against Communism yet also a
war to suppress nationalist self-determination. The war was rife with paradoxes: in the name of
protecting democracy, the United States propped up a dictatorial regime in South Vietnam; later
in the war, the U.S. military was destroying villages in order to save them. Because U.S.
objectives were often poorly defined during the course of the war, U.S. policy often meandered:
indeed, the United States would Americanize the war only to Vietnamize it five years later.
US President Eisenhower (1953-61) was a supporter of the Domino Theory. After China, North
Korea, South Vietnam will also goes the communist way sooner or later the communist revolution
would reach all other Southeast Asian nations. This will threaten huge US investments in Japan.

Causes of War:

The causes of the Vietnam War were derived from the symptoms, components and
consequences of the Cold War.
It was more of Proxy war between USA and USSR. North aligned with USSR while South
Vietnam with USA.
The government that was established in South Vietnam, with the support of the United States,
refused to abide by the decisions of the Geneva conference with regard to the holding of
elections and the unification of Vietnam.
It came to be increasingly regarded as being under the control of the United States which was
opposed to the unification of Vietnam under the leader ship of the communist party.
The South Vietnamese people led by the National Liberation Front carried on guerrilla warfare.
They had the support of North Vietnam.
The peasants in South Vietnam demanded Land reforms on line of China and North Vietnam
This led to opposition groups demanding a coalition government which would peacefully
negotiate with the North Vietnam for forming a united Vietnam. But proposal was rejected.
When Russia diplomatic relations was in rise Asia and African nation, US were aversive toward
such diplomatic dynamics.
Also, with growing China, India and Russian triad in Asia all toward Communism and Socialism
made US restless as it made US future insecure on the lines of Capitalism outlook.
US supported Southern province while China and Russia supported Northern Province. Usage of
lethal chemical weapon like Agent Orange was used.
After WWII, when Japan withdrew from Vietnam, Ho Chi Minh inspired from Communist China
and Russia established his government named Viet Minh in Northern Vietnam while French setup
their loyalist anti-communist French educated Bao Dai in South Vietnam, later on the conflict rose
and US intervened to support Southern administration(anti-communists) to fight Northern
revolutionaries. In 1975 this war ended with fall of Saigon(now known as Ho Chi Minh City now)

Consequences:

The war led to defeat of US forces and unification of Vietnam.


Vietnam was a biggest blow to American Hegemony. It is no doubt that Superpower needs to
entertain public opinion in an act of aggression both home and abroad.
US suffered heavy losses and faced pseudo defeat under hands of guerrilla warriors.
United Vietnam under socialist regime formed, self efficacy and esteem of Vietnamese rose high.
In US, President Kennedy was assassinated and opinion splits on continuing such expensive
war. This led to no costly future adventure by US in name of Cold war.
Large scale destruction of life and property due to advanced weaponry and bacteriological
warfare of the US
Guerrilla tactics and brave front put forth by the Vietnamese in retaliation
Worldwide protests against the US Govt and opposition even within, isolated US over the war
The emergence of Vietnam as a united and independent nation in the face of the armed
opposition of the greatest power in the world.
Although US didnt learn lessons well and still continue to adopt similar attitude in resolving
issues in West Asian countries.

Lessons for aspirants today's Superpower Status like India, China etc:

In a world of Nuclear and multi nodes of power, War is not the solution to any problem.
Lesson for Superpowers that they cannot use smaller countries for increasing their sphere of
influence.
Military intervention is a costly and the least effective mechanism to solve issues.
Engaging in active diplomacy for resolving conflicts.
Use of Global forum like UN for resolving conflicts rather than resorting to unilateral action.
Countries must be persuaded rather than coerced or forced to follow externally imposed
solutions.
Independence and self-government are best attained by negotiations.

Conclusion:

Need is for co-operation and mutual respect for each other for the overall development of whole
world.
Rather than creating sphere of influence, countries should strive for development of all.
Vietnam War is a classic example which shows that in the international chessboard, there are no
permanent friends or foes.
Wining the hearts and minds of people should be the main focus rather than military force for
solving a political conflict.
Topic: Poverty and Developmental issues

33) If you are asked to enumerate next set of Millennium Development Goals for least developed
and developing countries to achieve between 2015 and 2030, what would be your priorities and
goals? Also highlight, in your opinion which institutions might play crucial role in helping
countries achieve these goals and why.

Source: http://www.business-standard.com/article/opinion/jim-yong-kim-the-final-push-to-end-extreme-
poverty-115042101261_1.html

Introduction:

The Millennium Development Goals (MDGs) are the world's time-bound and quantified targets
for addressing extreme poverty in its many dimensions-income poverty, hunger, disease, lack of
adequate shelter, and exclusion-while promoting gender equality, education, and environmental
sustainability.
MDG has achieved very good result in last 15 years and responses it received from Developed
and funding nations are good.
We must remember that we already achieved great success in primary education, eliminating
extreme poverty, gender equality, checking IMR and MMR, control of AIDS. But not much
progress is done on environmental side.

Millennium Development Goals (MDGs):

Eradicate Extreme Hunger and Poverty


Achieve Universal Primary Education
Promote Gender Equality and Empower Women
Reduce Child Mortality
Improve Maternal Health
Combat HIV/AIDS, Malaria and Other Diseases
Ensure Environmental Sustainability
Develop a Global Partnership For Development

Goals for LDC and Developing Countries (2015-2030):

Universal Primary and Secondary Education with Quality as primary index for measuring
educational standards. Most of countries have worked out in enrolment students for primary
education but failed to provide quality education in initial years in previous MDGs.
Complete eradication of Diseases like Cancer, HIV/AIDS, TB, and Leprosy. Better
immunisation programmes like vaccination are needed for preventing deaths due to these major
diseases.
Reduction of IMR and MMR to less than 10 per thousand and per one lakh people respectively.
Sustainable development across all states with new technology for renewable energy
generation, Increasing Green Energy Fund, Promoting low carbon strategies, A Uniform index for
measuring Air, Land and Water pollution for all LDC and Developing states.
Encouraging Multilateral and Regional cooperation for building Peace in war torn states like
Middle East, Ukraine etc. Separate forums that represent concerns of LDCs in international
Arena.
Equipping young population with some minimum skill set regime and setting up vocational
training centre. Reducing unemployment.
Cheaper medical facilities and medicines.
Clean and Sanitation surroundings.
Increasing the green cover.
Ground water recharging.
River and water body revival and cleaning.
Spreading the use of IT and Communication in all parts of world.
Reducing racism, casteism or any kind of discrimination.

Requirements to achieve these goals are:

Adequate finance to undertake projects.


Transfer of technology to LDC and developing nations
Building human capacity to implement projects to achieve desired goals.

Institutions which play imp role:

As we can observe in last decadal development that, developing nations holds promising future
and potential to bring positive change in world.
So, organisation like ILO can monitor Skilling youth programme, AIIB+NDB can provide soft loans
for small projects in LDN and Developing nations in fast way as compared to World bank, G20
can come up with mandates for every central bank of nations to focus on some priority sectors so
that bank provide loans easily for those sectors.
CITES, UNESCO can play role in conserving biodiversity, nation heritage, and science.
International and Regional Institutions like IBRD, ADB, UNIMCEF, WHO, UNICEF can play
greater role in achieving these goals as these institutions can provide funds for development,
promote Cooperation in education social and cultural aspects, help in reconstruction of war
affected states, work for betterment of children and women etc.

Topic: Post-independence consolidation; Modern Indian history from about the middle of the eighteenth
century until the present- significant events, personalities, issues

34) Discuss the contribution of Moti Lal Nehru to the constitution development. (150 Words)

Moti Lal Nehru was one of the most prominent leaders of India's independence movement. When Indians
were being bullied by British as people who were unable to govern themselves ,Moti Lal Nehru took up
the challenge and prepared a draft ,famously known as "Nehru Report ".The salient features of the report
range from introduction of fundamental rights ,to universal adult franchise, federal structure of
government and equal status to women in civic and political affairs, etc .When county got independence
,this report proved to be a light house for constitution makers and without surprise we find several
features of our constitution having its roots firmly articulated in Nehru report .

Most of its recommendations have been declared as basic features of constitution by the Supreme Court
of India.
1. Federal system of government with Viceroy reduced to nominal figurehead and a responsible
bicameral legislature at the centre. Todays Parliamentary system in India is exactly like this.

2. Rejection of communal/separate electorates and weight age to minorities. Also, time bound reservation
for minorities. Today also we have joint electorates and reservations for scheduled castes and tribes.

3. He defined Citizenship and Fundamental Rights. He advocated for freedom of speech and expression,
associations, no state religion and religious freedom, free elementary education. Almost all of these rights
have found their way into our Constitution.

4. He advocated for woman empowerment. Today our political setup is mulling over reservations to
women.

5. He advocated for Hindustani as official language and use of English for some limited time only.
Reorganization of states on the linguistic basis. This had to be done post 1947.

6. Apart from this, the Nehru also demonstrated how legislatures could be innovatively used to enforce
executing accountability through debates, discussions, questions etc. These practices have become a
major essence of our parliamentary system.

7. Nehru also contributed to setting up of independent Secretariat under the President of the Central
Legislative Assembly which ensures independence of presiding officer and is incorporated under Article
98 of Indian Constitution.

5. He advocated for Hindustani as official language and use of English for some limited time only.
Reorganization of states on the linguistic basis. This had to be done post 1947.

Besides, other proposals were related to the then India like demand for Dominion Status within the
Commonwealth. So many of his proposals were incorporated in Constitution (1949) and even in Karachi
Resolution of 1931. And his vision of India turns out to be very close to reality.

Bipan Chandra, India Since Independence, Chapter 4

Topic: Developmental issues

35) Water security for rural India is measured on the following parameters: quantity,
accessibility and quality. Based on these parameters, critically analyse the situation of
water security and evaluate the performance of schemes in ensuring it in rural areas. (200
Words)

Water security attributes to three parameters viz. Quantity, Accessibility &Quality. Failing on anyone one
of three, water security is thought of being compromised
India has 4% of water resource of the world and has support 16% of population and 15% of livestock.

1. Quantity: - When water source is inaccessible then it directly affects the quantity of water. Also in
cases of public piped water demand is many folds in comparison to supply.
2. Accessibility
- Location of water resources is a serious concern as lot of time and energy is wasted in accessing
water
- This leads to huge opportunity loss for nation
3. Quality
- River water is polluted from waste and open defecation
- mixing of effluents, fluoride and arsenic are other pollutants affecting quality of groundwater

Remedies and scheme effectiveness

Central and state government through several schemes have improved accessibility. E.g.:
watershed development, drought prone area development, water harvesting and application of
scientific techniques National water mission Namami Gange National rural drinking water
programme National water policy of 2002 & 2012
Pollutants in river water are controlled and effective monitoring is performed through pollution
control boards
Many borewells, taps and other sources of water are provided in rural areas with 500 m proximity
which serve many families.
India has successfully achieved MDG of providing 80% of rural population with more accessible
water. But this is a small milestone and still much has left to achieve. Quantity
with depleting groundwater resources and rainfall shortage in arid and semiarid area, water
security is a serious issue especially for rural population
A recent NSSO survey estimated about 14% of the population face shortage of drinking water
and even more percentage of population have serious accessibility issue

Although there has been a marked progress in terms of water security, much need to be done to provide
access to each and every citizen.

Live mint

Topic: urbanization, their problems and their remedies.

36) Critically analyse the objectives and design of recently launched Smart Cities Mission and
the Atal Mission for Rejuvenation and Urban Transformation (AMRUT). Do you think such
schemes will be effective in addressing fundamental problems of urbanization? Comment.
(200 Words)
The smart cities mission is aimed at recasting the urban landscape of
the country by making cities more liveable and inclusive, besides
driving economic growth.

AMRUT is aimed at rejuvenating cities which adopts a project approach to ensure basic infrastructure
services relating to water supply, management, storm water drains, transport

Its design and objectives include


better sanitation
better public transport
power supply
solid waste mgmt and traffic mgmt
better use of IT services for daily use and better connectivity
use of e-governance
Greater accountability of govt.
for this govt. also announced 48000cr recently and 50k cr for AMRUT...
The design/ pathway for the completion of the combined projects will include central
funding under AMRUT to the projects sanctioned under JNNURM and not completed,
besides the 1 lakh crore sanctioned for them. Each selected city under the ambitious
scheme would get central assistance of Rs 100 crore per year for five years.

Smart City aspirants will be selected through a city challenge competition intended to link
financing with the ability of the cities to perform to achieve the mission objectives.

Special Purpose Vehicle will be created for each city to implement Smart City action plan.
The SPV will be signed with the urban local body, state government and the Centre for
implementation of the project.

It will be implemented through "area based" approach consisting of retrofitting,


redevelopment, pan-city initiatives and development of new cities.

To tackle this problem of Urban areas including metros are marred with problems like slums, pressure on
infrastructure, lack of basic amenities, women safety etc. ,Govt. has launched SCM, AMRUT. But the
guidelines issued are not clear on few areas:

1. Institutional & legal framework is lacking to support modernization process. National, State, local level
Institutions needed to be formed.
2. "SMART" concept is rotating around technology but technology base is needed to be strengthened e.g.
Cyber security
3. Focus should be on strengthening local, regional approach e.g. Transport system needed to be inter
linked based on local demands.
4. Professionalism, training local body institutions is needed. Funds should be raised based on the
capacity of local bodies. It will bring accountability, efficiency.
5. Preventive rather than curative approach should be used, as per the census of India large number of
semi urban areas is growing up. They are needed to be focused and strengthened to serve connectivity
between rural &urban areas.

These schemes will certainly help in combating the problem of urbanization; as such projects will provide
smart solutions accommodating the migrants in the city. In such solutions, the example of Singapore can
be taken, where the slums were eradicated by building eco-friendly low cost buildings, where the migrants
in India can live while working at nearby sites.

Thus SCM and AMRUT can take India to a new path of development

The Hindu

Business Standard

Topic: Post-independence consolidation and reorganization within the country.

37) How did the task of unifying post Partition India and princely states take place? Give a
critical account. (200 Words)
The lapse of paramountcy created a tremendous problem. At the time of independence more than565
Indian states (40% land) was under the rule of Indian princes. Most of them did not want to join any
dominion as according to them paramountcy cannot be transferred and they would be independent
after 15th august 1947. Nehru stated that India would not recognize the independence of any state,
but the problem remained.

The significant steps taken by Patel to prevent a possible balkanization of the country
included setting up of the new "States Department" in the place of the "pro-princes - Political
Department" headed by V.P. Menon.
An instrument of Accession was drafted requiring the signatory states to hand over to the
Union, defence, foreign affairs and communications while exercising autonomy in all other
fields.
Many states decided to join India due to its secular character and people's mood and also
due to assurance, convincing, cajoling and coercion by the duo but many states nourished
dreams of independence.
.
Case of Junagarh, J&K, and Hyderabad:- These three states were reluctant to join Indian
union so special steps were taken
a) Junagarh:- Indian troops were send and plebiscite was organised.
b) J&K:- Maharaja Hari Singh did not want to join any dominion but Azad Kashmir force
attacked on J&K. So under extra ordinary circumstances he signed Instrument of Accession
c) Hyderabad:- Nawab of Hyderabad signed standstill agreement with a belief that he would
get enough time to strengthen army; but when Indian armys(Operation Caterpillar)
marched he surrendered in 3 days.
Strict action and army interference forced their accession. Initially all states were to sign
Instrument of Accession whereby they surrendered Defence, Foreign Affairs and
Communications to the government of India. Later they signed supplementary treaties and
were formally merged into states to very closely resemble India as we know today.

However, the entire process had certain issues:

Privy Purse was guaranteed constitutionally to the princes in lieu of their surrender if power,
villages, and land and railways networks. While its burden was not much on the Indian
exchequer, it was abolished by legislative action. This is sought as a betrayal of trust reposed
by Sardar Patel.
Certain issues like Kashmir were taken to United Nations even after Kashmir had signed
Instrument of Accession to India. So it was then an internal security issues and involvement
of UN is regretted even today. Article 370 has always been debatable.

While the choice to join whether India or Pakistan must have been of the king, popular mood
of people was always prioritised. Also states chose or were forced to choose on religious
lines
. Pakistan's army left no stone unturned in planning geostrategic warfare against India, like
using Junagadh as a military post. Also it was accused of human rights violation in
Baluchistan and Kashmir. Similar situation happened in Hyderabad by Razakars.

Thus, balanced use of persuasion, threat and force, created a contiguous Union of India after
partition.

Bipan Chandra, India Since Independence, Chapter 6


April Paper-2

Topic: Parliament and State Legislatures structure,

1) Why does India needs two houses in its Parliament? Justify. (200 Words)

NCERT, Class XI, Indian Constitution at Work, Chapter 5

Indian Constitution has adopted Westminster model of Bicameral legislature with names Lok Sabha and
Rajya Sabha

The justifications for the Second Chamber are as under:

a. Control on political compulsions of ruling party in LS.


b. Scope for the talented and experts to scrutinise legislations, without having to fight elections.
c. Check on hasty and ill-conceived legislations.
d. Lessening the burden of law-making of Lower Chamber.
e. Provides enabling atmosphere for greater executive accountability.
f. Debates on wide range of public issues
g. Representative of federal units in a federation

Rajya Sabha is important

As a Revising Chamber
As a Federal Chamber
As a Deliberative Chamber
As a Chamber of Continuity
A Chamber not Concerned with Government Formation
As an Effective Smaller Chamber
As a Chamber Securing Executive Accountability
As a Chamber of Ventilating Public Grievances

Advantages of bicameral legislatures include their capacity to:

Formally represent diverse constituencies (e.g., state, region, ethnicity or class);


Facilitate a deliberative approach to legislation;
Hinder the passage of flawed or reckless legislation; and
Provide enhanced oversight or control of the executive branch
Frequent change in governments will not affect Rajya Sabha unlike Lok sabha and there will be
continuity of consensus to some extent in parliament on issues which will help in decision making.
Rajya Sabha reflects a federal character by representing the units of the Union. While, Lok Sabha
represents spirit of people.
They check on abuses of democracy in their own defined jurisdictions.
Two houses check the tendency of Government to take populist measures under an unstable
political Scenario.
RS allows entry of expert and professional in legislative making process, who may not want to
face elections.
The emergency situation when Lok Sabha is dissolved, only Rajya Sabha can take vital decisions
in interest of nation and States. Example Kargil War (1999).

The makers of our Constitution envisaged the role of the Rajya Sabha along three axes: as a
legislative chamber of elders discussing, revising or delaying legislation as per need; as an
institution where interests of the states of the Indian Union could be projected and
safeguarded and; as a deliberative chamber where greater and diverse experience is brought
to bear on questions of significance. The essence of democracy is participatory governance.
Experience the world over has shown that this ensures responsiveness and transparency.

http://rajyasabha.nic.in/rsnew/publication_electronic/2nd_cham_ip_rol_sta_2009.pdf

2) The legislature in parliamentary system ensures executive accountability at various stages:


policy making, implementation of law or policy and during and post implementation stage.
Discuss the instruments that the legislature uses for this purpose. (200 Words)

NCERT, Class XI, Indian Constitution at Work, Chapter 5

Accountability of the executive to Parliament is the very essence of parliamentary democracy. An


overview of the genesis of the parliamentary system clearly indicates the gradual process by which the
legislature has systematically asserted its authority. The control that the legislature exercises over the
executive stems from the basic principle that the legislature embodies the will of the people and,
therefore, it has the authority to supervise the manner in which the legislation enacted by it is
implemented by the executive.

What Is Accountability?

Accountability means obligations to account for all actions or omissions one does. The word
accountability in the context of political institutions has over the years moved beyond the traditional
definition of keeping accounts. Accountability entails following components:

Transparency

Representativeness

Responsiveness and

Power to sanction misdeeds

The concept of Accountability in Political institutions is an essential component to the ideology


establishing a responsible government.
Accountability entails two kinds of obligations:

Explanatory accountability: obligation to give account i.e. to answer, disclose, explain or justify.

Remedial Accountability: obligation to account for action or inaction i.e. to be answerable for the
misgovernment and subsequent and appropriate changes. This is retrospective or a posteriori.

Of

Mechanism for Enforcing Accountability of Executives

The accountability of the Executive is direct, continuous, concurrent and day-to-day.

1. Procedural Devises to Raise Matters of Public Importance:

The following are devised by the Houses of the Parliament under the authority conferred by the
Constitution under Article 118(1)

The Question Hour

Half-an-Hour Discussions

Adjournment Motions

No-day-Yet named Motion

Calling Attention Notices

Private Members Resolutions

Rule 377 & Special Mention

Petition

No-Confidence Motion:

Parliamentary form of government entails an institutional framework where the executive functions by
virtue of confidence relationship to any parliamentary majority. Thus, the House of People can introduce a
motion of no-confidence against the government which on being sustained would result in the fall of the
government. Reasons for loss of confidence against the government need not be stated.

Parliamentary Committees:

Parliamentary committees are of utmost importance at most of the business is carried out in committees.
Two types of committees Standing committees and ad hoc committees comprising of members of the
Houses with due representation from all political parties proportional to their strength in the House are
mechanisms which ensure the functioning of the Parliament effectively. There are Department related
standing committee to look in to demands of various ministries.

Constitutional Checks on the Executive:

The Constitution of India has envisaged Parliamentary form of government where the Parliament is
supreme. Certain provisions in the constitution authorises the Parliament to exercise control over the
government. This can be broadly categorized into the following:
Policy & Administrative Matters:

The Council of Ministers are collectively responsible to the House of People by virtue of Article 75
(3). Dismissal from the office by withdrawal of support by the legislature is the only sanction for enforcing
responsibility.

Parliamentary Control Relating To Legislative Matters:

Participation in Legislation: Generally bills are introduced by ministers relating to their departments. It is
very rare that a Private member bills are introduced and subsequently passed by the houses. These bills
in the second reading referred to the committees are thoroughly scrutinized and any evidence or
documents required for such scrutiny by the committee is provided by the executive.

Ordinances: The President can proclaim ordinances in the absence of House in session. However, the
ordinance lapses if the Houses pass a resolution disapproving it.

Emergency Provisions: The power of the executive to declare emergency at the State level is subject to
parliamentary control.

Parliamentary Control Relating To Financial Matters:

Parliaments control over financial matters is demonstrated below.

Consolidated Fund of India:

No money in the Consolidated Fund will be available to the Government for any purpose unless it obtains
a grant from the Lok Sabha; further the House cannot authorize a grant unless there is a demand by the
Government specifying the need and amount which it plans to spend.

Contingency Fund of India:

The Parliament has established a Contingency Fund under the disposal of the President with a view to
meet unforeseen expenditure of an emergent nature. Any amount expended form the Contingency Fund
is returned to the Fund from the Consolidated Fund of India after approval of the Parliament.

Excess Grant & Supplementary Grant:

It can obtain Excess Grant or Supplementary Grant only by approaching the House of People for such
grant.

Effectiveness of Accountability Mechanisms:

All these mechanisms have their own drawbacks.

Numerous legislations not debated at all in the floor of the house.

Stalling of the proceedings by the opposition to gain political mileage, protests etc have resulted
in complete inefficiency in functioning of the House.

In reality, the effectiveness of no-confidence motion is crippled because of limited alternatives


available to replace the incumbent government as this would be possible only in cases of minority
government or defections.
Members of Parliament work under the dicta of their political bosses and hence as the ruling party
commands majority on its own, effectiveness of Parliament is greatly crippled. Legislation and
ordinances are passed without undergoing rigorous review.

The objective of through financial scrutiny is managing resources effectively and efficiently.
However, the audit report of the CAG has clearly demonstrated how the States have overspent
and obtained excess grant for a humongous amounts.

Conclusion

Accountability of executive to parliament forms basic structure of parliamentary form of government. On


one hand it keep executive from surpassing their authority, at the same time it allows sufficient space to
them to come up with citizen centric legislations.

Topic: Structure, organization and functioning of the Judiciary

3) What do you understand by Judicial Independence? How can the independence of judiciary be
provided and protected in India? Critically examine. (200 Words)

NCERT, Class XI, Indian Constitution at Work, Chapter 6

Judicial independence is an idea that has both internal (and normative) and external (or institutional)
aspects. From a normative viewpoint, judges should be autonomous moral agents, who can be relied on
to carry out their public duties independent of venal or ideological considerations. Independence, or
impartiality, in this sense is a desirable aspect of a judges character. But judges are human, and the
things they must decide can matter greatly to people. Therefore, Judges should be provided institutional
shields against the threats or temptations that might come their way.

MEANING THE INDEPENDENCE OF THE JUDICIARY

The meaning of the independence of the judiciary is still not clear after years of its existence. Our
constitution by the way of the provisions just talks of the independence of the judiciary but it is nowhere
defined what actually the independence of the judiciary is.

The primary talk on the independence of the judiciary is based on the doctrine of separation of powers
which holds its existence from several years. The doctrine of separation of powers talks of the
independence of the judiciary as an institution from the executive and the legislature.

The other meaning of the judicial independence can be found out by looking at the writings of the
scholars who have researched on the topic. Scholars have followed the constituent mechanism (i.e.
what constitutes the judiciary) to define the independence of the judiciary. Scholars try to define judiciary
by talking about the independence of the judges which constitutes judiciary. Therefore the independence
of the judiciary is the independence of the exercise of the functions by the judges in an unbiased manner
i.e. free from any external factor.

So the independence of the judiciary can be understood as the independence of the institution of the
judiciary and also the independence of the judges which forms a part of the judiciary.
Shetreet in his work tries to explain the words Independence and Judiciary separately, and says that
the judiciary is the organ of the government not forming a part of the executive or the legislative, which is
not subject to personal, substantive and collective control, and which performs the primary function of
adjudication.

The final outcome that can be derived from Shetreets writings is that the independence of the judiciary as
an institution and the independence of the individual judges both have to go hand in hand as the
independence of the judiciary as an institution is not possible without the independence of the individual
judges and is the institution of the judiciary is not independent, there is no question of the independence
of the individual judges.

NEED FOR THE INDEPENDENCE OF THE JUDICIARY

The basic need for the independence of the judiciary rests upon the following points:

To check the functioning of the organs: Judiciary acts as a watchdog by ensuring that
all the organs of the state function within their respective areas and according to the
provisions of the constitution. Judiciary acts as a guardian of the constitution and also
aids in securing the doctrine of separation of powers.
Interpreting the provisions of the constitution: It was well known to the framers of the
constitution that in future the ambiguity will arise with the provisions of the constitution so
they ensured that the judiciary must be independent and self-competent to interpret the
provision of the constitution in such a way to clear the ambiguity but such an
interpretation must be unbiased i.e. free from any pressure from any organs like
executive. If the judiciary is not independent, the other organs may pressurize the
judiciary to interpret the provision of the constitution according to them. Judiciary is given
the job to interpret the constitution according to the constitutional philosophy and the
constitutional norms.
Disputes referred to the judiciary: It is expected of the Judiciary to deliver judicial
justice and not partial or committed justice. By committed justice we mean to say that
when a judge emphasizes on a particular aspect while giving justice and not considering
all the aspects involved in a particular situation. Similarly judiciary must act in an
unbiased manner.

THE INDEPENDENCE OF THE JUDICIARY

Many provisions are provided in our constitution to ensure the independence of the judiciary. The
constitutional provisions are discussed below:

Security of Tenure: The judges of the Supreme Court and High Courts have been given the
security of the tenure. Once appointed, they continue to remain in office till they reach the age of
retirement which is 65 years in the case of judges of Supreme Court (Art. 124(2)) and 62 years in
the case of judges of the High Courts (Art. 217(1)). They cannot be removed from the office
except by an order of the President and that too on the ground of proven misbehavior and
incapacity. A resolution has also to be accepted to that effect by a majority of total membership of
each House of Parliament and also by a majority of no less than two third of the members of the
house present and voting. Procedure is so complicated that there has been no case of the
removal of a Judge of Supreme Court or High Court under this provision.
Salaries and Allowances: The salaries and allowances of the judges is also a factor which
makes the judges independent as their salaries and allowances are fixed and are not subject to a
vote of the legislature. They are charged on the Consolidated Fund of India in case of Supreme
Court judges and the Consolidated Fund of state in the case of High Court judges. Their
emoluments cannot be altered to their disadvantage (Art. 125(2)) except in the event of grave
financial emergency.

Powers and Jurisdiction of Supreme Court: Parliament can only add to the powers and
jurisdiction of the Supreme Court but cannot curtail them. In the civil cases, Parliament may
change the pecuniary limit for the appeals to the Supreme Court. Parliament may enhance the
appellate jurisdiction of the Supreme Court. It may confer the supplementary powers on the
Supreme Court to enable it work more effectively. It may confer power to issue directions, orders
or writs for any purpose other than those mentioned in Art. 32. Powers of the Supreme Court
cannot be taken away.

No discussion on conduct of Judge in State Legislature / Parliament: Art. 211 provide that
there shall be no discussion in the legislature of the state with respect to the conduct of any judge
of Supreme Court or of a High Court in the discharge of his duties. A similar provision is made in
Art. 121 which lay down that no discussion shall take place in Parliament with respect to the
conduct of the judge of Supreme Court or High Court in the discharge of his duties except upon a
motion for presenting an address to the President praying for the removal of the judge.

Power to punish for contempt: Both the Supreme Court and the High Court have the power to
punish any person for their contempt. Art. 129 provide that the Supreme Court shall have the
power to punish for contempt of itself. Likewise, Art. 215 lays down that every High Court shall
have the power to punish for contempt of itself.

Separation of the Judiciary from the Executive: Art. 50 contains one of the Directive Principles
of State Policy and lays down that the state shall take steps to separate the judiciary from the
executive in the public services of the state. The object behind the Directive Principle is to secure
the independence of the judiciary from the executive. Art. 50 say that there shall be a separate
judicial service free from executive control.

Independence of judiciary recent concerns

Retired Judges taking post of Governor and other posts

NJAC Bill Some Judges have raised concern about interference of executive in selection of
Judges and resulting loss in Independence of judiciary-especially because of veto power.(Matter
is highly debatable and sub-judice -cant take stand on it)

Whenever there is a mention of the independence of the judiciary, there is always a concern about the
latent dangers of the judicial independence and there arises the importance of Judicial Accountability.
Importance of the independence of the judiciary was long ago realized by the framers of the constitution
which has been accepted by the courts by marking it as the basic feature of the constitution. Judicial
Accountability and Judicial Independence have to work hand in hand to ensure the real purpose of setting
up of the institution of judiciary.

http://mulnivasiorganiser.bamcef.org/?p=482

Topic: Structure, organization and functioning of the Judiciary

4) How is judicial activism related to the protection of fundamental rights? Has it helped in
expanding the scope of fundamental rights? Illustrate with suitable examples. (200 Words)

NCERT, Class XI, Indian Constitution at Work, Chapter 6

http://www.sci.nic.in/speeches/speeches_2009/judicial_activism_tcd_dublin_14-10-09.pdf

Blacks Law Dictionary defined judicial activism as a philosophy of judicial decision-making whereby
judges allow their personal views about public policy, among other factors, to guide their decisions

The post-emergency period (1977-98) is known as the period of Judicial Activism because it was
during this period that the Courts jurisprudence blossomed with doctrinal creativity as well as processual
innovations

A great transformation in the judicial attitude towards the safeguard of personal liberty has been noticed
after the infamous 1975 national emergency.

Instances of judicial Activism in protection of Part 3 of Constitution:

Orders issued on Cases relating to admission to professional colleges


o
Orders to close down factories and to tone up pollution levels
o
Monitoring the quantum of rainfall and flow of water into irrigation canals
Calling for data on deforestation
o
Directing CBI to inquire into scandals, crimes, and custodial deaths, setting deadline for
compliance
o
Courts have set aside the decisions of Speakers disqualifying legislators under the anti-
defection law
o
Expanding scope of Article 21 for example: education to children, life with dignity,
establishing green benches, etc

Some of the rights recognized and expanded through judicial activism are as follows:

Right to live with Human Dignity


Right to Livelihood
Right to Shelter
Right to Privacy
Sexual harassment of working women: volatile of Art 14 and 21.
Right to Health & Medical Aid
Right to live in a Pollution Free Environment
Ban On Smoking In Public Places
Compensation for Violation of Article 21
Right of Prisoners

Some prominent cases

Maneka Gandhi Vs Union of India (1978)-expanded FRs, discovery of inter connections between
Article 14, 19 and 21.Right to travel and go outside the country is included in the right to personal
liberty guaranteed under Article 21

Shreya Singhal vs Union of India -The Supreme Court, on March 24, 2015, struck down Section
66 A of the Information and Technology Act, which allows police to arrest people for posting
offensive content on the internet. The bench said the public's right to know is directly affected by
Section 66 A and the Section clearly affects the right to freedom of speech and expression
enshrined under the Constitution of India.

CRITICISM

While delivering a lecture on Canons of Judicial Ethics, Chief Justice of India S H Kapadia, cautioned
fellow judges, against judicial overreach. The Judiciary should not act as a super-legislature. Validity of
our decisions should not rest on popularity.

Judicial Populism:

Activism can easily transcend the borders of judicial review and turn into populism and excessivism.
Activism turns into populism when doctrinal effervescence transands the institutional capacity of the
judiciary to translate the doctrine into reality, and it is excessivism when a court undertakes
responsibilities normally discharged by other co-ordinate organs of the government.

CONCLUSION

Judicial activism is not an aberration. It is an essential aspect of the dynamics of a constitutional court. It
is a counter-majoritarian check on democracy. Judicial activism, however, does not mean governance by
the judiciary. It also must function within the limits of the judicial process. Within those limits, it performs
the function of legitimizing or, more rarely, stigmatizing the actions of the other organs of government.
Truly, judicial activism is the oxygen of the rule of law.

Topic: Welfare schemes for vulnerable sections of the population by the Centre and States and the
performance of these schemes
5) Write a note on government of Indias recently launched Mission Indradhanush programme.
Also critically evaluate how successful have been similar programs in the past. (200 Words)

The Hindu

Immunization is one of the most cost effective public health interventions since it provides direct and
effective protection against preventable morbidity and mortality. It has been a major contributor in the
decline of under-5 mortality rate from ~ 233 to ~63 (per 1000) in last five decades in India. However,
vaccine preventable diseases (VPDs) are still responsible for over 5 lakh deaths annually in India. This
underlines the need for further improvement.

Ministry of Health and Family Welfare (MOHFW) has launched Mission Indradhanush on 25th December
2014 with the aim of expanding immunization coverage to all children across India by year 2020. The
Mission Indradhanush, depicting seven colours of the rainbow, targets to immunize all children against
seven vaccine preventable diseases namely Diphtheria, Pertussis, Tetanus, Childhood Tuberculosis,
Polio, Hepatitis B and Measles. In addition to this, vaccines for JE (Japanese Encephalitis) and Hib
(Haemophilusinfluenzae type B) are also being provided in selected states. Immunization is the key to
protect children from life threatening conditions that are preventable. As per MOHFW, immunization
coverage in India has increased from 61% to 65% only from 2009-2013. So, MOHFW has decided to
intensify its efforts in immunization though this mission.

Objectives of Mission Indradhanush


To cover 201 high focus districts in the first phase of year 2015. These districts have nearly 50% of all
unvaccinated or partially vaccinated children. Out of these 201 districts, 82 districts lie in just four states
of India namely, UP, Bihar, Madhya Pradesh and Rajasthan. Nearly 25% of the unvaccinated or partially
vaccinated children of India live in these 82 districts of 4 states. Furthermore, another 297 districts will be
targeted in the second phase of year 2015.

Strategy of Mission Indradhanush


The government has planned to conduct four special vaccination campaigns between January and June
2015. All vaccines are already available free of cost under universal immunization programme in India.
Under this mission, government plans to intensify its efforts and thus increase accessibility of these
vaccines to all the children of India

Similar programmes in Past History:

India and China were two countries where "some form of inoculation" was practiced even before
16th century. However, modern immunization developed in India in 19th century, parallel to the Western
world.

Initial years saw considerable investment in research and development (R&D) in vaccines and about
fifteen vaccine institutes were established beginning in the 1890s.

India, on its part, launched its first vaccine exactly 50 years back: BCG in 1962 as a part of National
Tuberculosis Program.
WHO launched the Expanded Programme on Immunization (EPI) in 1974 to bring vaccination against six
prominent diseases to many underserved areas.EPI was launched in India in 1978. Initially, it included
BCG, DPT (3 doses) and typhoid vaccine; OPV was added the next year. In addition to 3 primary doses
of DPT and OPV, 2 boosters at 1.5 years and 5 years were also given to cover children upto 5 years of
age. Achieving self-sufficiency in production of vaccines was also a part of program.

In 1985, the program was converted into Universal Immunization Program (UIP) with a goal to cover all
eligible children in the country, immunization of all pregnant women with TT and to improve quality of
services.

The focus remained on 4 vaccines (BCG, DPT, OPV and Measles) and 6 diseases only till 2006. It was
only after 2006 that new vaccines like hepatitis B, second dose of measles and Japanese Encephalitis
vaccines were introduced

Pulse Polio Programme (PPP) was started initially in Delhi in 1994 and was extended to the whole
country in 1995.

Recently, 11 centers across the country have been identified for laboratory supported surveillance for
vaccine preventable diseases with special reference to potential vaccines in collaboration with the Indian
Council of Medical Research (ICMR

Availability of new safe and effective vaccines against major killers like diarrhea and pneumonia
necessitated serious deliberations on further strengthening of National Immunization Programme. The
Ministry of Health and Family Welfare formulated National Vaccine Policy in April 2011 to provide broad
policy guidelines and framework to guide the creation of evidence base to justify need of research and
development, production, procurement and quality assessment of vaccines under UIP [10].

Current Status of RI in India

After putting up a good show in its first decade (1985-95) with coverage of RI reaching 70-85%, there has
been deterioration in the performance of UIP.

The coverage of different vaccines has fallen by 15 to 20%.

Surveys carried out during National Family Health Survey (NFHS) I, II and III and by independent
agencies such as UNICEF, have revealed that the coverage levels may be lower by as much as 15-40%
compared to reported levels of coverage in the UIP.

Interstate disparity has been observed with only few state good in implementation.

Birth dose administration is still a challenge in all states.

About 7.6% of children between the ages of 12-23 months have not received any vaccine

There is a large inter-state variation in the coverage of RI. As per CES 2009, there are 4 states (Goa,
Sikkim, Punjab and Kerala) >80% of children between 12-23 months of age are fully immunized. This
percentage is < 50% for another 5 states (Bihar, Madhya Pradesh, UP, Nagaland and Arunachal
Pradesh) .Six states with high population contribute to 80% of 8.1 million unimmunized children in the
country, 52% of the total unimmunized children reside in Uttar Pradesh and Bihar alone .

Where are we Lacking?

The barriers to achieve 100% immunization coverage

There are a number of reasons why India lags behind its per capita GDP counterparts in vaccination rates
(compare to Bangladesh, where 82% of children are fully vaccinated by age two).

Huge population with relatively high growth rate is a barrier in itself. Approximately 27 million children are
born in India each year the largest birth cohort in the world but less than 44% receive a full schedule
of vaccinations .To reach each and every one of such a huge cohort every year is obviously a daunting
task.

Geographical diversity (snow bound/ hilly areas, deserts, tropical forest areas, remote island territories),
cultural diversity (with various religions, languages, traditions, beliefs and customs) and Political instability
("coalition" governments, "politically sensitive areas" like Naxal/terrorist-affected areas) are some
problems that are rather unique to India and make the task more complex.

Reaching out to mobile/migrant population (that is a significant proportion of population in some states) is
another challenge. Special efforts are needed to identify and reach some pockets of low immunization
that are still there in many states.

Coverage Evaluation Survey of UNICEF found that reason for partially immunization/ non-immunization
was "did not feel the need", "not knowing about the need" and "not knowing where to go for vaccination".
This means that lack of awareness is one great barrier to achieve cent percent immunization coverage.

Low levels of education negatively impact health-seeking behavior.

Adverse events following immunization (AEFI) even when these are shown to be unrelated to a vaccine,
have been widely reported in the media and have contributed to a culture hostile to vaccination in certain
communities .

Issues on "supply side" that pose challenge in achieving high RI rates.

They include inadequate delivery of health services (supply shortages, vacant staff positions, lack of
training); lack of accountability, inadequate supervision and monitoring; lack of micro-planning at district
level; general lack of inter-sectoral coordination and lack of coordination between state and central
governments resulting in missed opportunities to improve immunization coverage and quality.
Falsification of data and over-reporting of rates are other big concerns as they give false sense of security
and interfere with proper planning

The above barriers are further compounded by a weak VPD surveillance system in the country. There is
lack of disease burden data on many important VPDs in India that results in the perception that the
disease is not important public health problem. Further, there is utter lack of diagnostic tools for certain
VPDs. Lack of baseline surveillance data also is a bottleneck in monitoring the impact of vaccination.
Focusing on polio eradication exclusively while neglecting UIP ("de-linking" of UIP from Polio Eradication
Initiative: PEI) by the policy-makers has also led to deterioration of performance of UIP. It has been
suggested that house-to-house rounds of PPP have also made certain sections of society "dependent" on
health workers:

At Government level, resource constraints and competing priorities need careful planning and policy-
making. The fund allocation for RI is still less than desirable. There is need to step-up spending on
vaccination front and health.

What is needed?

The road ahead

Political and Bureaucratic Will

Such an elaborate program obviously cant succeed without political and bureaucratic support at all
levels.

The existing National level "Inter Agency Coordination Committee" (ICC) needs to increase its focus on
routine immunization.

A public-private partnership between GoI, NTAGI, Indian Academy of Pediatrics (IAP), Indian Medical
Association (IMA), development partners, ICDS, Ministries of Railways, Education and Defense, and key
NGOs involved with immunization and State representation should be strengthened

The program managers need to ensure and monitor that funds are appropriately released in a timely way
for operational costs.

Ensure an uninterrupted supply of all antigens to state level through a vaccine stock management system
that includes annual forecasting and wastage rates.

Central level should provide technical support and resources for states to develop evidence based social
mobilization plan.

In specific low performing States, a district / block based operations research scheme could be
considered and scaled up if successful. All hard-to-reach and urban slum areas should be reached at
least four times per year with RI or catch ups.

Effective IEC Activities

Since lack of awareness has been found to be main barrier, focus should be on increasing demand for
vaccination by using effective IEC and bringing immunization closer to the communities. The
immunization services provided at the fixed sites should be improved. There should be better monitoring
and supervision, and district authorities should be made accountable for the performance of RI in their
district.

Induct Innovative Methods to Improve RI


The number of immunization delivery points especially in rural and remote areas having poor access to
health facility, should be increased. Immunization booths should be constructed at every locality in urban
areas particularly in slums, and local municipality board member should be made accountable for their
performances. Large and varied cadres of volunteers, including, for example, local registered medical
practitioners, quacks, pharmacists, chemists and retired nurses and other health personnel can be
recruited to offer immunization services. Proper training including maintenance of cold chain and basic
minimum education on vaccines must be imparted to all of them. Complete immunization should be made
mandatory to get admission in school by appropriate legislation. Incentives in cash and kind may be
offered to those families having fully immunized kids.

Proper Monitoring of the Program

Although vaccination is a medical intervention, the vaccination program, UIP, is not simply a medical
modality it is a management-dominant modality. The managerial, administrative and governance-
related inadequacies need to be addressed on a priority basis. The need to monitor the progress of
control of diseases under UIP has not been realized; one element of the poor performance of UIP is
precisely this lack of monitoring

The fact that some states have been performing very well shows that we have the potential to achieve
excellence. The success factors (in well-performing states) and failure factors (in poorly-performing
states) must be identified and addressed with passion to reach our goal at the earliest

"Reach Every District" strategic approach can be recast as "Reaching Every Community" [16].

Develop Effective Surveillance Systems

Establish a surveillance system for all important childhood infectious diseases.

Adverse Effects Detection, Reporting and Redressal System

A functional real-time AEFI and post-marketing surveillance system should be there.

Regulatory and Ethical Issues

Regulatory capacity of the country need to be strengthened and needs to become reliable.

Support to Indigenous Vaccine Industry

Most low-cost traditional vaccines are now produced by vaccine manufacturers in developing countries.
Currently about 43% of the global UIP vaccines come from India, and the Serum Institute is the
worlds leading producer of measles vaccines .Though, the current national vaccine policy seems
supportive of Indian vaccine industry with liberal support from government-owned institutions like
department of biotechnology (DBT), National Institute of Immunology (NII), department of science, etc still
there is need to further empower Indian vaccine sector to meet the indigenous demand of vaccines.

Introduction of newer vaccines: There are about 23 new/ improved vaccines that are now available or
would be available soon. Although inclusion of a new vaccine in national schedule adds the cost of
vaccine and logistics to the health budget of a country, it also results in savings by reduction of the
disease burden. Thus, the decision to include a new vaccine in national schedule needs careful scientific
analysis regarding all the issues involved, ranging from policy issues (whether introduction of the new
vaccine is in sync with immunization policy of the country) to technical and programmatic issues (whether
implementation of the decision is technically feasible)]. New vaccines should not be introduced at the
expense of sustaining existing immunization activities. Instead, the introduction of a new vaccine should
be viewed as an opportunity to strengthen immunization systems, increase vaccine coverage and reduce
inequities of access to immunization services. Merely making the vaccine available in few pockets, for
certain sections and for limited duration will not have any impact at national level. The equity needs to be
ensured so that the vaccine reaches to the section of the society who needs it the most.

Integrated Delivery of Health Interventions

Strengthening of immunization systems in such a way that they support and integrate with other
preventive health services like providing vitamin A supplementation, deforming, growth monitoring,
distribution of insecticide-treated bed nets, etc. offer the opportunity to create synergies and facilitate the
delivery of services to bolster comprehensive disease prevention and control. Incorporating immunization
into integrated primary health care programs may also facilitate social mobilization efforts, help generate
community demand for services and address equity issues. The strategy of child health days, led by
UNICEF, has also helped to promote RI

Research and Development (R&D)

Investment in research and development is bound to pay rich dividends. A large number of vaccine
products are currently in the pipeline and are expected to become available in near future. According to
recent unpublished data, more than 80 candidate vaccines are in the late stages of clinical testing. About
30 of these candidate vaccines aim to protect against major diseases for which no licensed vaccines
exist, such as malaria and dengue. The benefits of development of better vaccines for existing VPDs like
tuberculosis, typhoid and influenza, increasing the ambit of VPDs by development of vaccines against
mass killers like HIV, malaria, dengue fever, RSV, enteric pathogens like E.coli, Klebsiella, etc,
development of more thermostable vaccines (so that need of maintenance of cold chain is obviated) and
development of alternative delivery of vaccines, like mucosal vaccines/ edible vaccines [36] cannot be
overemphasized.

Other initiatives

Apart from all the above mentioned measures, there is an urgent need of establishing a separate,
independent department of Public Health. All the community health projects should be supervised and run
under this department rather than in the form of different vertical programs. There must be prioritization of
the need of a particular vaccine based on the disease burden data of that VPD rather than on the
availability of the product in the international market. There must be clear cut transparent guidelines on
the policy of introduction of newer vaccines. And in the last, efforts should be made to devise guidelines
to regulate hitherto unregulated private vaccine market. There must be a code of conduct for marketing
vaccines in private sector.
Conclusion

Immunization has delivered excellent results in reducing morbidity and mortality from childhood infections
in the last 50 years. Although the success has not been as spectacular as in developed world, the fact is
we have eradicated small pox, and is polio-free country. There has been substantial reduction in the
incidence of many VPDs... We now need to step up our efforts to strengthen all components of UIP
(vaccination schedule, delivery and monitoring, and VPD/AEFI surveillance), overcome all barriers
(geographical, politico-social and technical) and invest heavily in R&D to achieve immunizations full
potential and a healthier Nation.
Topic: Welfare schemes for vulnerable sections of the population by the Centre and States and the
performance of these schemes

6) Write a critical note on the objectives, design and implementation of the Rashtriya Swasthya
Bima Yojana (RSBY) programme. (200 Words)

http://www.themenplattform-ez.de/wp-content/uploads/2012/07/2012_01_15_Health-care-in-India.pdf

The Hindu

Introduction
Rashtriya Swasthya Bima Yojana (RSBY) or national health insurance scheme
The scheme is today the worlds largest medical insurance programme covering over 120 million poor
people in the country
RSBY has been launched by Ministry of Labor and Employment, Government of India to provide health
insurance coverage for Below Poverty Line (BPL) families.
the objective of RSBY is to provide protection to BPL households from financial liabilities arising out of
health shocks that involve hospitalization.
Beneficiaries under RSBY are entitled to hospitalization coverage up to Rs.30,000/- for most of the
diseases that require hospitalization.
Pre-existing conditions are covered from day one and there is no age limit.

Unique Features of RSBY


RSBY provides the participating BPL household with freedom of choice between public and private
hospitals and makes him a potential client worth attracting on account of the significant revenues that
hospitals stand to earn through the scheme.
The scheme has been designed as a business model for a social sector scheme with incentives built
for each stakeholder. This business model design is conducive both in terms of expansion of the scheme
as well as for its long run sustainability.
A beneficiary of RSBY gets cashless benefit in any of the empanelled hospitals. He/ she only needs to
carry his/ her smart card and provide verification through his/ her finger print. For participating providers it
is a paperless scheme as they do not need to send all the papers related to treatment to the insurer.
The key feature of RSBY is that a beneficiary who has been enrolled in a particular district will be able
to use his/ her smart card in any RSBY empanelled hospital across India. This makes the scheme truly
unique and beneficial to the poor families that migrate from one place to the other. Cards can also be split
for migrant workers to carry a share of the coverage with them The use of biometric enabled smart card
and a key management system makes this scheme safe and fool proof separately.
The World Bank and the United Nations have hailed it as one of the best health insurance schemes
around the globe.
Ashtray Swasthya Bima Yojana (RSBY) is an effective instrument for providing basic health cover to
the poor.

Criticism of RSBY
variation on the implementation part in different states.
RSBY provides a maximum cover of Rs 30,000 to each BPL (below poverty line) household in the
country. However, the insurance is available only for inpatient care (or hospitalization).
In the absence of a strong primary and secondary health infrastructure, it increases the tendency
among patients to get hospitalized at the first instance. As an off-shoot, it leads to increased frauds.
Various studies and reports have shown how empanelled hospitals did not have adequate facilities, or
how hospitals conducted unnecessary hysterectomies on patients to make easy money through the
cashless insurance schemes.

Higher disease burden on the poor is for outpatient care and drugs rather than hospitalization. Out-of-
pocket expenses for the poor have been increasing. But the scheme insurance is available only for in-
patient care (or hospitalization).

Hospitals frequently complain that the rates fixed by the government are not viable.

Most private hospitals try to cherry-pick patients and diseases that can fetch them more money through
treatment.

RSBY does not aspire to create a health infrastructure. It only aspires to put money in the hands of the
patient so that he may not have to spend from his pocket if he finds a hospital.

Suggestions
a. in case of any dispute fast and easier mechanism with 24x7x365 customer care should be required.

b. more publicity informs the beneficiaries regarding enrolment and benefits of the scheme.

c. If in case the original smart card lost/ damaged issuing the new card process has to be fast.

d. Claims and settlement process should be fast.

What we need is we have to create a strong pubic healthcare system where treatment should be
provided free of cost to all BPL households instead of providing health insurance coverage of 30,000rs to
private hospitals.

Topic: Issues relating to development and management of Social Sector/Services relating to Education,
Human Resources.

7) Recently the Karnataka Legislative Assembly adopted a bill that enforces Kannada as medium
of instruction for primary education in the state. Do you think teaching in mother tongue should
be encouraged in schools? Is English harming regional languages? Critically examine. (200
Words)

The Hindu
The Wardha scheme of Education of 1937 has prescribed for mother tongue as medium of instruction for
children. Mahatma Gandhi was in favour of linguistic division of state because he considered education in
regional language as necessity to propagate education among masses.

Advantages of Mother-tongue education

Studies have found evidence to suggest that mother-tongue education leads to better academic
performance.

The 1996 Hague Recommendations Regarding the Education Rights of National Minorities proposed that
in primary school, the curriculum should ideally be taught in the minority language (mother tongue)

Socio-linguist Thabo Ditsele in his studies has shown that teaching younger children in a language that is
not their mother tongue appears to disrupt cognitive ability and interferes with the learning process.

Mother-tongue education might also actually make the acquisition of a second language easier.
Mother-tongue instruction and foreign-language instruction are mutually supportive. Many linguists argue
that when it comes to learning a second language it is crucial to first have a solid foundation in ones first
language.

Furthermore, being educated in their mother-tongue can give children a sense of pride in their heritage
and identity.

Arguments Against

English has emerged triumphant as the global language. It is the language of the labour market.

Learners must take English right from when they start school so that they dont struggle having to
adopt it at a later stage.

Mother-tongue education could lead to linguistic divisions in the society.


.
What can be done?
Multilingual teaching can be adopted. Children should be encouraged to be multilingual and teachers
should be adept at teaching multilingual students.

We need to involve community members with diverse language skills in formal school and train teachers
with varying language capacities and levels of education to be effective in Mother Tounge Based-
Multilingual classrooms. As knowledge develops, we must get better at communicating research findings
so that practitioners, policy makers and donors are informed and motivated by evidence about how the
potential of MTB-MLE can be harnessed to achieve Education for All.

Topic: Important International institutions, agencies and fora- their structure, mandate.

8) The United Nations was intended to be a temple of peace, but this once great global body has
been overrun by the repressive regimes that violate human rights and undermine international
security. In the light of its recent decision and the way it functions, critically comment on the
statement. (200 Words)

The New York Times

http://www.cfr.org/human-rights/global-human-rights-regime/p27450

The above criticism often comes from human right activist, historian and diplomats, supporters of Israel,
who accuses UN of anti-Zionism, anti-Semitism and alleged support for Palestinian militancy.

Reasons given by them

UN has fewer than half democracies. They accuse these undemocratic regimes that deny democratic
rights to their people, to abuse the United Nations democratic forums to advance their interests.

Commission on the Status of Women has single out Israel for human right violations, ignoring countries
like Iran and Sudan.

United Nations Human Rights Council has membership includes Saudi Arabia, Qatar and Venezuela
nations with restricted human rights and right to speech. These governments stand in judgment for
human right violation in other counties.

In 2007, Sudan chaired a committee overseeing human rights even as its president, Omar Hassan al-
Bashir, was being investigated for crimes of genocide and crimes against humanity in Darfur, for which
the International Criminal Court later issued arrest warrants.

UN is accused to pass 20 resolutions critical of Israel in 2014-15 whereas very few were passed against
North Korea, Iran, and Syria

Analysis of criticism

Some of the charges above seem to be true. But every organization, especially dealing with so many
nations and complex situation cannot work without problems.

Selection of countries to various UN forums are done by voting in General assemblies, so countries in UN
Human right council or Commission of Women are elected ones. Even except few aberrations their
decision seems to be right and just in most cases.

UN has carried out criticism of human right in every corner of world from Syria to Sri Lanka to North
Korea. Multiple resolutions against a country show its unwillingness to adopt measures suggested by UN
for peacekeeping.
Decision in all forums of UN is generally taken on consensus, so even if few rogue countries are selected
they may not be able to take any wrong decision.

UN Reform

To answer this criticism way forward is to reform the UN to make it more representative of todays global
reality.

Security Council may be expanded to admit more permanent member so that crucial decisions are broad
base and have got backing from all major counties of world.

Topic: India and its neighbours

9) Examine the events and factors that helped Bhutan become a democracy. (200 Words)

The Hindu

http://www.ipcs.org/pdf_file/issue/RP24-Marian-Bhutan.pdf

Bhutan has transitioned from an absolute monarchy to a multi-party democracy. The development
of Bhutanese democracy has been marked by the active encouragement and participation of
reigning Bhutanese monarchs since the 1950s, beginning with legal reforms such as the abolition of
slavery, and culminating in the enactment of Bhutan's Constitution. The first democratic elections in
Bhutan began in 2007, and all levels of government had been democratically elected by 2011.

Factors & Events

POLITICAL MODERNIZATION AND COMMITMENT TO DEMOCRACY UNDER THE THIRD AND


FOURTH KINGS

While the first two kings of Bhutan concentrated primarily on securing the power and legitimacy of the
monarchy and initiated careful modernization and development, while simultaneously sustaining the
traditional foundations of society like Buddhism, it was the third Druk Gyalpo Jigme Dorji Wangchuk who
initiated key processes of democratic institutionalization. Under his rule, from 1952 to 1972, the system of
government changed considerably.

Influenced by the decolonization of South Asia, the founding of the Peoples Republic of China and its
annexation of Tibet, the King became convinced that modernization had to be extended to the political
sphere

In the economic sphere, he abolished slavery and serfdom and initiated extensive land reforms which
have ensured that today all Bhutanese citizens living in rural areas of the country own their share of land.

Bhutan gradually moved away from international isolation, formally applying for UN membership in the
late 1960s and being granted full membership in 1971.
The judiciary and legislative is separated from the executive. The High Court was created, but the King
remained the highest appellate authority and nominated the judges to the court

As the Bhutanese tradition of consensus had to be translated into the changing political system, the three
main sources of legitimacy for the political system - the clergy (though not being engaged in Bhutanese
politics since the introduction of monarchy, but still an influential part of traditional Bhutanese society and
culture); the bureaucracy (being both the instrument of the monarchy in the development process and
the incubator of the modern elite); and the people, were represented in the National Assembly

GROSS NATIONAL HAPPINESS AS AN IDENTITY CREATING CONCEPT

First introduced in the 1980s by King Jigme Singye Wangchuk, it basically combines modernization and
conservatory policies. Its essence is that economic development in particular and modernization in
general, has to take into consideration the wellbeing of the people not only in material terms, but also
spiritual and social terms.

ACCELERATED DEMOCRATIC REFORM

The radical changes within the structure of the executive combined with the reintroduction of the
monarchs responsibility vis--vis the parliament and by extension, the people.

In 1999, television and the Internet were introduced, modern sources of information that had been
prohibited before.

In 2002, the first sub-national election was conducted.

The Anti-Corruption Act established an independent Anti-Corruption Commission to pursue cases of


corruption and build confidence within the administration.

The Judicial Service Act provided for the establishment of the Judicial Service Council, which is now
responsible for the allocation of positions and promotions within the judicial branch.

King announced that the first democratic elections at the national level would be held in 2008 and that he
would then abdicate in favour of his son, the Crown Prince.

In 2006, the Electoral Commission of Bhutan was inaugurated and it started to prepare for the general
elections in 2008 through voter education, promotion of political awareness and organizing the conduct of
the elections.

On 14 December 2006, King Jigme ingye Wangchuk, the fourth Druk Gyalpo, announced his immediate
abdication.

In June 2007, the ban on political parties was lifted to allow for their formation in the face of the upcoming
elections.

In December 2007, the first elections to the upper chamber of the parliament took place.
. On 6 November 2008, Jigme Khesar Namgyel Wangchuk was crowned the fifth Dragon King of Bhutan
and head of state.

THE NEW SHAPE OF THE BHUTANESE POLITY

The Constitution declared that Bhutans form of government shall be that of a Democratic Constitutional
Monarchy in which sovereign power [will] belong to the people of Bhutan

Fundamental rights and civil liberties, the cornerstone of liberal democracies in theory and practice, are
granted

In March 2008, the kingdom of Bhutan, an often invisible Shangri-La tucked away strategically in the
Himalayas between India and China, became the worlds youngest democracy. An absolute monarchy
gave way to a constitutional monarchy, a new Constitution mandating a parliamentary democracy was
adopted, and, for the first time, the people of Bhutan voted, on the basis of universal suffrage, to elect a
new Parliament consisting of a National Council or Upper House with 25 members, and a National
Assembly or Lower House with 47 members.

Topic: e-governance- applications, models, successes, limitations, and potential;

10) Write a critical note on Union governments eBiz project. (200 Words)

Live mint

The E-Biz portal is a platform aimed at single window clearance of all approvals required for setting up a
business in India.

The platform will bring together 26 central departments and some state ministries for improving ease of
doing business in India. E-Biz is not just considered important to the larger objective of Make in India
getting operational but also in bringing about transparency in business dealings.

BENEFITS:

1. Moves from department-centric to consumer-centric approach, thus indicating the change in the
governments approach to business.

2. The shift also means that there will be less red tape and power dealings associated with getting
approvals for setting up business. It will thus cut down both nepotism and corruption.

3. Inordinate delays caused due to the large number of stakeholders concerned will be cut down as the
portal facilitates single window clearance.

4. Enables entrepreneurs to submit one integrated application for multiple licenses and make a
consolidated payment for the various licenses that they require

5. Reduces the number of procedures and correspondingly the cost and time taken for obtaining
approvals for starting and operating a business.

6. A world-class G2B portal that enhances Indias business competitiveness through a single, service-
oriented, event-driven interface for all G2B interactions.
7. Integrated G2B Services across Central, State & Local Government& across all geographies in India.

8. Currently, government of India has decided 10 pilot states in which they are going to execute the plan
and will develop later in other states of India.

9. The eBiz portal will provide total 11 union government services to reach its purpose.

10. Provide all information regarding business 24X7 on a single portal

Challenges

1. E-Biz too has to focus on seamless implementation through proper maintenance and absolute data
security.

2. The task of horizontal integration of 26 departments is a tall task and the government is rushing against
time to complete it. Challenges

3. Technological capabilities of govt. staff

4. Business process re-engineering

5. Change in management structure.

6. Concerns over security, fear of spam.

7. Seamless implementation of project

However, the project faces issues like slow integration of services across various departments. This must
be completed on a priority basis. Design of software for the project has been awarded to Infosys for 10
years. Developer options can be diversified and auctioned next time to avoid monopoly.

Topic: Structure, organization and functioning of the Executive and the Judiciary

11) Today the Supreme Court of India is an active intervener in policy formulation. In the light of
some of recent judgments of the Supreme Court, critically analyze the statement.

Live mint

Though it is the legislator who is responsible for the formulation of the laws and executive for public
policies, but the judiciary plays a complimentary role by judging the efficacy of these laws and policies
and also giving directives on them if they fail to meet the constitutional benchmark or public aspirations. In
this context SC has become an active Intervener in policy formulation:-

It can be understood from:-

1. SC actively uses PIL to give directions to the executives that help in formulation in welfare
policies and environment policies.

e.g. - Delhi govt. was given a directive to make shelter homes, GOI given directive to clean rivers.
2. Similar directives are given on administrative and development matters.

e.g. - The Administrative policy of formulation of civil services board was based on SC directive,
also SIT on black money. In development sphere examples include Interlinkages of rivers

3. Most importantly by quashing laws and executive policies SC rows the direction of Public policies.

e.g. - The transparent auction policy in coal and 2G were based on SC quashing earlier auctions.

4. SC questioning govt on the policy implementations like that of clean Ganga or SIT formation for
black money

5. Cancellation of coal block allocation: SC acted on basis of CAG report stating proper mechanism
were not taken to provide impartiality in allocation, leading to gains to few selected, while it is duty
of govt to make sure the benefit reaches to people through equitable distribution.

Legislature and executive is bound by the rule of law, principle of constitution and are obliged to provide
form policies in the interest of public and with good governance being the prime motive.

The instances of bypassing the rule of separation of powers should be limited to some specific cases
only, when the govt. needs to be awakened regarding an issue, which has long been ignored and needs
reforms. Otherwise, the policy formulation process is for the legislature to follow, and should be left to
them.

Topic: Structure, organization and functioning of the Executive and the Judiciary

12) The idea of an Indian judicial service on the lines of all-India services like the IAS or IPS has
been considered in the past and abandoned. Do you think is there a need to reconsider this idea?
Discuss.

The Indian Express

The Indian judiciary is not just afflicted with manpower and Infrastructure crunch but it is the qualitative
manpower problem with further augments the delay and quality of judgments at lower levels.

The idea of an Indian judicial service on the lines of all-India services like the IAS or IPS can help in
resolving some problems of Indian Judicial system:-Indian judiciary with present separate recruitment by
respective higher courts in collaboration with state PSC's and collegiums system witnessed some
problems like:

1. High pendency cases


2.Comparatively less salary
3.Un healthy Judge to population ratio

The judicial system of India needs rejuvenation both in process and in structure. The structure part could
be addressed by incorporating an all India service on the lines of IAS IPS.
If implemented in letter and spirit, the scheme will have its own distinct advantages:

1) Recruitment of judges right from the entry level will be handled by an independent and impartial
agency like the UPSC through an open competition thereby ensuring fair selection of
incumbents.
2) Attract bright and capable young law graduates to the judiciary, who otherwise prefer immediate
remunerative employment in the government and the private sector.
3) For the subordinate judicial officers it would ensure equitable service conditions besides
providing them with a wider field to probe their mettle.
4) The total indifference exhibited by the state governments calls for a national approach to remedy
the malaise affecting the lower rungs of judiciary;
5) The creation of such an all India service will also help build a talent pool out of which judges can
be selected for the HCs and the SC resulting in an overall improvement in the quality of judiciary
in India.
6) In addition, the objective of introducing an outside element in court benches can be achieved
better and more smoothly because a member of an all-India judicial service will have no mental
block about inter-state transfers. It will enrich their experience and make them better judges.
7) There is an urgent need for the judiciary to revisit the manner in which cases are heard and
decided.
8) Ceiling on the time each party can be given to adduce its evidence, documentary or human.
9) There should also be a limit on the number of adjournments each side can seek.
10) Consider disallowing certain types of disputes from admission all together.

However these progressive steps need to be complimented with more synchronized efforts of
Bar, Bench and legislators to transform the criminal justice system in India, where rule of law
prevails over everything else.

Topic: India and its neighborhood- relations.

13) Do you think Chinas ambition to become a global superpower is peaceful? Critically
comment.

The Hindu

The dramatic rise of China as a world power has changed the dynamics of the global politics, with the
world no longer being unipolar in the truest sense.

- Consistent with its "stated" policy of peaceful rise, China is expanding its soft power by going in for huge
infrastructural development in Indo China and also in South Asian countries.

- China aims to be a driver of the global economy through its large foreign exchange reserves and
the setting up of AIIB is a step towards that.
- Projects like Maritime Silk Route and "One Belt, One Road" policy aims to expand Chinese
presence in the region through economic means.
However, China's history creates some doubts especially in the mind of South Asian power like
India.
- Chinese acts to establish hegemony in the South China Sea and East China Sea gives rise to
scepticism regarding its so called peaceful presence in the Indian Ocean.
- The setting up of ports in the Indian Ocean Countries can well be its extension of its military
strategy in the form of "String of Pearls".
- Its annual increase of more than 10% on its Defence Budget and the Second highest Defence
Budget after U.S., the Docking of its Nuclear Submarine on a Sri-Lankan port from where South
India is well within its reach etc. seem contrary to its announcement of Peaceful Rise
- India should cooperate with China in the region in a mutually beneficial way. However, we should
also keep a critical watch on Chinese activities and sound the alarm whenever there are signs of
any visible aggression.
- Presently there are series on incidents like disputed waters of South China Sea, Declaring no fly
zone, Daulat Beg Oldi incident with India, Frequent ceasefire violation with India, huge investment
on its defence forces etc.

Though it is undeniable fact the china is bound to change global power balance but it should be done in
more accommodating way, giving due respect to small fishes, giving enough space to its neighbors
without bringing in its vested interests and with a tendency to take all together towards a real win-win
situation. For India it is wise to move cautiously, as every move on the chessboard of international politics
will invite its counter move and not missing boarding the leaving train at the same time. The victory lies in
taking right steps at right time.

Topic: India and its neighborhood- relations.

14) Critically analyze Indias nuclear doctrine Vis a Vis Pakistan and Chinas nuclear doctrines.
Also examine how the West views nuclear situation in South Asia.

The New York Times

Indias nuclear doctrine

India's nuclear program started in the pre-independence era with scientists realizing potential of nuclear
energy for peaceful purposes. Though this gave India the technological capability to produce nukes, an
internal debate preceded such production.

Currently India and China have a well-developed nuclear doctrine which states:

Credible Minimum Deterrence: This advocates holding sufficient nuclear arsenal that would act as
deterrence to other states to launch attack on them.
No-First Use Policy which states use of nuclear weapons only as second strike as part of retaliation
strategy
Massive retaliation with unacceptable damage
Non Use of nuclear weapons on non-nuclear states

Because of such policies India was able to secure a NSG waiver in 2008 and was successful in signing
various bilateral agreements, and as declared, India's as a responsible nuclear power is currently working
towards peaceful uses of nuclear energy.
Pakistan on the other hand has no defined nuclear doctrine and has even stated that it might use nukes
first against India in a conventional war due its lack of sophistication in such arms. It continues to expand
and modernize its nuclear arsenal only as a response against India.

It is fastest growing state in term of the nuclear arms, will double her capacity in next decade.

Pakistans entire program has been built in retaliation to India's nuclear program, whereas China started
its own program as a response to the nuclear blackmail policy of US during the Korean War.

Wests view of nuclear situation in South Asia

China and India are less of a headache to the West as can be seen from China's membership in the
P5+1 and India's NSG waiver and bilateral agreements.

But Pakistan continues to be a nightmare because:

- It does not have a nuclear doctrine

- It is politically unstable with the military in a dominant position and infested with terrorist organizations
like Al-Qaeda and Taliban increasing the fear of such weapons of mass destruction coming into the
hands of terrorists.

Topic: India and its neighbors

15)Many advocate that a solution to the Kashmir dispute would necessarily have to include
Pakistan. Do you agree with this view? Substantiate.

---The dispute between India and Pakistan regarding the Kashmir area has been ongoing since the
independence time, with both claiming the rightful ownership.
---Also, it can be conveniently said that the final solution for the problem can only be achieved through
constructive and regular talks between both the nations.
---Several stages of talks, which were stalled several times due to wars and ceasefire violations, have
already taken place between various Indian and Pakistani PMs since Independence, with no fruitful
solution coming to the fore.
---But along with the will of the govt. of both countries, it is also necessary to win the trust of the Kashmir
citizens, who have been through a long stretch of difficult times, with the regular terrorist attacks, cross
border firing and AFSPA.
---Several good will measures have been undertaken by the Indian govt., like Amritsar Lahore Bus
service, trade initiatives, SAARC games, etc. but all this has not been able to get a substantial
breakthrough in achieving a compromise on the border problem.
---The solution proposed by former PM Indira Gandhi during the 1972 bilateral talks in Shimla with the
former Pak President Zulfikar Bhutto has been seen as the most pragmatic till date. It was to turn the LoC
into an international border, which will legally give the PoK to Pakistan and the rest of Kashmir will be
Indian Territory. But Pak had recently suffered the 1971 defeat, and was not ready to take any solution
coming from Indian side.
---The recent visit the Indian FS to Pakistan, is being seen as a move to break the hiatus on the Kashmir
issue and an initiative to restart the bilateral talks, so as to frame a solution for the dispute and provide
relief to the hundreds of kashmiris who have suffered all these years for no fault of their own.

IRRITANTS

If Pakistan was one of the reasons for the current state, it has to be a part of the solution. Any solution
without consulting Pakistan will leave it dissatisfied and continued infiltration of jihadis and terror strikes
will make a political solution meaningless.

A political solution means a settlement acceptable to all parties with slight ifs and buts. The biggest irritant
in this effort could be different power centers in Pakistani establishment like civil government, army and
jihadi forces. But Pakistans involvement may see a common stand by all of them easing the process of
resolution

The dispute can be addressed from two sides and we need to analyze which pays off better in
order to determine whether Pakistan is in the equation or not

16) With Pakistan:

The Kashmir dispute is escalated by political instability in Pakistan. Needless to say that Pakistani
foreign affairs is managed by its army rather than the government. The dispute over Kashmir is
engraved in the hearts of its military because of the crushing defeat it has faced each time it tried
to counter India. It is very much unlikely that they forget their defeats and join hands with India.
As our PM had said, Talks are of no use when there are bombs and guns are exploding in the
border.

2) Addressing the people of Kashmir

--Kashmir dispute is mired by separatist groups functioning there who claim independence for
Kashmir

--Not all people in Kashmir are satisfied with the development that Kashmir has seen.
They are also highly affected by the misfiring that happen time to time and are not happy with the
huge military presence in the valley.
--When these issues are resolved their confidence on the Indian government would be undoubted
and the Hurriyat would face its end due to lack of support.

The recent turnover in the election is heartening and we are on the way to achieve what is
intended.
When there are no separatist tendencies within the valley then there is no question of Pakistan
coming into the equation of Kashmir. India is a Sovereign nation right from its independence and
as the Supreme Book of Indian states, so shall we follow.

Solution of Kashmir issue means, nullifying the resolutions by US, Tashkent and Indus water treaty
amendments. This all needs Pakistans cooperation. Also, for limiting the Pakistans access and phasing
it out slowly in Pakistan Occupied Kashmir has to be done with rounds of dialogue. Best strategy would
be to first involve all neighboring nations on the Indias side and project this issue as utmost important in
UNSC and UNGA. These will lead to international pressure on Pakistan, because any independent step
from India would be taken as hostility and encouragement for terrorism to feed Pakistans internal politics.

Topic: issues and challenges pertaining to the federal structure, devolution of powers and finances up to
local levels and challenges therein.

16) In the light of recommendations of the Fourteenth Finance Commission, critically


discuss implication of these recommendations for the fiscal federalism in India.

[Finance commission is a constitutional body constituted under article 280 of Indian constitution. It is a
quasi-judicial body; define the financial relation between center and state and suggest vertical and
horizontal distribution of taxes. Its recommendation is purely advisory in nature. The 14th finance
commission headed by Y.V.Reddy

Intro:-
Fourteenth Finance Commissions recommendation to increase tax devolution to 42 per cent of
the shareable pool of taxes has increased the flow of untied resources (or resources transferred without
condition) to States. (Against 32% suggested by the earlier commission)
-Estimates show that posts the 14th commission award, the untied statutory transfers would be more than
70 per cent of the aggregate resource transfers from the Union to the States.

Other recommendations:-
-To setup an independent council to undertake assessment of fiscal policy implication of budget
proposals.
-To wind up National investment fund and maintain all disinvestment receipts in the consolidated fund.
-Also to setup autonomous and independent GST compensation fund.
-It recommended the ratio of basis-to-performance grant be kept at 90:10 for panchayats and 80:20 for
municipalities.
-Taking into account the plan revenue expenditures while assessing revenue deficit grants.
-Discontinuing the distinction between special category and other states.
-Suggested institutional mechanism for better monitoring of fiscal rules and to achieve 'cooperative
federalism'.

Implications:-
14th Finance Commission gave a decisive push towards greater devolution of financial resources towards
the States.
-It simultaneously occupied the resource allocation space vacated by the Planning Commission by
transferring a substantial portion of revenues directly to the States. It thus shaped the direction in which
the central government could now move towards cooperative federalism.
-The recommendation gives high degree of autonomy to the states to execute and allocate share of funds
according to the local needs.
-These recommendation has wider implication for fiscal federalism. This move implies that grant for
centrally sponsored scheme will have to be curtailed.
-Previously many of the schemes were left redundant only because they had top down approach in policy
formulation and implementation. For ex. the universal enrolment for the school is very high in Kerala, still
funds earmarked for it were given to it.
-Empowering the 3rd tier of the democracy by providing a major chunk of the funds and making it
progressive by adding performance clause in it.
-Giving up distinction b/w plan and non-plan expenditure.
-Giving due importance to the forest cover of the states and hence understanding the opportunity cost
involved.
-Recommending the improvement of constitutional bodies like interstate council (ISC)
-It is also noted from the economic survey 2014-15 that the increased devolutions would benefit the less-
developed states more than the well-developed ones. States like Jharkhand, Arunachal Pradesh, Bihar,
UP, etc. stand to gain in terms of revenues. Hence, increased devolution of tax revenues has a very
progressive nature, i.e. poorer states benefit more than richer states.

However, instead of cutting down on its own expenditure, the Centre directed its axe towards key
centrally sponsored schemes in the social sector. Whether most of the States are sufficiently endowed
with governance capabilities to absorb and effectively spend the additional resources that have come
their way is a moot point.
-Whether they will be able to keep essential schemes running remains also to be seen.

This may not have an adverse effect on the state performances since according to the economic survey;
grants to central assisted schemes have not always been progressive.
Further, as per the Survey report, even after spending on the Central sponsored schemes after the
central support is withdrawn, the States would still have a surplus to spend on development, owing to the
increased tax devolutions.
Considering the expected increase in tax revenue with introduction of GST, the states stand to benefit
further.

Conclusion:-
The Finance commission recommendations signal the beginning of a new chapter of fiscal
federalism in India. It is to be noted that this phase lays a strong emphasis on the prudence and integrity
of all the states' political and administrative machinery in planning for their respective growth.
-Although this increase in the share of untied funds is a marked improvement in the structure of a
transfer away from a conditional to an unconditional one, it is time that attention is paid to reform the
transfer system by focusing on non-Finance Commission transfer to enable States to effectively utilize the
enhanced untied fiscal space.
-This can only be done by reforming the coverage, content and architecture of the non-Finance
Commission transfers and a further consolidation of schemes.
India 2015, Chapter 16

Topic: Issues relating to development and management of Social Sector/Services relating to Health

17) It is alleged that, India, which is home to largest number of leprosy patients in the
world has discriminatory policies against these patients. Critically examine.

Intro:-
Although the incidents of leprosy in India have been reducing continuously, the country was still seen to
have largest number of leprosy patients in 2014.
-India has the largest number of leprosy patients (nearly 58%) globally; majority of those affected are
children. Leprosy patients continue to suffer from many discriminative policies and laws.
- Various hospitals and schemes have been launched to further slowdown the spread and eventually
completely eradicate the disease from the country; still there is no environment sympathy for these
patients in the country, both in society and in the administrative structure.
Discriminatory laws and policies:-
There are several laws containing discriminatory policies against leprosy victims, like, prohibition of
leprosy patients from standing in elections. Such laws have been supported by the central government
and ratified in several judgments of SC.
-Several state legislations prohibit leprosy patients from contesting elections for local bodies.
- Other laws include the Motor Vehicle Act of 1939 which restricts leprosy patients from obtaining a driving
license and the Indian Rail Act of 1990 which prohibits leprosy patients from traveling by train.
- Many of such laws have been formulated before the Multi Drug Therapy came into being, and they have
not been updated since. For example, leprosy is still considered as a legitimate ground for divorce,
according to some of the marriage acts, which declares leprosy incurable. Such laws do not reflect the
current status of medical science against leprosy.
- As per many Personal laws such as Hindu marriage act-1955&Dissolution of Muslim Marriage act-1939,
leprosy affecting either spouse constitutes a ground for divorce, annulment of marriage or forfeiture of
maintenance.
- "Industrial Dispute Act-1947" is misused by employers to terminate employment of persons diagnosed
with leprosy
- Lepers act-1948 enforces complete segregation of leprosy patients.
- The Life Insurance Corporation Act charges higher premium rates from persons with leprosy.
- State Beggary Acts puts them under the same category as those with mental ailments

Social discrimination:-
Today with medical advances leprosy is completely curable. However due to ignorance of masses, the
disease continues to carry worst social stigma; leprosy patient is treated as an outcast from society; a
majority of children are threatened with isolation and discrimination at tender age.

Further, in 2010 UN general assembly passed a resolution to eliminate discrimination against person with
leprosy. India, despite being a member, has taken no step towards tackling this situation.
-In this regard, the Law Commission has submitted its report and also provided for a draft model to
eliminate discrimination. Some of the progressive suggestions for leprosy persons are:
- Making them eligible for legal aid
- Recognizing their land rights in leprosy colonies or alternate settlement options
- Guaranteeing the right to healthcare, housing, education, employment
- Right to travel in public transport and obtaining license to drive
- Replacement of term leper with 'persons affected by leprosy'
- Concessions during treatment for their travel, lodging and medicines
- Awareness campaigns in schools, hospitals, government institutions, private establishments

Conclusion:-
The need of the hour is to update such old acts, according to the present scenario. Also, there is a need
to make people aware of the facts of leprosy and the treatment available for it. National Leprosy
Education Program of 1985, should be given more funds, so as to diversify its activities and cure more
people, and help in making India a leprosy free country.
The Hindu
Topic: transparency & accountability and institutional and other measures; Paper 4; Right to Information

17) Should intelligence agencies provide information to the public under the Right to
Information Act? Substantiate.

Intro:-
Government of India has notified 25 security and intelligence organizations as exempt from the ordinary
obligations of transparency under the RTI Act. (Section 24)
-But are required to provide access to information if it relates to allegations of corruption and human rights
violations.
- They also need to appoint public information officers and submit reports to the Central Information
Commission about the number of RTI applications received amount of fees collected and details of cases
where access to information was rejected.
The Delhi-based Commonwealth Human Rights Initiative analyzed annual CIC reports from 2005 to 2014
and found that 11 of these security agencies, including the National Investigation Agency, the Research
and Analysis Wing, the Special Protection Group and the Enforcement Directorate, have never reported
any RTI information to the CIC.
- The Central Bureau of Investigation stopped reporting data after it was included in the list of largely
exempt organizations in 2012.
- Even among the agencies which do submit data to the CIC, the rate of rejection is very high, the data
shows. The Intelligence Bureaus rejection rate between 2008 and 2014 averaged 98 per cent.
Secrecy vs. transparency:-
The exemption of intelligence agencies from most of the requirement of RTI is done considering the
protection of whistle blowers and that the leaking of this important information to the people of our country
and then inevitably to foreign countries can be fatal.
- Intelligence agencies collect information for purposes like crime investigation, identifying threats to
society, preventing anti-social elements, vigilance in contentious areas etc.
- Intelligence agencies have a different nature of work than other regular ministries and agencies. They
are engaged in covert operations, data mining, intelligence gathering and national security.
-RTI Act on the other side is enacted to provide the civil society with information on day to day
governance, functioning and processes followed by the public service organizations for improving
transparency.
-For intelligence agencies it is secrecy not transparency that is needed. In a way both are for providing
better and peaceful life to citizens but the ways and means adopted are different.

But some of these agencies are misusing their discretionary powers, unable to hold high standards
and are eroding public faith on them. Some cases of misuse are:-
- In 2010, Exercising his Right to Information, a petitioner, had asked for information related to cases of
corruption, human rights violation and sexual exploitation in IB, RAW and Central Paramilitary Forces, but
information provided was only about Assam rifles, while other agencies denied information.
- In 2008, RAW official had allegedly attempted suicide before the prime minister's office claiming sexual
harassment by her superiors. But, RTI on this is still pending before CIC.
- CBI has refused to part with information on a purported letter of the Ministry of External Affairs asking it
not to pursue extradition proceedings against then chairman of Union Carbide Warren Anderson after the
1984 Bhopal gas tragedy.

Way forward:-
It becomes pertinent to re-balance the secrecy requirements of such agencies with transparency
requirements of civil society as intelligence agencies are for civil society and not the other way round.
-The agencies wrongfully cited section 24 of the RTI Act for denial of response to the petitioner. Proper
information related to violation of human rights or corruption should be address by agency and they
should submit required reports to CIC.
- This calls for an enhanced stress on time bound disclosure of information under RTI in cases of abuse
of powers, human rights including sexual abuse. GoI should come up with a suitable legislation which
ensures that such agencies give information in cases pertaining to crimes or charges mentioned above.
- It is also essential for Government to bring the law which governs the working of Security agencies such
as RAW, IB, other intelligence entities by act of Parliament same as USA and Britain.
- Thus, though mandate of Security and Intelligence agencies are different from
other public bodies, they cant act as an alien to governance and public scrutiny.
- All such agencies must appoint PIO and submit annual report to CIC even if all received queries are
discarded for the sake of national security.
- An audit can be done for enquiring the grounds of rejection for seeing validity in such rejection.
- Appellate authority even in instances of first rejection must be CIC directly.

Conclusion:-
This will ensure appropriate checks and balances and reconcile national security with transparency and
will go a long way in building a healthy information regime. Intelligence agencies, most of them, are not
governed by legislation and do not have their expenditures mentioned in the budget, so their freedom
from laws like RTI must not be absolute.
The Hindu

Topic: Development processes and the development industry- the role of NGOs

18) Critically analyze the role and contribution of civil society to developmental activities in the
country. Do you think some of the criticisms directed towards them regarding source of their
funds and their opposition to certain projects, is fair? Comment. (200 Words)

Introduction:

Civil societies are a combination of NGOs, labor organizations and other institutions that propagate the
interests of general public. They act as a link between citizens and the government and businesses. Their
play the role of putting forth the public welfare agendas to the govt. and direct their efforts to fill in the
gaps left by the government.

With the huge size of the country and the disparities of inequality , rich poor ever deepening divide and
many other social and economic gaps the roles played by over 20 lakh NGOs is certainly commendable.
NGOs too have a wide range of organizations ranging from the political interests based, social work
based, tribal welfare based, northeast rights based, Human rights based and many more and they are
both locally funded and foreign funded and even non-local in their origin

Contribution of civil societies in the development in India- instances:

1. Making the politicians accountable by voicing their opinions on the policies and opinions of the govt.
e.g. the Anna Hazare campaign against corruption.
2. Filling the gaps left by govt. in development either due to awareness issues or funds crisis e.g.
bachpan bachao andolan

3. Working in areas in which the govt. does not want to enter say fighting caste discrimination.

4. working in areas where govt. efforts prove inadequate like health and education e.g... Kerala Sahitya
Shashtra Parishad.

5. By protesting against environmentally bad decisions of govt. such as Posco and Vedanta protests,
Narmada bachao andolan

6. Spreading awareness campaigns against environmental and social issues such as protesting against
deforestation, spreading the benefits of sustainable agriculture, protesting against female
feticide and promoting girl education.

7. Fighting for necessary laws such as RTI

8. Fighting for the cause of those who cannot fight for themselves by filling PILs.

Background of the criticisms:

Allegations have been labeled against them that particularly NGOs that are foreign funded are using their
funds to mobilize support and create hue and cry against the development projects proposed and that
has caused a loss of about 3% to the GDP. So, on this ground their foreign funding sources have been
blocked (of about 4000 of them), the Green Peace activist was detained at IGI airport and not allowed to
travel to London was in the news recently, on the charges that it will hurt Indias image diplomatically.

But the question that comes to the mind here that is opposed to safeguard the tribal rights is anti-growth,
fighting for the trees and rivers in Chipko and Narmada Bachao respectively is anti-national, is fighting for
the grave human rights violation is sedition

Criticisms:

1. Recently decades have seen a spurt in civil society organizations across the country; Many of them do
not server the larger interests of society such as lobbying for industrial groups or foreign nations
by advocating a policy.

2. Most dont file annual returns on a regular basis; their funding sources are often remain disclosed or
shady.

3. Many of them receive substantial foreign funding; but how they spend such funding often remains
behind curtains.

4. Some CSOs such as Greenpeace have accused by state agencies such as IB for instigating villages in
vicinity against developmental projects such as POSOCO steel in Orissa; this ate up 2-3% of our GDP.

Such opaqueness in CSOs deserves criticism; the current situation call for CSOs to take the lead and
remain/become open & transparent towards the state & society at large.
Topic: Issues relating to development and management of Social Sector/Services relating to Health,

19) In a recent index called Social Progress Index (SPI), devised by an NGO named Social
Progress Imperative, India ranks lower than its neighbors Nepal and Bangladesh. Write a critical
note on this index and Indias performance in the same. (200 Words)

Social Progress Index (SPI):

Social progress index is an Index run by a USA based organization called the Social progress initiative.

Social Progress Index ranks countries on indicators of well-being such as health, water and sanitation,
personal safety, access to opportunity, tolerance, inclusion, personal freedom and choice.

What makes the SPI unique is that unlike the HDI and the Gross Happiness Index, it doesnt take into
account any economic parameters and therefore advocates the idea that:
Measuring progress in monetary terms fails to consider the wider picture of the real things which matter
to real people;
there is no link whatsoever between economic and social progress.

While it is true that GDP is not a holistic measure of a nations development, it would be incorrect to state
that the economic progress is completely divorced from progress made in areas mentioned above. It is a
rather simplistic approach because economic growth of a country does makes lives better. Therefore, a
correlation exists between these two, however imperfect.

SPI measures the extent to which countries provide for the social and environmental needs of their
citizens. Fifty-two indicators, in the areas of basic human needs, foundations of well-being, and
opportunity, show relative performance in order to elevate the quality of discussion on national priorities
and to guide social investment decisions.

The primary goal of the Social Progress Index is to provide a rigorous tool to benchmark progress and
stimulate progress within countries. It mainly measures the indicators of social outcomes, rather than
measuring inputs, so as to access the social progress in the country.

Several indicators, like GHI and HDI, go beyond GDP, but none captures social progress as finely as SPI.

But the index still contains 133 countries, and leaves out many other countries, so this report can be
considered as a pre mature report, as several ranks may change by including other countries.

Indias Position:

India has got a lowly ranking of 101, which is even lower than our neighbors Bangladesh (100) and Nepal
(98) respectively. India is also the lowest rank holder in BRICS.

This is because India has scored really low in categories like tolerance and inclusion and also in health
and wellness. These indicators go on to show the persisting narrow-mindedness of our society and the
problems we face being a religiously and culturally complex and diverse country. It also shows us that
even though we have grown economically and increased spending on health-care, due to a growing
population and problems within the delivery system, our health care measures are not as effective as they
ought to be.
To improve the rank, the present social beneficiary schemes need to be implemented with better audit
mechanism, so as to ensure zero fund leakage and no bogus beneficiary.

--Skill imparting programs should be diversified to the villages too, so that people can live with a better
standard of living.

--Universal Health coverage, better medical facilities.


Such measures will help India in achieving a better position in the Index, while simultaneously
improving on GDP too, due to more skilled population.

Topic: Bilateral agreements involving India and/or affecting Indias interests

5) It is said that India shares special relationship with France unlike other Western countries. Critically
analyze the nature of bilateral relationship between two countries, especially in the light of recent
developments related to high technology and defense sector. (200 Words)

The Hindu

Topic: Salient features of the Representation of Peoples Act.

20) Critically examine which factors have necessitated reforms of electoral laws in India;
especially the Representation of the People Act and what reforms have been implemented so far.

INTRODUCTION

Being the largest democracy of the world, India has the reputation of handling the worlds biggest
democratic exercise at the time of elections in a fairly peaceful manner.

FACTORS RESPONSIBLE FOR ELECTORAL REFORMS

Factors which necessitated electoral reforms in India include the large number of criminal
antecedents of members of Parliaments and legislatures, the immense amount of wealth
acquired by some politicians during term of office, the various scams like cash for vote scam etc.

Under the active Election commissioner T N Seshan electoral ID cards were issued to citizens to
minimise fraud voting and clean up the voter list. Along with this Seshan implemtented the model
code of conduct and remove the various malpractices by not bowing down to politicians
pressure.
Criminalization of politics has increased lately, due to inclusion of mafia in financing some
political parties, along with booth capturing and black money generation.
Misuse of public funds for publicity, violence, fraudulent voting etc.
REFORMS WHICH HAVE BEEN IMPLEMENTED

1. Inclusion of NOTA button, to allow people to express dissent against all parties in the election.
2. Result of exit polls to be announced half an hour after polls close, to avoid biasing the public
opinion.
3. Prohibition on seeking assistance of any govt employee to further ones candidacy.
4. The election commission of India, with time shifted from paper ballot to EVMs making the vote
counting procedure more reliable. They transited to EPICs to reduce cases of fake voters.
5. They have initiated online registering for voter ID cards to enable expansion and in keeping with
the changing times. They have also registered NRI voters to increase their political engagement
with the country

But still, there remains room for further reforms, in various sectors, due to the following problems:

1. The expenditure of some parties in election is in tune of about 20-30 times the legal limits, which
renders some independent candidates as ineffective in elections.
2. Electoral rolls are prepared by both EC and State ECs, which leads to duplication of duty and
avoidable time and money wastage. Inclusion of a large number of non serious independent
candidates in the election fray, which puts unnecessary pressure on the EC.
6. Some candidates resort to admitting false information regarding their assets and other properties,
which is overlooked if the candidate is from a strong party.

CONCLUSION

Although changes have been made the real changes would occur when political funding of the parties is
made more transparent. Implementing some provisions of the RTI act on political parties would not be a
bad idea,

Topic: Government policies and interventions for development in various sectors and issues arising out of
their design and implementation.

21) Do you think there was a need for another identification number for Indian citizens when the
Election Commission of India was already involved in providing unique identification number
called as Electors Photo Identity Card (EPIC) number? Substantiate.

Introduction

What is EPIC Card? EPIC card is none other than your own Voter ID card. Yes, Voter ID card
and EPIC card is one and the same thing. EPIC is an acronym used for Electors Photo Identity
Card.

The unique identity number provided by the election commission in EPIC is limited to
electors i.e people of and above 18 years of age who have registered themselves to
become voters.
In contrast, the unique identification number espoused by Aadhar has no such limitation and
when introduced, it was mandatory for all citizens to register for it.

Why a need for another identification number

The Aadhar card has a universal appeal as it captures finger prints as well as retina scan of the
citizens in its database and is thus more wholesome than an EPIC card number.
So the intention of providing a UID outside of EPIC card is not redundant as the former covers
more people and provides a database that can be used by the various governmental agencies.
The Aadhar details are sought to be linked to various central and state service schemes including
opening of bank accounts and pushing forth mobile banking. Because finger prints are used they
provide an effective way for identification and it makes it more difficult to dupe or create fake
beneficiaries.
Mission aimed to cleanse the electoral rolls by weeding out bogus and duplicate voters.

Conclusion

So I would say that there was definitely a need to create another identity number as the right to vote or
even registering to vote is only a duty. It cannot be enforced. Whereas the aadhar card upon passing of
requisite legislation can be made mandatory for the citizens.

Topic: Welfare schemes for vulnerable sections of the population by the Centre and States

22) Recently, while ruling on inclusion or exclusion of certain castes or communities for availing
reservation benefits, the Supreme Court argued that caste cannot be the only standard to
determine backwardness, but can be an important factor. What is your opinion on this issue? Do
you think reservation is used more as a political tool these days than as a tool of social justice?
Critically comment.

INTRODUCTION

The framers of our Constitution were conscious that rigid hierarchies and structured inequalities
had dehumanized many sections of the population, excluding them from the mainstream.
They wanted to salvage the nation from this social exclusion at the earliest and make them full
citizens of the nation. Thus, reservation is a fundamentally political promise made in
acknowledgment of the fact that caste literally excludes sizeable communities from Indian society.
Even after 65 years of working the constitution, caste is still the primary basis for social exclusion.
There are other secondary factors too, like gender based, disability based etc. The correct way to
tackle these is to provide quota within the larger SC/ST/OBS quota, like the PH quota and women
reservation in local bodies.

RESERVATION ON BASIS OF CASTE


Over the years backwardness has not only been confined to social status but also economic,
religious and sexual status as seen in the findings of Sachar Committee Report and the recent
SC judgment on the inclusion of transgender in the Backward Class category.
Further, reservation has also led to an intra-class divide whereby only the economically and
politically well off in a particularly are able to enjoy the benefits of reservation leaving the true
beneficiaries at a loss.

RESERVATION AS POLITICAL TOOL

Unfortunately, reservation is increasingly been used as a political instrument to ensure electoral


support. Inclusion and exclusions from the list are done on the basis of political ideologies and
vote calculations than for social justice
This was seen in the decision to include Jats in OBC list against the recommendation of NCBC
and the decision to exclude Muslims in Maharashtra despite several reports, mainly those of the
Sachar Committee, the Ranganath Misra Commission and the Mehmood-ur-Rehman Study
Group
Moreover, experience has shown that policies which target broad and heterogeneous social
groups cannot provide social justice, as the most disadvantaged sections tend to be excluded in
favour of the more privileged, Yet, politicians continue to take advantage of the ambiguous
meaning of 'backwardness' as implied by the Mandal report to achieve their own political ends.

CONCLUSION

However, this tool can be used to a certain extent, due to greater democratization of our polity
and first past the post system that requires maximum support of all communities.
Thus, there is greater rhetoric of secular and development oriented politics than those based on
caste.
We therefore need to evolve our system of protective discrimination which takes into cognizance
social, economic, religious, physical and gender-centric disabilities.

Topic: Bilateral agreements involving India and/or affecting Indias interests

23) Critically examine the implications of recent nuclear deal between Iran and the E3+3 (France,
Germany and the United Kingdom + China, the Russian Federation and the United States) for
India.

INTRODUCTION

The interim nuclear deal agreed between Iran and the six world powers, P5+1, on November 24, 2013, to
curb Irans nuclear programme, has been implemented recently. This deal envisages easing of some
international sanctions on Iran in return for the partial roll back of Iranian nuclear programme.

IMPLICATION ON INDIA
POSITIVE

1. The revival of oil supplies from Iran could provide much support for energy starving country
coupled with low oil prices and better relations with Iran. Iran has the 4th largest oil reserves and
2nd largest natural gas reserves and India is a major importer or oil.
2. Irans Bandar Abbas port, conceived as the hub for the International North-South Transport
Corridor (INSTC), remains the shortest and most economical route from India to Central Asia,
Russia and Europe.
3. Connectivity - Chahbahar port and International North South Transport Corridor, would
receive a boost. This would help in greater connectivity to Central Asia and Afghanistan, in turn
boosting tourism, trade and competitiveness of Indian exports. Cultural ties of Persia can be a
better bet for diplomacy and other relations in important groups like UNGA, UNSC support.

4. Iran stability is important to counter effects of Islamic State, Saudi dominance, Syria disturbances
which could otherwise affect Indian Muslims threatening Indian stability.

5. Safer destination for Indian firms to invest as well as to export their produce- generating revenue,
employment creation.
6. Other than this, India also has to clarify its intentions to the other middle east nations, regarding
relations with Iran, which is going to need the best of diplomatic capabilities, with which a cordial
relation can be maintained with the West Asian countries

NEGATIVE

1. The easing of sanctions against Iran could cause Tehran to favour trade with countries willing to
pay wholly in US dollars. As international demand for Iranian oil rises, there will be less incentive
for Iran to continue the oil-for-rupee agreement with India. It is also likely that Iran will reduce its
exports to India.
2. While the removal of sanctions will ensure a better oil supply deal between the two countries, it
will also mean that now India will have to compete with other countries entering to trade with Iran.

WAY AHEAD

inadequate infrastructure restrict India from approaching Iran, though investing but insufficient &
India itself in dire needs of infrastructure(both physical & social)
India needs to show its diplomatic manoeuvre while balancing the relation between Iran &
countries of Arabian Peninsula (which resides 7m Indians) & Israel.

However, the growing inclusion of Iran shall be looked at by all the nations particularly China to
involve Iran in its Silk Road project and maximize its interest in Central Asia.
Therefore, India will have to exhibit great agility in capitalizing on a new Iranian economy through
defence collaborations, investments and trade pacts. Also, India will have to show a great deal of
diplomatic dexterity in managing relations with Iran and the gulf and Israel.

Topic: Issues relating to development and management of Social Sector/Services relating to Health,
Education, Human Resources.
24) Recently, the United Nations Educational, Scientific and Cultural Organisation (UNESCO)
published the Education for All Global Monitoring Report to evaluate the progress of countries
on these goals. Critically comment on its findings on Indias performance. (200 Words)
The Hindu

Introduction:

Education For All (EFA) is a global movement led by UNESCO


It aims to meet the learning needs of all children, youth and adults by 2015.
The EFA goals also contribute to the global pursuit of the eight Millennium Development Goals
(MDGs).
The Fast Track Initiative was set up to implement the EFA movement, aiming at "accelerating
progress towards quality universal primary education".
UNESCO also produces the annual Education for All Global Monitoring Report.
In 2000, the participants agreed on the Dakar Framework for Action which re-affirmed their
commitment to achieving Education for All by the year 2015, and identified six key measurable
education goals which aim to meet the learning needs of all children, youth and adults by 2015.
Goal 1: Expand early childhood care and education
Goal 2: Provide free and compulsory primary education for all
Goal 3: Promote learning and life skills for young people and adults
Goal 4: Increase adult literacy by 50 percent
Goal 5: Achieve gender parity by 2005, gender equality by 2015
Goal 6: Improve the quality of education

Indias performance:

Enrolment targets - India has achieved 100% primary enrolment and 100% enrolment in schools.

Gender Parity Achieved in primary and lower secondary enrolments.

Adult illiteracy Poorly performed. Target was 50% reduction, only 26% achieved. Most other
nations too have fared badly, with only 25% managing to achieve this target.

Education expenditure Poor performance. Govt expenditure has to be increased.

o The report also disapproves of more and more teachers being hired on contract basis.

o Some state governments like Rajasthan curtailing the number of schools. This will hurt in
future.

Quality of education - The UNESCO report draws on both ASER (Annual State of Education
Report) by an NGO and NAS (National Achievement Survey) by NCERT to point out that
students across the country are not performing according to their grade level. This point to
serious lacunae in the education system.

Critical comments:
Need to improve quality of education by improving teaching style. Focus of SSA must be shifted
to include quality too.
Support to weak students to grasp better.
Improve teacher training.
Try to emulate primary level performance of enrolment in secondary level.
higher investments in infrastructure and construction of toilets
using digital technology for quick grasp of things for improving quality in education
better monitoring and evaluation of student methods

Topic: Issues relating to development and management of Social Sector/Services relating to Health,

25) It is hoped that the Law Commission of Indias 256th Report, Eliminating Discrimination
against Persons Affected by Leprosy, will initiate a new chapter in Indias fight against leprosy.
Examine. (200 Words)
The Hindu

Facts:
1. Leprosy is also known as Hansens disease.
2. The disease is curable with multidrug therapy (MDT). The treatment was developed in 1983. MDT
was incorporated in Indias health programme. The incidence of leprosy reduced but rate of
decline slowed since 2005.
3. India still account for 58% of all cases globally.
4. The Leprosy Act of 1898-institutionalized leprosy victims and separated them based on gender to
prevent reproduction.
5. The Motor Vehicle Act of 1939 which restricts leprosy patients from obtaining a driving license
6. The Indian Rail Act of 1990 which prohibits leprosy patients from travelling by train.
7. The Special Marriage Act of 1954 declaring leprosy incurable. leads to divorce
8. The Life Insurance Corporation Act charges higher premium rates from persons with leprosy.
9. Several State Municipal and Panchayat Raj Acts bar persons with leprosy from holding or
contesting civic posts.
10. Persons affected by leprosy are usually made to relocate to Leprosy Colonies in India, but they
do not have land rights, and are constantly under fear of eviction.
11. Recently Most of these laws discriminate against leprosy patients in spite of the fact that it is
curable now. Patients are still called lepers.
12. These are the reasons for which Law Commission submitted its report Eliminating Discrimination
against Persons Affected by Leprosy.

Need of the reforms:

1. To give better life to people affected.


2. Since the disease has become curable, it should be recognized so in statute books and
required amendments made.
3. Recommendations draw from the constitutional values of dignity, equality and freedoms.
4. The Law Commission recognizes Indias international obligations arising out of the UN
General Assembly Resolution on the Elimination of Discrimination against Persons affected
by Leprosy, 2010, and its obligations under the UN Convention on the Rights of Persons with
Disabilities, 2007.

Recommendations of Law Commission that go against fighting social stigma:


1. Repeal provisions of laws including personal laws which discriminate on grounds of leprosy.
2. Stop the official use of the word leper.
3. Amendment of the Persons with Disabilities Act to broaden the definition of disability to
include all categories of person affected by leprosy.
4. Enactment of a legislation that promotes the inclusion of persons affected by leprosy and
their family members through affirmative action. Such legislation should provide for measures
against discrimination, land rights over leprosy colonies, right to employment and educational
opportunities.

However, one particular recommendation can raise controversy: the suggestion that persons
living in leprosy colonies need to be granted title of the property in their possession. But this will
only perpetuate isolation as these colonies are the sites of discrimination and leave its residents
segregated from the rest of the society.

Conclusion: The law commission report can actually go a long way to improve conditions
of leprosy patients in India. Broadly, these recommendations are based on constitutional
values of equality and dignity, are a step in the right direction.

Impact of recommendations:

1. Implementation can help to ensure better and respectful life for affected people.
2. Thus, it can actually go a long way to improve their life.
3. However, its result depends on the political will to implement and educate people to
stop discrimination.

Topic: India and its neighbourhood- relations.

26) How has recent election outcome in Israel altered the possibility of granting independent
statehood to Palestine? Also examine if there is any change in Indias policy towards Palestine in
recent months. (200 Words)

EPW

The 2 state solutions envisages the formation of two nations living in peaceful coexistence i.e.
1) State of Palestine comprising of West Bank and Gaza
2) State of Israel comprised of Non Palestinian Territories

Although Israel removed Jewish settlers in Gaza to make it a Palestinian Exclusive territory, the
reverse is true for West Bank.

Jewish settlements, deemed illegal by United Nations, have propped up with the support of Israeli
State.

Recent election in Israel:

Benjamin Netanyahu was re-elected.


He has a right wing political orientation and this will impede the possibility of granting independent
statehood to Palestine as

Netanyahu rejects the 2 state solution, which he sees as a threat to national security, and
encourages settlement in West Bank.

So, with his re-elections, independence of Palestine is becoming difficult.

Importance of Palestine for India:

Large Muslim population in India.

Import of oil from Gulf which supports Palestine.

Gulf is the largest source of remittances to India.

Importance of Israel:

Largest supplier of arms

Import of high end technology

Dry land agriculture technique

Change in Indias stand towards Palestine:

Closeness of ties between India and Israel.

For the first time, India is openly accepting close relations with Israel.

Modi to be first Indian PM to visit Israel.

India is giving preference to Israel in arm deals.

Relation with Palestine has not degraded.

Conclusion:

Closeness and openness with Israel speaks volume about changing ties.

India should not hurt Palestine as it is important to maintain internal peace.

Diplomatic approach should be adopted to balance relations with both.

Topic: India and its neighbourhood- relations.


27) Some analysts argue that the imminent crisis in IndiaChina relations will come to the fore
when the question of the Dalai Lamas succession in Tibet can no longer be ignored. In your
opinion, how should India try to resolve its differences with China? Critically comment in the light
of the statement. (200 Words)
EPW
History of Tibet issue:

Qing dynasty rule ended over China in 1912.

National Govt of China was formed. It signed an agreement with Qing dynasty taking over all of
its territories. According to them, this included Tibet too.

Meanwhile, Tibet signed an agreement with Mongolia identifying itself as separate state.

In 1950, National govt of China lost civil war to Peoples Liberation Army. It entered Tibet and
overtook it.

Tibet Autonomous Region is a province level autonomous region. It was created in 1965 on the basis
of 1951 agreement.

Present status:

1. A govt in exile is functioning from Dharmshala.

2. Demand: Independence in no longer demanded, but genuine autonomy for Tibet region.

3. India had given refuge to the govt in exile and it has been functioning from Dharmasala.

4. China opposes this support of India to Tibet govt.

Issue:

Issue of Indias support to exiled govt is generally avoided by India.

Boundary disputes are not settled between India and China.

China claims Arunachal Pradesh as its own.

A part of J&K (Aksai chin) was occupied by china in 1962 war and since then, it is under
Chinese possession.

Why in news?

Over the question of succession of Dalai Lama.

Who will be his successor, and should present one choose his successor now.

The reaction of Tibetan Diaspora on choice of successor.

Chinese reaction to it.

Opportunities for India

1. India may use Tibet as leverage to extract favourable solution to boundary disputes.

2. It is possible that Indias support to Tibet may worsen relation between two countries,
making solution to disputes impossible.
India should do following to resolve issue with China keeping Tibet in mind:

1. India has accepted Tibet as integral part of China, so there is no way around.
2. Tibetan nightmare can disrupt border issue with China. Solve as many border and other issues as
possible before the problem of succession crops up.
3. India must strongly oppose any inhuman activities in Tibet.
4. Any matter on Dalai Lama's reincarnation must be ignored, because it is matter of faith and not
politics.
5. India must allow exile government of Tibet trying to control political situation in TAR.
6. Make the Chinese government see the benefits of dealing with the Dalai Lama and allowing him
to choose his successor
7. Passing resolutions through various international forums like the UNGA seeking peaceful
resolution of problems in Tibet.
8. Lastly, advising the Dalai Lama about the benefits of choosing a successor and the need for
Tibetans to have a leader to look up to after his passing.

Conclusion: Tibet has remained biggest stone in balancing ties with China; India must act consciously
considering western opinions also on Tibetan problem, but without enraging Chinese aspiration and
expectation towards India.

Topic: Parliament and State Legislatures structure, functioning, conduct of business, powers &
privileges and issues arising out of these.

28) Recently the re-promulgation of the Right to Fair Compensation and Transparency in Land
Acquisition, Rehabilitation and Resettlement (Amendment) Ordinance, 2015, created
controversy regarding misuse of ordinance power by the government. Critically examine
the issue and comment if, in your opinion, such re promulgations goes against
democratic principles.

Introduction:

Articles 123 and 213 of the Constitution empower the President and Governor with the power to issue
ordinances which are essentially enacted to effectively deal with any emergencies that may arise
when the legislature is not in session. This power, which is unique to the Indian Constitution, & is
envisaged to be of a special nature to be used sparingly, has become a norm of the day for
successive governments and repromulgation is part of it.

Issue
However, of late, this power has been grossly abused by the executive to enforce its will by force. An
example is the re-promulgation of the Land Ordinance (Amendment) Bill. Government having failed to
convince the opposition in the Upper House of the need to amend the Land Acquisition Act 2013 took
to the re-promulgating route, thus effectively bypassing the legislature.
Such re-promulgation is against the democratic ethos in the following way:
1. The repeated re-promulgation of ordinances was held to be a "fraud on the constitution" by the SC in
D C Wadhwa v. State of Bihar. But SC had prescribed 2 conditions under which ordinance could be
repromulgated
When there is too much business pending in the legislature
Lack of time with the Parliament due to the short session
2. This is a violation of the doctrine of separation of powers as Legislating power lies in the domain of
the Parliament, which is part of the Constitution's basic structure. It is for this reason that the
constitution envisages this power to be used sparingly.
3. spirit of Article.123 doesn't envisage for the degeneration of the ordinance making power into a
source of adhoc legislating power for the executive
4. The Land Ordinance has been re-promulgated on account of the Government's inability of having the
bill passed in the Parliament. This does not amount to an emergency situation envisaged under A.
123.
5. The re-promulgation is undemocratic as the ordinance making process takes place behind closed
doors that
forcefully imposes the will of the executive upon the citizens
prevents public scrutiny of such exercise,
bypasses the collective will of the legislature
When a law is debated in the parliament, it brings forth many different perspectives and issues
related to the law from a range of political parties which helps overcome the drawbacks of the law
6. Moreover representation of legislature is more diverse as compared to Executive.

Why government is resorting to this measure so frequently?


Stiff opposition in the Parliament by the opposition parties preventing the government from passing
the law that it desires.
Lack of majority in Rajya Sabha which will act as a hindrance in passing of the bills
Since the Land acquisition ordinance was set to lapse, the Government re-promulgated it to provide
continuity in its policies.
Increasing number of national parties has made the consensus building difficult.

Steps required to resolve the issue


o Government needs to respect the separation of power principle. It must build up consensus through
dialogue instead of bypassing the parliament. Parliamentary debates and discussions cannot be
subverted.
o Parliament sessions must be extended if legislative business is not able to complete.
o Ordinance making power shouldn't be used frequently and with malafide intention. It is not parallel
law making power against parliament
o Supreme Court has also held that ordinance promulgation is subjected to judicial review on grounds
of malafide intentions or want of exigencies.

Conclusion
While the power has been abused in the recent past, calling for its abolition is not an effective solution
as the power has on many instances come to the aid of the executive in dealing with emergencies.
Therefore, the problem is not with the existence of power, but with the way it is exercised.
Solution lies in the executive exercising restraint and ordinance making should not become a usual
practice creating an Ordinance Raj. Though the land reforms are need of the hour for economic
growth, but building a consensus among the public and giving it a statutory accord would grant
legitimacy to the act.

The Hindu

Topic: Judiciary, its structure and functions

29) Recently the Union government notified the National Judicial Appointments Commission
(NJAC) law. Critically analyse objectives and functions of NJAC and their effect on
structure of Indian judiciary.

Introduction:

With establishment of the NJAC through enactment of 99th constitutional (amendment) act and
National Judicial Appointment Commission act, the 2 decade old collegium system for appointment of
judges to higher judiciary has ended.

Objectives & functions:


Objective: NJAC act seeks to do away with opaqueness in appointment and transfer of judges of
higher judiciary; replacing it by a broad based participatory process & ensuring transparency in
judicial appointments.
NJAC will be six member panel headed by CJI & will also include two senior most judges of SC,
union minister of law and two eminent persons/jurists, nominated by a committee comprising of prime
minister, CJI and leader of opposition or leader of largest opposition party in Lok sabha.
NJAC will perform the following functions:
appointment of CJI
Appointment of SC judges
appointment of Chief justices and judges of HCs
transfer of chief justices and judges of HCs
Ensure persons recommended are of merit, ability and fulfil other criteria mentioned in regulations
related to NJAC act

Criticism
No time period has been fixed for NJAC to take decision (complete its selection and make
recommendations to the President)
No obligation on Commission to make its selection process public or declare the rationale behind its
decision etc
Transparency in selection is not mandatory but only subjective.
Empowers NJAC to regulate its own procedure
Legal fraternity feels that NJAC is a ploy to bring judiciary within the ambit of executive; the act will
limit judiciary in scrutinizing executive's malafide actions; executive getting a bigger hand in
appointment.
This is against the separation of powers principle of the Indian constitution and interference by the
executives. Their power of selection could be easily vetoed by the executives
Compromises independence of judiciary; panel's non-judicial members can veto a candidate
irrespective of views of three judicial members. Implication of veto rights are
o delay in appointment in judges which already face scarcity
o if vetoed by any 2 executive, judges will have no say so their view is no more sacrosanct
o Judges can be influenced to get posting easily.
Give rise to sycophancy & favouritism as judicial members in panel might say ok to recommendations
of political executives in hope for future favours with judges being appointed as governors.
administrative burden of appointing and transferring judges without a separate secretariat or
intelligence-gathering mechanism for collection and checking of personal and professional
backgrounds of prospective appointees
NJC's recommendations for the appointment and transfer of Judges will be binding on the President
but it is silent on whether the NJC's advice to the CJI or the Chief Justice of a High Court, after an
inquiry into a charge of misconduct or such deviant behaviour by a Judge, will be equally binding on
those authorities
NCRWC wanted to exclude political appointees from examining complaints of deviant behaviour
against Judges and recommended that only a committee comprising the CJI and two senior-most
Judges of the Supreme Court be empowered to do that. The NCRWC suggested the creation of the
NJC only to choose Judges of the Supreme Court, not of the High Courts

Positive effects

a) Stake Holders Inclusion - NJAC is to be assisted by Permanent Search Committee similar to line in
U.S and UK. Committee shall seek consultation from BAR and other stakeholders which was missing
in collegiums system.
b) NJAC is to end the menace of nepotism, patronage, corruption and favouritism prevalent in
collegiums system which has led to the degrading of image of judiciary, as several judges were
involved in corruption cases. This created mistrust among the people and other judicial members
about the credentials of selected person, besides making the whole process opaque. This would
usher transparency in selection process.
c) By including the executives and the 'eminent persons' in the selection panel it will curb the
arbitrariness in the selection process. This allows a balance between the judiciary and the executive
to appoint judges after more discussion on the proposed names and help in purifying the judiciary of
the country
d) The proposed new system is the next step to the first, second and third Judges cases of the 80s era.
It is an initiative to include the say of the executive branch in the appointment of judges in the courts,
to avoid appointing of brother judges, by the judiciary.
e) Focus shouldn't only be on who selects the candidates, but the criteria for their selection should be
the priority, like, what weightage to be given to merit & seniority criteria.

Conclusion

Although NJAC will prevent "judges from appointing brother judges"; but promotion of sycophancy is
implicit in the act; the act should specify certain cooling off period before SC judges can be appointed
to posts like governors.
NJAC attempts some long pending reforms in the judiciary & is intended to bring to required
transparency and a sense of accountability in the selection and transfer of judges. There should be
cooperation between both branches to induct worthy candidates in the higher judiciary & to improve
judicial structure in the nation.
The Hindu
Topic: Welfare schemes for vulnerable sections of the population by the Centre and States and the
performance of these schemes;

30) Contrary to popular perception the decline in MGNREGAs performance was not due to the
lack of demand but was a result of a carefully crafted strategy to stifle the programme by
burdening it with administrative requirements and delays in fund flow. In the light of the
statement, critically evaluate the performance of MGNREGA. (200 Words)

Livemint

Intro: The MGNREGA, which started with much drumbeat in 2006, has now declined in popularity, due
to the various administrative and systemic bottlenecks, which have crept up over the years.

Criticisms of MNREGA (against the Scheme)

1. No real skill imparting.

2. Low quality of assets made.

3. Irregular and unequal duration of work in different states.

4. Not created any stable physical infrastructure.


5. Leakages and Ghost beneficiaries. However, with the trinity of Jan Dhan, Aadhar and Mobile,
leakages will be reduced substantially in times to come and ghost beneficiaries eliminated.

6. Heavy fiscal burden on the state.

Problems with MNREGA (Loopholes):

1. Private workers wages are now more then what are given in the schemes, pushing them away
from the scheme.

2. High level corruption at ground level, where workers are not given full payment or promised
regular days of work.

3. Lack of funds: Budget allocation to the scheme going down every year.

4. Although the villagers want to work in the scheme, they are driven away due to the apathetic
treatment by the administration.

5. Less creation of employment days, because one survey found that nearly 35 per cent avg
working days made available in past 10 years.

6. Delay in payment for months and allocation of lesser funds all demotivated the rural labour.

7. Suddenly joining it with DBT or through post offices without creating required awareness make
illiterate people less excited toward this.

8. Stagnation of minimum wages paid which is less than market wages.

Parameters of success of MNREGA

1. Execution
2. Grass root level reach
3. Efficiency of implementation
4. Community assets built, and
5. The number of people benefited

Earlier Appraisals Reasons (Benefits from the Scheme):

1. Regular supply of work to the jobless labourers


2. Purchasing power to rural workers
3. Social Audit
4. Increasing consumption
5. Rescue from droughts
6. Stop distress rural-urban migration
7. In Global Hunger Index, India's status improved and MNREGA is cited as an essential
component for it.
8. Empowerment of disadvantaged sections like SCs, STs and Women.
9. Development of agro-allied sector infrastructure
10. Promoting afforestation and land reclamation

New Changes Done:

1. Linked to the Green India Initiative to augment the afforestation drive seen as a good move to
counter deforestation and maintain the green cover.
2. Linked with Jan Dhan Yojana and Direct Benefits Transfer Scheme.
3. Recently MNREGA's assets have been deviated towards green drive and in public construction
activities (like railways, roadways etc.).
4. Scheme shifting from demand based to a supply based program.
5. Created sustainable rural assets like check dams, ponds, earth works etc. thus improving rural
infrastructure.

Fresh changes required for better Implementation:

1. Impart skill to the labourers regarding scientific farming practices.


2. Shifting targets towards productive areas.
3. Streamlining of MNREGA with other schemes like Indira Awas Yojana, Nirmal Bharat Abhiyan,
Bharat Nirman etc. Such step will not only help in creating more sustainable and required assets
in rural areas but also eliminate any duplication of efforts.

Conclusion:

1. The World Bank recently said that MNREGA scheme is the only and the best bet India has
for mitigating the impact on the poor of the recent unseasonal rains. Further, it has asked
the government to take steps aimed at arresting delays in wage payments and the rising unmet
demand for jobs.

2. Weighing the arguments against and in favour of the scheme, it can be said that the scheme is
financially weak but socially strong. Hence, effort should be taken to improve the targeting of
the scheme, linking with sustainable activities and curbing corruption.

Topic: Government policies and interventions for development in various sectors and issues arising out of
their design and implementation.

31) In the West, especially in America, a heated debate is going regarding the merits and demerits of
using body-mounted cameras (Cop-Cams) on police officers. Critically examine why this ideas has been
mooted and what are the main concerns with this strategy. Do you think such a policy can be used in
India as a tool to bring reforms in policing? Comment. (200 Words)

The New York Times

Intro: Cop cams or Camera attached to body of Police is the new topic of engaging debate. Such an
Idea was mooted after some recent cases happened in U.S. in which arbitrary actions by police officers
were taken and some time it even took the racial colour.

Merits of Cop Cams:


1. Transparency, Evidence based Talking. Everything will be Videotaped.
2. In time of a suspect civilian running away, its image can be transmitted online at much ease.
3. Abuse of power can be checked.
4. Computer chip and not human will be memorising the Crime Scene.
5. Less likeliness of police authority to take unwarranted action due to fear of being tracked.
6. Society will be more transparent and open.
7. May reduce number of wrongful convictions.
8. Minimise time of investigation.
9. May be beneficial in faster and correct judgements in Courts, because evidence is easily
available and cannot be modified through use of money power.
10. Ensure that standard operating procedures were followed during any arrest/ investigation.

Concerns:
1. Privacy concerns.
2. Fear of such videos being manipulated or Wired.
3. Various social relationships based on private sphere may get damaged.
4. Camera being sign of mistrust of both police officer and fellow citizens towards each other.
5. Affecting dignity of women.
6. It may even affect one's culture or rituals. For instance, crime based in temple where
videography isn't allowed.
Especially in India, which is a multi ethnic, multi religious society. Already at many occasions, it
has been felt that police has not been able to win the trust of citizens. Including camera will only
increase the mistrust between the police and citizens.
7. Cost constraint. In fact a project of tracking the crime like CCTNS is only a fraction of this
project, which has been initiated in only 1-2 states [like HP].

Conclusion:
1. Considering the merits and demerits of Cop Cams, it may be argued that it can be implemented
in India to some extent. Prior to this, however, security has to be increased by installing sufficient
number of CCTVs in public places and providing latest transport facilities to cops to reach
incidental spots as soon as possible. Despite its positives, implementing it across India will be a
huge challenge due to concerns mentioned above.

2. Instances of fake encounters (Ishrat Jahan case), forced arrests and excessive use of force
(especially in areas of JnK) can be adequately addressed through this policy. Moreover, the use
of this policy can also help the police escape the false charges that are often slapped on them by
the accused in order to belittle their procedures. Overall, high cost of the tool notwithstanding, this
is indeed a good policy and must be followed in India.

Topic: Bilateral agreements involving India and/or affecting Indias interests

32) Critically examine the context and significance of the uranium supply deal that was
signed recently between Prime Ministers of India and Canada. (200 Words)
Introduction:

- India's economy has seen rapid expansion, resulting in a surge in demand for energy. As a result,
India is looking to increase its dependence on nuclear energy.

Present Condition:

- India has 22 nuclear reactors and plans to build some 40 more in the next two decades.
- India and Russia sign civil nuclear agreement, ensuring nuclear fuel supply for Kudankulam plant.
Uranium supply deal is also signed with Kazakhstan and Australia. Local uranium enrichment is
also gaining momentum by:
1. Expanding military gas centrifuge uranium enrichment facility at Ratnahalli enrichment plant
2. Building large unsafeguarded centrifuge uranium enrichment plant in Chitradurga District
(Karnataka)

But, India plans to generate 45,000 MW of nuclear power by 2032 - an almost 14-fold increase on current
levels.
Though agreement was reached in 2012 with Canada, but differences over the supervision of the use of
uranium in India delayed ratification of the deal.

Canada banned the sale of uranium and nuclear hardware to India after India used Canadian technology
to make its nuclear bomb.

It is pertinent to mention that globally Canada is the largest producer of


uranium; it is in this context that, the recent India-Canada uranium supply
deal under which Cameco Crop, Canada's largest uranium producer will supply
3220 metric tonnes of uranium concentrate for Indian nuclear power reactors
over five years, assumes special significance.

Importance of this deal:

1. It is also politically vital because long restriction of trade came to an end which shows trust in
Indias Non-proliferation commitments on international platform.
2. This deal will also diversify sources of uranium supply to India which is necessary for
uninterrupted fuel supply to India's Nuclear Power reactors.
3. Help in controlling climate change in India
4. It marks a fruitful conclusion of two years of protracted
negotiations that followed the 2013 civil nuclear deal between two countries.
5. India diversifies its uranium supply; as it already has
supply agreements with Australia and Kazakhstan.
6. Ensures energy security for India in future; beneficial
to economy.
7. Upholds our commitment to cleaner environment.
8. The deal is a booster for Indo-Canadian ties; marks an end to moratorium by
Canada on nuclear trade with India since 1974.
9. Presently GoI is focusing on nuclear energy; to meet growing energy needs of
our economy. By 2032, India is expected to have 45000 MW of nuclear capacity;
making assured supply of nuclear fuel (uranium) absolutely essential; moreover
indigenous uranium production is too meagre about 350-400 MT.
10. It will lead to diversification of import of nuclear fuel.
11. It will increase India's clout as a peaceful nuclear power as it will obtain nuclear material without
signing the NPT.

Issues:
The deal is significant yet certain issue remains such as
*it will not give a push to thorium based program which is abundant in India.
*also Canada belongs to NSG which act as a bloc so it may stop the supply of fuel hence
dependence on Uranium is not feasible in long term.

Topic: India and its neighbourhood- relations.

33) Secrecy in government operations is necessary, but it has to be limited by absolute


necessity, keeping the confidentiality strictly time-bound. In the light of recent debate on Official
Secrets Act 1923, critically comment on the statement. (200 Words)

Introduction

Government operations especially when related to security aspects of the state do contain secrecy as its
main necessity. But any such secrecy or confidentiality must be reasonable and not curb the
dissemination of the information to public as it goes against the objective of transparency and
accountability which are important for good governance. In this light the discussion on amending Official
Secrets Act has assumed attention.

Need for secrecy

Secrecy is necessary for many government operations for smooth and successful conduct. It can be seen
from the fact that British government enacted the official secrecy act in 1923. However, the context and
situation then was much different and the law needs to be explored from today's point of view. Secrecy is
definitely advantageous in certain cases like Security of nation, matters related espionage by foreign
countries. It enables the officials to function without fear and enables quick decision making. Besides,
certain sensitive matters are best disclosed only at the right time.

How much secrecy?

Level of transparency & secrecy in any Government depends upon the maturity of masses and
democracy. In western countries, the democracy has deep roots and people are highly educated and
demanding forcing Government to be more transparent and accountable. But in India the case is
different. People in India are less educated; democracy is still at immature stage. India needs to step up
its literacy- political, social and educational to have more transparent governance across levels.

Secrecy must be limited by absolute necessity and should be time bound

Often, secrecy is used as an excuse to shield information from even RTI queries, even though RTI act
itself mentions its overriding powers over Officials Secrets Act. While secrecy in government operations is
very much necessary, it cannot remain for perpetuity. Declassifying information after certain number of
years is essential. This is practiced in democracies like USA and UK, where Official secrets are regularly
declassified.
In addition, the declassified information would also be useful for the scholars to understand the history
better.

Problems/ Misuse of OSA

Increasing tendency among policy makers to shield under Official Secret Act which has vague
and broad provision often leaving room for arbitrariness. They tend to take close door decision.

Escaping from accountability by quoting the secrecy provision by government officials.

Public information can be kept out of public eye in the name of secrecy.

Time criteria of secrecy are often violated and the information is hidden for perpetuity, even when
the matter no longer requires secrecy.

Secrecy breeds corruption as these decisions are outside public scrutiny and lacks transparency.
For eg-2G scam and Coal gate scam.

In recent cases, this Act has deprived Indian public from accessing classified documents relating
to our freedom fighter Netaji Subhash Chandra Bose, the Henderson-Brooks report that deals
with Indias defeat in 1962 war. This information could dispel certain false propaganda and help in
proper understanding our history and government decisions.

Way Forward

Large number of Government's developmental work should be having utmost level of


transparency and accountability to bring efficiency by participation of masses.
There are areas like defence, security and issues of national interest like files related to Subhash
Chandra Bose which can disrupt Law & order situation in country. For such larger public interests,
the secrecy can be maintained but it should be properly scrutinized time to time for not being
misused for furthering anyone's personal interest.
In cases of corruption, maladministration etc., as stated by Supreme Court, executives should not
be covered under the veil of Official Secrets Act, or under the guise of national security.
If any information is being denied for an important public matter, officials involved must be
questioned and punished in case of guilt.

Secret information which doesn't hamper national interests after a certain period of time should
be declassified after that time period as is the provision in many developed countries like the
USA.

Inculcating the culture of voluntary disclosure of information by public servants.

Conclusion

The RTI Act, 2005 which aims at promoting a culture of transparency and openness in administration also
provides for exemptions under it. It is the misuse of this provision by citing Official Secrets Act on
unreasonable grounds which needs to stop. This will bring a balance between transparency and requisite
secrecy.
Topic: Development processes and the development industry- the role of NGOs, SHGs, various groups
and associations, donors, charities, institutional and other stakeholders

34) In your opinion, how should civil society conduct itself in a country like India? Do you think
the state should have a say in the conduct of civil society? Critically comment. (200 Words)

Civil Societies have a very important role in the progress and development of a country. They put forward
different demands of society and provide different views regarding the happenings in a country and have
come to be regarded as a representative of vulnerable and needy.

In India there is large section of people whose demands either do not have any voice or go unheard by
authority. India is striving for inclusive development and her civil society can be a good medium to
implement, execute and verify the programmes of government.

Civil society can provide representation to these unheard and voiceless people and put their demands
before government. It can oversee the implementation of programmes sanctioned by government and
report any misappropriation and delay in the execution.

To perform its tasks efficiently, civil society need to be unbiased and have only the development of
masses at heart. It should not act as impediment to genuine development and work in cooperation with
government without compromising with its duties.

As far as possible government should not have any say in conduct of civil society. If there is clash of
interests both sides need to short out it with discussion. Civil society can act as a great source of
information for the actual situation of masses to government. Government should harness this source
and correct its course of development depending on ground situation.

The state should be active in promoting and providing the right milieu for the effective functioning of civil
society. It should not try to stifle dissenting voices, but instead help in promoting a transparent functioning
mechanism for civil society organizations.

Topic: Effect of policies and politics of developed and developing countries on Indias interests,

35) Considering ongoing geopolitical conflicts across the world, do you think Non- Nuclear States
should push Nuclear States towards disarmament of their nuclear weapons? What factors have
determined Indias stance on disarmament? Critically discuss. (200 Words)

Nuclear disarmament refers to both the act of reducing or eliminating nuclear weapons and to the end
state of a nuclear-weapon-free world, in which nuclear weapons are completely eliminated. Although key
treaties such as CTBT (not yet in force) and NPT have been signed, due to several issues, these have
failed to either stop nuclear proliferation or encourage disarmament.

The reasons for which Non-Nuclear States should push Nuclear States towards disarmament are:

(a) High potential of humanitarian dangers from the use- deliberate or accidental- of nuclear
weapons by state and non-state actors
(b) Rather than acting as factors of deterrence, nuclear weapons have only made the world far more
insecure (Security Dilemma concept)
(c) A non-universal nuclear disarmament regime (in which the nuclear weapon states are given indefinite
rights of holding nuclear weapons) is not only unfair to non-nuclear weapon states but also encourages
proliferation of nuclear weapons

Thus the non-nuclear weapon states should push NWS towards comprehensive, time bound and
verifiable nuclear disarmament. Although India (especially Nehru) was one of the first few states to
have promoted cooperation towards the goal of non-proliferation and a nuclear weapons free
world, it is not signatory to either NPT or CTBT. The factors that determined Indias stance on
disarmament are:

(a) Indias suggestion of universal disarmament was not accepted. India did not support the division of the
world into nuclear haves and have-nots
(b) No time bound, verifiable nuclear disarmament plan of the NWS has been agreed to. Thus the NPT
stopped only horizontal proliferation but not vertical
(c) Indias geo-strategic considerations also play a role as we have two nuclear neighbours with
expansionist ambitions

However, by signing the Indo-US civil nuclear agreement, India has agreed to IAEA inspections (recent
signing of additional protocol) at 14 of its facilities. We have been a responsible nuclear power with a
commitment to no first use policy and a self-issued moratorium on nuclear weapons testing. India is now
open to signing the NPT, but only if it is recognized as a NWS.

Topic: Welfare schemes for vulnerable sections of the population by the Centre and States and the
performance of these schemes;

36) Discuss the distinguishing features of the National Rural Livelihood Mission (NRLM). Do you
think the design of the scheme is suited to meet its objectives? Critically analyze. (200 Words)

NRLM is a poverty alleviation initiative launched by the Ministry of Rural Development. Primary
objective - organise the poor into SHG (Self Help Groups) groups, make them capable for self-
employment and thus enable them to come out of poverty to lead meaningful lives.

The following are the distinguishing features of the same:

(A) Special emphasis on the most vulnerable social groups including - women, manual scavengers,
victims of human trafficking, persons with disabilities and bonded labours.
(B) The delinking of the process of selection of beneficiaries from the BPL list. Instead, a beneficiary
selection is through participatory identification process at the village level.
(C) Funds provided to the initiative are treated as resources in perpetuity.
(D) One of the objectives of the NRLM is that of universal financial inclusion and so efforts are made to
promote financial literacy among the rural poor.
(E) NRLM is designed to be implemented in mission mode.
(F) Key processes of the NRLM are driven by the rural poor especially women, who have themselves
come out of poverty.

Therefore, on the plus side, the NRLM is designed as a programme for the poor, of the poor and by the
poor. Its implementation in mission mode will provide the States enough room to tailor this initiative
according their own requirement and needs. However, the design of NRLM issue raises some critical
issues:
(A) The thrust of the NRLM is upon SHGs to usher in livelihood opportunities. There is nothing novel
about this approach. Also SHGs have a mixed performance record when it comes to such programmes.
(B) Not everyone in rural area may be a member of SHG group. Therefore, making it mandatory to be a
part of SHG for access to various services under NRLM may exclude some people from this system.
(C) There have been many instances where SHGs have been taken over by the elite among the rural
poor. This could lead to alienation of the real beneficiaries.
(D) NRLM has not given serious attention to value added agriculture and rural MSMEs, which play
an important role in enabling and sustaining inclusive growth in rural areas.
Despite having good intentions, NRLM comes with an array of loopholes in its design which require
immediate redressal on part of the govt to prove its pro-poor stance.

Topic: Issues and challenges pertaining to the federal structure, devolution of powers and finances up to
local levels and challenges therein.

37) A 2013 central bank study found state expenditure to have a far bigger beneficial impact on
growth than spending by the Union government, which is much more likely than states to use
borrowed funds to make payments. Examine the sources of funds for states, recent trends in
devolution of funds to states by the union government and how can states spend the money
effectively for the welfare of the people.

Source: http://www.business-standard.com/article/opinion/andy-mukherjee-the-states-race-to-spend-
115042101266_1.html

Introduction:

State expenditure definitely has bigger beneficial effect on growth than spending by Union govt.
because most of the subjects related to larger social welfare effect on Indian conditions are in
state list viz. agri, education, healthcare etc.
Also union govt have a tendency to spend from borrowed funds which roots out money from
market for pvt investors by increasing interest rates.
State expenditure with borrowed funds from World Bank, ADB and from states share of the
revenue provides better decision making capacity which enhances interest & political will thus
grooming accountability, responsibility & transparency leading to better governance.
States are the major stakeholder in success of any flagship program or infrastructure projects.
The development of any state is more in consonance with the political will of the state than the
centre.
Example: Gujarat, Maharashtra & recently Bihar did exceptionally well in improving the human
development indicators.
Sources of funds to state:

Devolution of funds for states under statutory and discretionary grants.


Taxes that states levy on the people like VAT, Sales tax, Octroi, Excise etc. The new GST if
implemented would make states get more devolution of funds and also as compensation from
centre to some.
Special grants like under Special status states like J&K, NE states, Himachal Pradesh etc.
States can borrow funds from the market.
States can borrow funds from Nationalised banks, RBI and Union Govt.

Recent trends:

Accepting FFC recommendation 42% share in Tax proceeds which is approx addition 2 lac crore.
Greater autonomy and Delinking with Previous restriction which were there in CSS schemes to
the states. It is now upto states how they can utilise it
No distinction between general and special category states: rather tax devolution to fill the
resource gap among states based on fiscal and cost disability as well as expenditure
responsibilities. ( pushing states to spend prudently)
On tax front Building consensus on GST and proper compensation till 2017 , to Tax lost for some
states
Share from resource allocations. Example: coal
Foundation of NITI Aayog has strengthened cooperative federalism.

Effective way of spending money by States:

Funds with rationalising subsidies can follow golden rule of economy that is borrowing /using
funds for capital investment
Increasing purchasing power through employment elastic growth that is promoting MSME
Cooperative federalism that is congruence of Schemes among neighbouring states to realise the
outcome of funds can pave way for Holistic growth environment ,
Micro planning and decentralisation of Powers and social audits for effective implementation and
monitoring of funds and their use. Hence "participative governance"
Scenario planning in cases if one plan fails than other plans to be ready
Skill development and promotion of Health education and sanitation
Funds to be Gender budgeted so that Full demographic potential is realised
Monitoring mechanism should be strengthened
Transparency and use of IT tools to insure accountability
Need based, long term planning and performance based funds
Capacity building in states is very poor, hence need for upgrading capacity building
Shouldn't resort to populism as has been witnessed in many states like Uttar Pradesh etc.
Should ensure that funds are not used primarily for bridging revenue deficits, but also for capital
investment
The states should further devolve these funds to PRI to ensure even more adequate utilisations
of resources.

Conclusion:
Central government has also indicated to change their role from the provider to the facilitators
and also move towards the cooperative federalism, they have taken their step, now it is for the
states to act on it.
As states are responsible for Health, Education, Sanitation, they can spend for overall Human
development that boosts HDI and turns into economic growth for holistic development.
Still, the opportunity is ripe for States to make an efficient use of the current trend of devolution of
funds by improving transparency, accountability and linking grants with performance.

Topic: India and its neighborhood- relations.

38) China-Pakistan economic ties have long been the weakest leg of their relationship. In
contrast how do you analyze India China economic ties? Discuss in the light of the statement.

Source: http://indianexpress.com/article/opinion/columns/two-birds-one-46-billion-stone/

Introduction:

It was the geopolitical situations in South Asia that brought China and Pakistan closer and made
them strategic partners.
But in this strategic partnership, economic ties have never realized its full potential viz-a-viz to
India. Even after decades of bonhomie, China and Pakistan trade is nothing close to China-India
trade around $70 billion.
China has favoured Pakistan because of:
o Balance Indias growth in region.
o Frustrate India by showing pro-Pakistan stance like what China believes India show as
pro-Tibet stance.
o Supporting Pakistan will increase chinas newly formed defence equipment industries.

Reasons for weak economic ties between China-Pak:

Due to the political instability in Pakistan


But when it comes with economic ties, China delayed concerned by security, terrorism, poor
growth of Pakistan.
It can be attributed to the concerns arising from internal instability within Pakistan. Though
Gwadar port project was conceptualized decades ago, it has not seen its fruition.
Pakistan with economic liberalisation a decade before India has failed to exploit the opportunity to
boost its trade due to political instability.
China which was on trail to boost its economy in double digit in last 3 decades has not turned to
any state which can prove ill-fated for its dream to be economic superpower.
Pakistan has not been able to liberate its economic policies from the clutch of its strategic vision
whose primary aim was to counter India. This weakness has been exploited by China to counter
India by supporting Pakistan thus boosting their strategic relation.
Example: Due to failure of pipeline link between India-Iran-Pakistan, china is also not easily
ready to invest in Pakistan.

India-China Economic Ties:


China is the largest trading partner of India.$ 70 bn trade is highly skewed in favour of China.
Indo-China trade target of $500 billion by 2020 is very much achievable in more balanced
framework. China providing its expertise in high speed rail and industrial parks in Gujarat &
Maharashtra.
The 6 point template of (Action oriented approach; Broad-base bilateral engagement;
Convergence on common regional & global interests; Develop new area of cooperation; Expense
strategic communication; Fulfil common aspiration to a self-Asian century) on the path of Indo-
China cooperation is a big step.
Today China is sitting where, India is aspiring for that seat i.e. industrial giant for the region.
China doesnt want to give it up which is visible with articificially keeping the Renminbi at low so
that their export becomes competitive.
Also, china doesnt import Indias any item which it feels threatening her domestic market. This
led to huge trade deficit (40 billion $ on 2014)
China doesnt want to capture Indias market which she knows is impossible hence, China
economic agenda is to contain India from all side by investing on all surrounding nations, so that
China can have monopoly on all export in the region.
China recently announced to invest on Indias IT companies which is also in backburner and
delayed.

Conclusion:

China has policy of delaying project and creating an environment of diplomatic ties to boost her
export for the time being.
China has higher aspiration of economic corridor to Europe, One belt one road, Maritime silk
route to reach wider market to sustain her growth.
While China-Pak relations have been that of "lender-borrower only to satiate the economic
interests of China, India-China stand to be "partners in driving the Asian and Global Economy.

Topic: India and its neighborhood- relations.

39) In the light of ongoing regional conflicts in the West Asia, critically analyze the relationship
between Pakistan and Saudi Arabia and its implications for India.

Source: http://www.nytimes.com/2015/04/22/opinion/toward-a-saudi-pakistani-rift.html

Introduction:

The Sunni-Shia dynamic finds a more political expression in the rivalry between Saudi Arabia and
Iran that are backing competing groups in the various civil wars. If Tehran has lent support to the
Bashar al Assad regime in Syria, Riyadh is arming various rebel groups trying to overthrow the
government in Damascus.
Pakistan has long had a close relationship with Saudi Arabia and has been involved in protecting
that country and the House of Saud.
An example of Saudi Arabia's favours to Pakistan include provision of oil to Pakistan in 1998 to
help it weather international sanctions against it for conducting a nuclear test.
The Saudis also saved Nawaz Sharif after he was overthrown in a coup in 1999.

Relationship b/w Pak and Saudi Arbia:

The Sunni majority of both countries are a major force that binds both.
Cultural and religious relations with Islamic Shrines connecting people and both having Islam as
state religion. Both the countries are very ideological and have a Sunni-majority
Cooperation of Pakistan with Saudi against Iran in international negotiations.
The relationship with US for both in exchanging support at areas of military cooperation or strikes
against rival countries.
Saudi Arabia has allegedly provided financial assistance to Pakistan for its strategic program
(missile and nuclear weapons) in return for a promise that this would provide a security umbrella
for Saudi Arabia.
Pakistan has provided military aid and expenditure to Saudi Arabia (especially the Air Force) and
some Pakistani troops are also directly deployed in Saudi Arabia.
Millions of Pakistanis work in Saudi Arabia. People ties with Pakistani youth working in Saudi Oil
fields.
The Saudis want Pakistan to act as a counterweight to Iran, and have long cultivated a close
relationship with its military
For Pakistan, Saudi Arabia is not only a long-standing source of aid but a principal source of
foreign exchange through much-needed remittances.
Being drawn into the Middle Easts sectarian battles carries a greater domestic and regional risk
for Pakistan than it does for most of the Saudis other partners due to need to develop a
favourable relationship with Iran to solve its own Taliban problem

Implications for India:

Though strain in the relations between Saudi Arabia and Pakistan is definitely in Indias favour,
Indias needs to balance between Saudi Arabia and Iran, the two adversaries who happen to be
New Delhis allies.
India needs a multi-vector policy and maximising every bilateral tie irrespective of the size and
political weight of the country.
Indias benign image and non-interventionist policy puts it at an advantage compared to other
West Asian powers.
If Pakistan agrees to deploy its nuclear weapons on Saudi soil by way of an extended nuclear
deterrence, this would strengthen Islamabads second-strike capabilities
Given increased deterioration in the US ties with the two Sunni nations, Washingtons influence is
slowly waning from there and this could lead to better ties with an energy hungry China in turn.
Trade between Saudi Arabia and India is on the rise and relationship with Pakistan will have
significant impact on that.

Conclusion:
India can be benefitted by moving in quickly to provide help to Saudi Arabia which lack in
education and military strength and have a long term deal on oil from Saudi Arabia
India must maintain equal relations with all for sustaining its growth and development which can
be termed as Non Alignment at work in the 21st Century.

Topic: Development processes and the development industry- the role of NGOs, SHGs, various groups
and associations, donors, charities, institutional and other stakeholders

40) Its been proposed that an autonomous accreditation authority National Accreditation
Council of India or NACI and a mechanism to rate NGOs should be set up to address some of the
crucial issues related to functioning of NGOs. Critically examine the merits and demerits of
implementing these proposals, especially in India.

Source: http://www.thehindu.com/todays-paper/tp-opinion/ngos-the-good-bad-and-the-
ugly/article7127580.ece

Introduction:

A non-governmental organization (NGO) is an organization that is neither a part of a government


nor a conventional for-profit business.
NGOs in India have strived hard to achieve the goal of social and economic democracy besides
only political democracy as enshrined in our constitution.
Recently, several NGOs have been accused of not understanding the peoples plight, as the
members are themselves are from well off sections of society, and also that, some NGOs are not
doing any real work on ground, besides only creating unrest in public order.
There are also issues related to their vested interest running opposite to nations larger interest,
dubious funding, false propaganda etc. which need to be addressed in a rational manner.

Merits of having accredation agency (NACI):

It will lead to an increase in investor confidence and betterment of public perception.


The deserving NGOs will get formal recognition for the work done on ground, which will help in
motivating them even further.
The NGOs will be able to compete with political parties, who claim to have a better platform, as
the reputed NGOs will also be able to obtain adequate funds from the govt, thus removing the
problem of fund shortage, and they will also be at a stage to demand a say in policies regarding
social initiatives.
The govt would be able to outsource their work of social impact assessment of any public policy
to reputed NGOs, and save the time of govt depts., avoiding the unnecessary delays in execution
stage.
Can help bring out the outstanding performers
Transparent process can reward more productive organisations
Motivation for good organisations to expand
It will act as a guidance for donors, increase the investor confidence, could check for compliance
of regulatory norms, will provide a positive public perception
Equal representation of both government and voluntary organisations in NACI would reflect a
positive and collaborative spirit which is beneficial for our country.
Weeding out of a large number of non performing NGOs which have mushroomed in recent times

Demerits of having accredation agency:

In India around 25 lakh NGOs are working on diverse fields. So ranking them on a set of
conditions is not an easy job.
It will not be able to adequately rank the NGOs, which have intangible results of their activities,
like those only campaigning to remove a social evil from a society. The ranking agency will not be
able to link the actual achievements on ground with their activities.
It may lead to the autonomy of only some of the NGOs due to large fund availability, which will
demotivate the other organisations, even if they are doing an equal amount of hard work.
The list can be politically motivated too, to provide credence to a particular NGO.
Biased ratings can help corrupt organisations
No definite parameters to measure effectiveness of the NGOs work
Could affect autonomy and pure humanitarian goals
Since the NGO sector is diverse in purpose, nature and approach, standardized measurement
methodologies across sub-sectors would not provide a true picture.
It can hamper the independence of NGO's and put financial constraints.

Conclusion:

In India, such a ranking system can be helpful, as they can serve as a nice supplement to the
efforts of the central govt and it will help in bringing recognition to the worthy NGOs.
But Instead of ranking if the agency will try to only regulate and monitor them effectively like
making their fundings more transparent, clear the doubts related to promoters or members
conflict of interest etc. It would serve the purpose more rationally while keeping the NGOs
autonomy intact.
Such body with transparent structures and functions shall be fruitful in standardisation in their
performance.

Topic: Issues relating to development and management of Social Sector/Services relating to Health,
Education, Human Resources.

41) .while the global economy is expected to witness a shortage of young population of around
56 million by 2020, India will be the only country with a youth surplus of 47 million. In your
opinion, what challenges does India face in optimally utilizing this demographic surplus for the
development of the country? Critically examine.

Source: Economic Survey 2015, Volume 2, Chapter 1

Introduction:
India is at a stage where the labour force temporarily growing more rapidly than the population
dependent on it, so as, freeing up resources for investment in economic development and family
welfare.
65% of the population is India is less than 35 years of age hinting towards its young age
composition and demographics.
A country with both increasing numbers of young people and declining fertility has the potential to
reap a 'demographic dividend' a boost in economic productivity that occurs when there are
growing numbers of people in the workforce relative to the number of dependents
Although it sounds optimistic, but still there are several challenges to be overcome, if its to be
utilized properly.

Indian economy is based on domestic consumption unlike China or Japan. Indian Youth needs:

Proper health measures


Cheap but quality Education
Skill development to contribute to economy.

Various Challenges:

Education
o Currently Indias education system is well below global standards. RTE must be properly
and qualitatively implemented with proper funding.
o Diploma, Degree and Certificates courses should focus more on skill development
o Policy paralysis in education sector, to provide adequate and compulsory education at a
large scale, leaves a large part of the youth as unemployable.

Employability issue:
o Only about 6% of the Indian workforce is employed in organised sector because of
unskilled labour force. Therefore providing adequate skill sets to over 90% of the non-
farm labour force will be a humongous challenge.
o Low initiatives to encourage entrepreneurship also inhibit the progress of the country
o Promoting growth of micro, small, and medium enterprises (MSME) to facilitate long term
employment to youth is vital.
o Promoting the domestic consumer base is of utmost importance. Then only the pace of
quality employment generation will be sustainable.
Funding/Financial Issues:
o Structural reforms needed to remove bureaucratic hassles, swift project implementation,
consolidation of the fiscal deficit, increase in FDI and insurance sector. But it should not
hurt farmers and poor
o Multiplicity of labour laws and difficulty in their compliance has been an impediment to
industrial development; these laws should be simplified and made contemporary to other
nations laws.
Skill Development:
o Skilling of rural youth need to refocused by National Skill Development program with
inclusion of minorities so that necessity reservations in jobs decrease from root of
society.
Social Issues
o Gender inequality, stereotypes are stopping half of population to realize its potential.
o It needs to be ensured that the secular fabric of the country is not weakened and the
nation is free of all the distractions like communal disharmony.
Health is of prime importance. Young India should be healthy to achieve its potential.

Iniatives to overcome:

Combining the initiatives of Make in India and Skill India, so as to provide skill and employment
both, to the youth of the nation.
Medical facilities should be encouraged to check the spread of diseases in rural areas.
Initiatives like Beti Bachao, Beti Padhao can also help in achieving gender equality and more
contribution in nations development.

Conclusion:

Good & Qualitative Governance can bring above mentioned changes. Politics & Radicalism in
reforms will derail development. So all governments (Union and State/UTs) must comply with
inclusive growth of country.
We are at crossroads with wide opportunities and challenges. The experience of the aged and
the enthusiasm of the youth is the best mix that our nation must have.
Ensuring proper resole of above challenges will empower our youth and help in success of Make
In India and make India a Super Power.

Topic: Issues relating to development and management of Social Sector/Services relating to Education,
Human Resources.

42) Achieving gender parity in school education is sine qua non for ensuring the larger goal of
women empowerment. In this regard, critically examine the progress made, roadblocks
that exist and measures needed to overcome these roadblocks in achieving gender parity
in school education. (200 Words)

Historically, girl education has been a challenge to the government for a varied number of social, cultural,
economic reasons. Some of the major causes have been socially driven, culturally upheld and
economically facilitated

Women Empowerment is 3rd of the 8 MDG, and given top priority for the Sustainable Development Goal
2030, this itself reflects its importance at global platform.

- School education is the cradle of the prejudice development. It imbibes various social frameworks which
set with individual till it serves mankind. Therefore, gender parity at school level both primary and
secondary is prerequisite for achieving gender equality.

The progress made in this direction can be elaborated by:-

PRIMARY EDUCATION

National Educational Policy,1986 gave special emphasis on school education of girls


Launch of District Primary Education Programme launched in 1994
Sarva Siksha Abhiyyan,2001

India achieving MDG of gender parity in primary School enrolment in 2008


Art 21 A and RTE,
Sukanya account,
ICDS and provisions like midday meal

Roadblocks:
1). Patriarchal society and bigotry approach of male member towards girl education.
2). Poverty, lack of sanitation facilities
3). Insecurity and lack of school infrastructure in nearby areas.
4). Lack of legal mechanism & grievance redressal for improving the status
5). Social negligence and attitude.

Governments efforts:

i. Providing bicycle facilities to make girls self-reliant & confident Gender sensitisation by co-
education and games.
ii. Curbing drop-outs by providing sanitation facilities.
iii. Beti Bachao Beti Padhao Abhiyan
iv. Hostel & scholarship to continue and pre-matric & post matric education

v. Rajiv Gandhi Scheme for Empowerment of Adolescent Girls (RGSEAG) Sabla aims at all-round
development of adolescent girls of 11-18 years by making them self-reliant.
vi. Better infrastructure through Shala Darpan under the Swacch Bharat Abhiyan is taking up
measures to provide cleaner toilets for girls
vii. Scholarship programmes like UDAAN is an initiative for doubly discriminated SC/ST girl children
to provide professional education in mathematics and science
viii. Health department is also providing folic acid tablets to provide for better health of the girl child,
who is more prone to anaemia
ix. With cooperative the voluntary sector in order to create infrastructure as well as awareness
amongst the communities to bring about a change in the social- understanding.

Measure needed to overcome the roadblock:


1). Awareness among society about the silent apathy against women.
2). Economic empowerment of women will have trickledown effect on social empowerment which will
change the environment of girls.
3). Legal security and institutional mechanism for grievance redressal.
4). NGO, private sector and government should adopt holistic approach as facilitator of enabling
environment.

Addressing roadblocks is necessary for achieving larger goal of women empowerment, as gender parity
at school level helps to inculcate a sense of equality among both genders.

Livemint
Topic: Indian Constitution- historical underpinnings, evolution, features, amendments, significant
provisions and basic structure.

43) The core of the commitment to the social revolution lies in parts III and IV of the Indian
Constitution. These are the conscience of the Constitution. Critically comment. (200
Words)

For a newly formed nation which was torn by economic exploitation and socially divisive schemes of an
imperial power, and also suffered various age old discriminations and dissensions, it was necessary for
the Constitution framers to devise mechanisms with ample safeguards.

1. Mostly social revolution is aimed at establishing egalitarian society by abolishing existing


disparities, old faultlines, promoting sense of equality and same legal status for all citizens
irrespective of their social origins or stature. Our leaders had shown Socialist attitude during
freedom struggle and commitment for upliftment of poor and backward class. The inclusion of
Part 3 and 4 testify this.

2. PART 3 act as a check on the govt against applying autocratic rules on the society,

3. PART 4 direct the govt on various ways to introduce the concept of a welfare society in the
country, by ways of using the Gandhian and the liberal intellectual standards.

--But there have been some criticisms regarding both parts of the constitution.
FRs have been criticised to be vaguely defines in several respects, like the words public
interest, reasonable restrictions, etc. have not been properly defined.
The right to ask for remedy is too expensive due to the long litigation process, which
makes it unapproachable to the majority part of the society.
The clause of preventive detention is also a unique feature, not found in any other
democratic country of the world.

Regarding DPSPs, they have been said to be haphazardly arranged, with the least
important directives, mixed together with the important ones.

Their non justicability is also the source of much criticism. Also, they have been seen to
be too conservative, when compared to todays modern society.

At times, there have been several instances of conflict between the two, leading to
various amendments being passed in the parliament

While the Preamble has ornamental words like Justice (social, economic and political),
Liberty (speech, thought, expression and religion), it is the Part 3 and Part 4 that give the
real text to such end. Quite aptly, Supreme Court has held all three to be basic features
of Constitution.

Bipan Chandra, India Since Independence, Chapter 4


Topic: Indian Constitution- historical underpinnings, evolution, features, amendments, significant
provisions and basic structure.

44) Write a brief note on the basic features of the Indian constitution. If you are asked to alter
or replace any of these basic features, what would they be? Justify. (200 Words)

Keshav Nanda Bharati case (1973) evolved "BASIC STRUCTURE" doctrine. The basic structure concept
implies that the Parliaments power to amend Constitution (Article 368) is not unfettered. So any law or
amendment made are subject to the judicial review and the judiciary has the power to struck it down if
found ultra vires Basic Structure reflects the understanding of the thought and wisdom of our founding
fathers and seeks to curb any autocracy on the part of the State.

Basic Features:
-FR, FD, DPSP
-Written lengthiest constitution
-Federal with Unitary Bias
-Borrowed from multiple sources
-Judicial Supremacy and Parliamentary sovereignty
-Single citizenship
-Adult Franchise
-Emergency Provisions
- Secular state
-Three tier sys

The following items merit some debate over their inclusion in the basic structure:
1. Judiciary's independence. Absolute independence to any organ of the State should be discouraged to
keep checks and balances.

2. Directive Principles can be made semi-justiciable in certain topics like implementation of uniform civil
code, etc. The government should be asked to submit a plan if action towards this end.
But still the status quo of the Basic Structure is almost sufficient to address concerns over amendability of
the Constitution. The Supreme Court keeps modifying the list of Basic Structure.

There is no fixed list of these features and SC keeps on adding them from time to time leading to
uncertainty and ambiguity in Indian political discourse.

However, in-spite of these, this doctrine has emerged as the cardinal feature of Indian Polity and ensured
check and balances in the Political system!

Bipan Chandra, India Since Independence, Chapter 5


April Paper 3

Topic: Science and Technology- developments and their applications and effects in everyday life

1) Why nuclear deal talks with Iran by the West are cantered around curbing the number of centrifuges
that Iran should be allowed to possess? Examine the science behind this condition with suitable
illustrations. (200 Words)

The New York Times

The purpose of the deal is to limit Iran's nuclear program to something that is small, safe, and peaceful.

Technical reasons behind this deal are:

Uranium and thorium are natural elements that can undergo nuclear reactions and produce
energy, excessive plutonium and uranium.

Uranium consists of two isotopes U-238 =~99.2%, U-235=~0.72%. U-235 is fissionable and U-
238 is fertile.

To produce power in reactor huge quantity of U-238 or less quantity of U-235(=~3-5%) are
required and yield is high in reactors operates with U-235. enrichment up to 3-5% is required for
power production economically.

Enrichment beyond 20% can be used as nuclear bomb.

Uranium can be enriched by:

1. Gas centrifugation: Uranium is converted to gaseous state and centrifuged in gas centrifuges

2. Laser enrichment: selective separation of U235 by laser

3. Membrane separation: U-235 selective membranes are used to separate U-235 from U-238
etc...
Of all enrichment techniques, Gas centrifugation is economical. Less number of centrifuges
reduces enrichment beyond threshold and hence deal is focusing on curbing centrifuges.

Right now, Iran is having Centrifuges of 9000 rpm which is not very efficient but if cascaded thousands of
times it can enrich the Uranium up to 20% which is much above the 5% enriched requirement for fuel for
NPP. By this treaty, Iran can have Centrifuges with 6000-7000 rpm and enrich only up to 4% for NPP fuel.
Iran US deal impact on India

Positives:

Current bilateral trade between India and Iran is about $14bn (8.96bn) with the balance of trade in
heavily in Tehran's favour. Indian exports to Iran were around $4.2bn last year.

India primarily imports oil from Iran, but has been hampered by restrictions placed by global powers.

Due to the sanctions, India has been paying Iran in Indian rupees, with the money kept in an Indian
account. In fact, the country is yet to release an estimated $6bn in pending oil payments to Iran.

Now Delhi, which is the fourth largest consumer of oil in the world, is free to import Iranian oil but will have
to pay in dollars.

Importing goods or sending shipments to Iran is currently expensive because of high shipping charges.
India hopes the removal of sanctions will make it easier for companies to get shipments.

Negatives:
India has been exporting automobile components, tools, motors and chemicals to Iran. Indian exporters
will have to compete with Eastern European manufacturers who produce low-end products like spanners,
hand tools and auto parts. Since the value of the euro has depreciated in the last few years, we will be
facing stiff competition from European manufacturers.

Now Iran is no more side-lined. It negotiating power has increased and India will have to meet
competition from other countries now.

Topic: Infrastructure energy; Paper 2 Effect of policies and politics of developed and developing
countries on Indias interests

2) There is a call for granting regulatory independence to the Atomic Energy Regulatory Board (AERB) by
law. Examine why. (200 Words)

The Hindu

Atomic materials and atomic energy are governed by the Atomic Energy Act, 1962. The Act empowers
the central government to produce, develop and use atomic energy. At present, nuclear safety is
regulated by the Atomic Energy Regulatory Board (AERB). Some of the drawbacks of the present
mechanism are discussed below.

Key issues under the present nuclear safety regulatory mechanism

The AERB is not empowered to operate as an independent operator.

The AERB was established by the government through a notification and not through an Act of
Parliament. Its powers and functions are therefore amendable by the Department of Atomic Energy
through executive orders. The parliamentary oversight exercised upon such executive action is lower
than the parliamentary oversight over statutes.

Furthermore, the Atomic Energy Commission that sets out the atomic energy policy, and oversees the
functioning of the AERB, is headed by the Secretary, Department of Atomic Energy. This raises a conflict
of interest, as the Department exercises administrative control over NPCIL that operates nuclear power
plants.

Various committee reports, including a CAG Report in 2011, had highlighted the drawbacks in the present
regulatory mechanisms and recommended the establishment of a statutory regulator

To Change this scenario, the Nuclear Safety Regulatory Authority (NSRA) Bill was drafted by the
DAE.
Vital provisions:
1. to establish The Nuclear Safety Council, oversee these policies includes the Secretary, Department of
Atomic Energy.
2. The Bill provides that members of the NSRA can be removed by an order of the central government
without a judicial inquiry. This needs to be changed.
3. NSRA must be established in parallel safety provisions of IAEA.
Concerns raised about bill

Regulatory mechanism may not be independent

The provisions of the Bill may not secure an independent character to the NSRA. In particular, the
provisions of the Bill related to composition of the Council, the composition of the search committees and
the procedure for removal of the members may affect the independence of the NSRA.

AEC Chairman as a member to the Nuclear Safety Council

The Bill establishes the Council to oversee and review nuclear safety policies. The Council includes the
Chairman, AEC, who is also the secretary to the Department of Atomic Energy. The Department controls
the functioning of NPCIL. Thus, the Chairman, AEC is on the Council that reviews safety policies, and
also heads the administrative department that controls nuclear power plants. This could lead to a conflict
of interest.

The Chairperson of NSRA would be on the search committee for other members

The Bill provides for separate search committees for the Chairperson and the members of NSRA. The

Chairperson of NSRA will be on the search committee for members. The strength and composition of
these committees is not provided in the Bill. This raises two issues.

The Standing Committee recommended that the number of members to the search committee be
specified.

It stated that the composition of the search committees would play a vital role in the scheme of the Bill.

Second, the Chairperson of NSRA would be on the search committee for the remaining members. This
may affect the independence of the NSRA as each member is expected to make independent decisions
on each case. Some recent Bills that seek to establish search committees for regulatory bodies do not
have similar provisions.

Removal of members to the NSRA

The Bill provides that members of the NSRA can be removed by an order of the central government. In
case of misbehavior (acquisition of a financial or such other interest), such removal may be ordered after
providing the member a reasonable opportunity to be heard. However, the Bill does not require a judicial
inquiry to be conducted before removal. This is different from the process provided under some other
legislation. For instance, the Competition Act, 2002, the Right to Information Act, 2005 and the Protection
of Human Rights Act, 1993 require a judicial inquiry prior to the removal of the Commissioners if there is
an allegation of misbehavior against them. The Electronic Delivery of Services Bill, 2011 and the Lokpal
and Lokayuktas Bill, 2011 also have a similar inquiry procedure.

The bill is a step in right direction. However above lacunas may be removed and AREB may be given real
independence to perform its function independently.

For secure nuclear outlook and precautions to any catastrophic incident, India needs to change role and
status of AERB.
Topic: Security challenges and their management

3.Critically compare and contrast how India and China are managing security issues such as
evacuating their citizens from conflict zones, setting up foreign bases etc outside their
respective borders.

The Indian Express

India and China both having significant population living beyond borders, extricating compatriots from
zones of conflict or natural disaster has becoming a recurring challenge

THE DIFFERENCES

1. While china's interests go beyond the borders and securing them is a top priority, India has a natural
advantage when it comes to its neighbouring states e.g. Bangladesh, Nepal and sometimes Pakistan.

2. China has its advantage when it comes to defence establishments. It is helping china's political
commitment in establishing appropriate polices and institutional capabilities.
In India however strategic efforts fall back in these contexts. Even during the present crises the political
leaders turn to the armed forces as the instrument of first resort.

3. China has a growing presence of its military outside Chinese perimeters. The same will be assisted by
the new military posts that will come up outside Chinese territory.

India doesn't have that much presence. It uses its bilateral and foreign relations and its evacuation is
more strategic, than military as compared to China.

4. China is being seen as new economic power for middle east and as a counter to US. With maritime silk
route project it has garnered wide support in the region and encashses this trust in its evacuation plan.

India is doing its economic penetration in selective areas and is still way behind China in outreach and
scale.

6. India has advantage of being a SAARC country as citizens from SAARC countries form a major
chunk of workforce in gulf countries. At the time of distress all countries have to evacuate their
citizens. SAARC together can provide a more strengthened course of action to evacuate all
SAARC citizens utilizing the synergy of individual members. China doesn't have that leverage.

7. India with its policy of non-interference has always stood ahead of china when it comes to
international citizen and responsible power. The peaceful relation with its neighbours including
Pakistan is a driving force for its familiarity with such expeditions.

Thus China is a singular player, much wider in its military, financial, economic and strategic strength and
can go "solo" for executing the evacuation plan. India is still "catching up" and will have to rely on
goodwill, foreign relations and accumulative response from neighboring countries for any future
evacuation plan.

3) Do you think the new monetary policy framework signed between government of India and the
RBI will help the latter get more autonomy? Critically examine.

The proposed framework establishes a Monitory Policy Committee(Urjit patel led MPC's
recommendation) of five people with three people from RBI and two from non RBI field preferably Finance
minister and other eminent person in field of Economics etc. This provides flexibility and a forum to
negotiate differing view with a majority to the Central bank thus increasing transparency in monetary
policy making and filling the lacunae of single mindset.

MPC can help RBI get better autonomy in the following ways:

1) Committee: Now a committee with experts will be responsible for deciding the monetary
policy, earlier only RBI governor was responsible for this job.

2) Target: CPI targets have been set by the Government for the coming year which ranges from
4% +- 2%, this will give RBI the space to work around rates to achieve the target. RBI will also
have the window of three quarter to achieve the targets in case they miss it by any margin. In
case if it missed it will be liable to report to the centre about the causal failure accompanied with
the redressal mechanism.

However, there are other angles to the very basis of CPI and MPC formulation which not only can
create challenge for RBI in containing inflation but can also curtail their autonomy

a) Inflation and its reason: A major reason for the CPI inflation in India is food prices and the
bottlenecks in supply chain, both are out of the scope of RBI and hence RBI virtually has no
control on inflation in practical sense.

b) MPC: If the government appoints any of its members particularly any bureaucrat in MPC, it can
create undue influence on monetary policy, which will thwart RBI autonomy.

c) Public debt Management Agency: The Government is also planning to set PDMA, which will
pull away RBI authority to contain public debt. Until now debt mgmt was under RBI domain,
setting independent office under govt. will reduce autonomy and create a milieu of conflict of
interest as govt. is important player in bond market

d) FrBM and govt. debt line, that is Fiscal responsibilities are frequently challenged or crossed.
In this sense targeting inflation may be an issue

e) Besides the interest rates control no other possible means of controlling the financial market
seems visible. Fiscal irresponsibilitys may cause forever tight monetary regime hurting Investor
sentiment.
Government support & faith in MPC shall be welcomed and Regarding Indian economy ,it is the
new economy and from past experiences what worked abroad has not succeeded here ,Thus
suitable time for experimenting ,trial and error shall be imparted to the Diligent institution for
greater Public interest.

4) Critically examine the importance of findings and recommendations of the Interim


Report of the Bankruptcy Law Reform Committee for Indian economy

Indian laws are often termed as the toughest laws for the running of businesses in India. To improve
business condition in the country, Bankruptcy Law Reform Committee (TK Viswanathan committee) was
setup by the government in Aug, 2014. Some of the recommendations of the committee which submitted
its report are-

1. Early recognition of financial distress in company and timely intervention by the


government to rescue the organization

2. Liquidate un-viable company as soon as possible Currently it is normal practice to


liquidate companys assets to minimize losses to stakeholder if company goes bankrupt. But
it ignores employees working in that company, their future and their liabilities, which are also
responsibility of company.
3. Allow secured creditors apply for the rescue of the company, earlier it was filled after the
company have been defaulted by 50 per cent of its outstanding debt. There is no defined
framework to rescue company from default. Interim report suggests allowing sufficient space
and time for a rescue by new creditors, unless it defaults on 50 percent debt. This can help in
stabilizing assets of company. In short, the message is for early intervention for rescue.
4. Unsecured creditors representing 25 per cent of the debt be allowed to initiate rescue
proceedings against the debtor company This recommendation, if implemented, will
encourage investors investing in companies by subscribing to debentures to take further risk
as well as take part in the funding process along with banks.
For banks, SARFAESI Act is there, there was nothing much for the unsecured lenders in
India.

The report also seeks to give creditors a say in determining its sickness. The committee reasons this by
stating that viability should be the most important consideration for allowing a company to be rescued. As
per the current provisions of the Companies Act, the viability of the business can be considered by a
committee of creditors only after a company has been declared sick, which itself will take 60 days for
determination after receipt of initial application by the NCLT.
The NCLT will have the discretion to grant, refuse or lift a moratorium to a company if it feels that the
business seems viable and needs to be protected from piecemeal sale of liquidation.
The NCLT has also been given the power to direct the company administrator to take over the
management or assets by the company administrator, Suo Motu, or on an application made by 75% of
the secured creditors in value (or 75% of all the creditors by value if there is no secured debt in the
company).
5. Recommendation on individual solvency

Effect of recommendation if implemented-


Positive effects
1. Improve the rank of India on ease of
doing business.. This law will encourage people to take up entrepreneurship, will
improve ease of doing business, improve recovery rates, and reduce Non Performing
and Bad Assets. Thus, it can lead to healthier bank and creditor environment.
2. Easy exit policy is one of the criteria considered by entrepreneurs before setting any organization
3. Early intervention by the government will save the organization from liquidation

Negative effects-

Various provisions which deal with the early intervention by the government to save organization
from being defaulted are already available under various laws, ineffective implementation of
existing laws are major problems
Many small businesses and micro enterprises are managed by people who do not have much
knowledge about laws and are largely illiterate, thus, without required knowledge of existing laws,
proper implementation seems to be a distant dream.\
In lieu of getting subsidies and tax benefits through state intervention, entrepreneurs may show a
well running organization as a sick organization.

Thus, Bankruptcy law can go a long way in not only supplementing Make in India
campaign but also creating a lot of jobs in India. However, there can be
certain challenges which we have seen with SARFASEI law and Asset Restructuring
Companies. There will be possibility of hands in glove situation to cheat the
creditors and Government. Hence, this law should also take care of such
scenarios by provisioning stringent punishments for frauds.

5) What is the importance of Great Compression in American history? Do you think India too can
experience Great Compression? If so, suggest what policy measures need to be implemented by
the government.

Great Compression refers to the period during which income inequality declined drastically with
rich losing their share of total wealth and income and the working class gaining unprecedented
growth in their income .The period overlapped with the years of the Great Depression of 1930s in
America and was at its peak during 1930 to mid 1940 .It saw huge growth and evolution of middle
class population in America with rising income levels .It lasted till the year 1970 and ever since it
has reversed. During this time, the riches were forced to pay greater taxes in order to raise the
government revenues which could be utilized to boost spending particularly on world war II and
welfare programs and economic activities that could create employment during the crucial time of
economic slowdown and global war .There was the net transfer of income from the wealthy class
to the working class with wages paid to them being high resulting in less profits to corporations.
GC was the used as US govt. policy to narrow the Gini index [ reduce the income inequalities,
wealth concentration] ,which included measures viz unionization, legislation for minimum wages
,Overall the view was to take the slice of income from best paid and giving it to the worst Paid.
Indeed this Economic policy was way ahead for various other countries. This has caused
emancipation of the weaker classes and a condition of the full employment, a dream for every
economy. It also had helped to part the gorge between poor and rich.

In context to India which recently in its Economic survey 2015 has theme creating opportunities
[of economic growth] and reducing vulnerabilities [economic inequalities] has similar stance,
though in the milder format. In India economic inequalities are relatively high as shown in the Gini
Index which is 33.4% [mean 33% people have greater control over resources]

It is less likely that India is going to experience something like the Great Compression in the
foreseeable future because,

*There is no crisis like war,

*corporate tax is being lowered from 30% to 25% in next four years to attract investments leaving
more cash in hand for investment to kick start growth ,

*stable tax regime is being brought to make business environment more corporate friendly.

* GAAR on hold for 2 years and abandon of DTC.

*Royalty tax has been reduced .So; there is no mark of any increase in tax burden on the riches.
Tax on royalty and fees for technical services brought
down from 25% to 19% to allow small and mid-sized firms to achieve technology
from abroad

And hence, no reduction in their share of wealth at all. In fact, the opposite is true. There is a
huge income inequality in India with widening wealth gap. Wealth concentrated in the hands of a
few while most of the population either being in the lower middle class groups or below poverty
line.

*Make in India initiative to be more indigenous to promote goods and reduce the imports.

Indias policies always take care of the global aspects and according to that it tries to function so
that no external shocks disturb the common people, the most.

The prospects of such an experiment in India to be successful are bleak given the size and
population of the country and the disparities and social inequalities on not only the economic front
but on the skills , education as well. Having said that even we India tries for any such expedition
then few must to do things will be :-
1) We have to impart certain level of skills to every aspiring worker and job seeker, to be better
placed.
2) Necessary elementary education needs to be given along with the skilling programmes
under the National Skill Development Council such as Internship in a firm side by side while
studying.
3) There will be a huge challenge to normalize the high paid corporate jobs especially in the
foreign MNCs to make them employ more people and having a cut on the pay check of big
earners to balance the balance sheets.

6) the most recent union budget and the economic survey have pointed out that many big
infrastructure projects under PPP model are stalled thanks to variety of reasons. They
suggest revisiting and revitalizing PPP model to complete stalled and new projects in a
time-bound manner. In this regard, what amendments have been proposed to PPP model?
What challenges these amendments would face? Examine.

Recent ES 2015 and Union Budget observed various points, regarding the bottlenecks to growth
such as stalled projects and Balance sheet syndrome of Indian characteristics. It said stalled
projects both public and private sector had different and variety reasons. for Private it was
unfavorable market condition ,weak and slack demand from govt. as well as market while for
public sector project were stalled due to clearances, Chaos etc . It said that both caused the
Credit restriction on future private investment through banks, with parallel NPA issue.

Some other has suggested that EPC model on investment to hasten the growth is a viable option
where majority risk is with public while private is concerned with Design and implementation. This
was already with the govt. till late 90, and had some other issue as well

However it is felt that PPP projects in time bound manner are important for economic growth and
infrastructure.

Some of the provisions to revitalize the sick PPP model are:-

Increasing the capital expenditure of the public sector units (PSUs) in order to utilize funds from
these state owned companies to develop infrastructure. It emphasizes on big public sector led
push in railways and modification in existing design of PPP projects. A major push of public
investment will be seen. For this purpose, the fiscal deficit target date of 3 percent is postponed to
2017-18.
Move to take risks away from the private players in PPP model projects to public sector so as to
allay their apprehensions and risks involving escalating costs due to various reasons like delays,
land acquisition, clearances and returns on investments.
It also suggest a high-powered Independent Renegotiation Committee for deals gone sour, and a
better clean-up policy in the form of bankruptcy laws and guidelines for asset restructuring.
On an encouraging note, to reduce the stress of NPAs on banks and thus a possible legal hassle
for players involved in long gestation infrastructure projects, government proposed to institute a
National Investment and Infrastructure Fund (NIIF) with an annual inflow of Rs 20,000 crore to
finance infrastructure projects.
It endorses combining construction and maintenance responsibilities to incentivize building
quality, and recommends that states be allowed to experiment with concession agreements in
which the private parties would be allowed to operate a specific business under governments
jurisdiction subject to certain conditions.
Increasing capacity to respond to economic volatility. A promise on new Bankruptcy law and
aiding private sectors in acquiring lands, getting various clearances swiftly and eliminating other
obstacles for early recognition of stress to refinance them and also enabling an early exit for
private players in case of unviability, will increase the private sector confidence to partner in PPP
projects for infrastructure.
Use of Capital which until now parked in the banks or used as filling fiscal deficit, such shall be
used in capital investment, overseas acquisition. Given China slowing economy, lower Import bill
for Oil companies this is opportunity for use which will bear fruits as dividends for Domestic
infrastructure later.

But there are some challenges in implementing these amendments. These are:
1. transferring risks to the public sector involves exercising great cautions. Under current
scenario, PSUs are not in good shape and huge loss and dependent heavily on government for
capital infusion.
2. Fiscal condition is presently vulnerable to instability giving tough time to the government to
consolidate the fiscal space.
3.Vagueness and lack of enough experience in PPP as a model for infrastructure development as
it is quite of model of recent time implemented in India a few years ago.
4. Experience with states containing all the risks tells a dismal story .It is marred with delays,
inefficiencies, rigorous regulations and politics. In the past money was spent with political
motivations in fixed areas like for railway in VIP constituencies rather than expanding congested
rail corridors with maximum freight and passenger needs.

In sum it is using both Capabilities that are Efficiency of private and framework of Public sector in which
economic growth and infra can foster. Implementing an efficient PPP model requires building up capacity
to accurately specify service quality in the contract, to distinguish between viable and unviable bids, and
to respond to economic shocks or unanticipated demand shortfall.
This will happen from learnings after doing some of the projects

7) Critically examine what effect the ban on cow slaughtering by some of the states in India will
have on economy and society of India.

Cow has been revered since Vedic times and called aghanya, not to be killed. Also it is considered as s
symbol of prosperity and wealth. DPSPs ask the State to prohibit cow slaughter and same was echoed by
Gandhiji as well. The legislative competence in this matter is with the states and so based upon their laws
we have different categories of states like those where no cattle slaughter is allowed and those where
allowed while some not allow only cows.
The ban on slaughter of cattle has various implications on the economy:

(1) India is worlds largest beef exporter (20% share of the global market and exports worth more than
$4bn (2.6bn) a year). Cow meat export is strictly prohibited. India earns considerable foreign exchange
and still the markets of Russia and China are untapped. Ban on slaughter will harm exports and render
huge unemployment. The meat processing sector employs people from all faith and mostly downtrodden
semi-skilled laborers.
(2)Being the cheapest meat available, it will push up the price of other meat adding slightly too high food
inflation.

(3)Ban on slaughter disincentivises the farmer to rear cattle as he cannot sell cattle after its productive
period has ended. This affects our White revolution also.

(4)More undesirable unproductive cattle mean huge pressure on fodder for productive cattle and already
we are facing fodder shortage. It will affect healthy animals as they will not get enough food and it will
have to be shared

(5)Banning slaughter will affect associated industries like leather, pharmaceuticals and medicines

(6)Economic disparity will come in many communities as few communities are totally dependent on this
business. Disparity will come in meat sellers of cow and other meats like chicken, as the demand of
chicken will increase and price will go up. Due to demand for meat, black marketing and underground
slaughterhouses will be encouraged.

Impacts on the Society:

(1) Beef is poor mans source of protein. How will they compensate?
(2) Cattle slaughter in India is a historically taboo subject, because of the cows traditional status as a
sacred animal in Hinduism, the largest religion in the country. The new rule is expected to hit
some communities hard, some of whom own a majority of the beef-related businesses.
(3) Undemocratic governments crude attempt to regulate, dictate and censor the dietary habits
of Indians. Banning cow slaughter involves preventing people from choosing what they eat.
Permitting it would not impose a particular diet on an individual or group. Might hurt the sentiment
of minority.

(4) Abstinence from beef-eating is largely a caste or class question among Hindus. The low castes
prefer beef to other meat for reasons of taste and habit too.

(5) More important, surveys of butchers in different states show that three-fourths of all beef is
consumed by non-Muslims. A higher proportion of the sellers of cattle are Hindus. It will create
differences among different communities
(6) communal violence may get reduced in areas where Hindu were having problem with cow
slaughtering
(7) Banning slaughter will address pollution caused due to abattoirs which is a big concern. Though
APEDA guidelines exist but proper treatment of waste is often ignored

Cultural norms shall be respected, However Government shall take pragmatic view given the fact Milk
has ability to reduce malnutrition, supports various sub urban dairy activities and cooperative
business. Any decision based on Emotional Aspect may deter the Health of Citizen and economy so
in my opinion we should move towards a system that culls unproductive cattle together and this
ensures co-existence of mutually beneficial White and Pink revolutions. Also strict policy in terms of
licensing abattoirs and proper waste treatment need to be done.

Topic: marketing of agricultural produce and issues and related constraints


8) There are lots of criticisms against the Agricultural Produce Market Committee (APMC) Act,
even on the Model APMC act. Critically examine what these criticisms are. Also examine why
it is perceived to be difficult to reform APMCs.

Intro:-

Govt. policies in India related to agriculture affected mainly from three points of triangle namely food
security, remunerative prices to farmers and fair prices to consumers.

-Being a state subject, agricultural matters are controlled by state governments.


-APMC acts in various states since 1950 to bring transparency and eliminate discretion of traders in
agricultural commodities trade.

APMC acts divided state in geographical areas headed by market committees and any production in that
area shall be brought to that APMC market for sale. Pertinent to note that APMC is applied on notified
agricultural products which vary from state to state.

The APMC have been a laggard in implementation of a free market. The main criticisms associated with
the APMC are:-

-The APMC Levis multiple fees (like market fee of buyers, license fees, Mandi fees, VAT etc...) of
substantial magnitude which would add as an additional burden on the farmers. For e.g. fees to the tune
of 10 - 15% are levied by some states like AP, Punjab and Orissa. This results in a cascading effect on
the price of the commodity especially when introduced at the very first level.
-Recently, govt. studies note 18 types of taxes levied in various markets.
-The APMC has under it different kinds of markets which are not well integrated and no proper
infrastructure in place. Middlemen add to further delay and decrease efficiency.
-Most of the taxes collected are not reported to the legislature, so there is less accountability
as a result it could be more politically driven.
-There is no role of private players which would result in a less competitive environment.
-The lack of transparency in the law creates the space for corruption and impedes the creation of a
national market.
-In the agricultural markets regulated under APMC, there are APMC charges to be paid by the buyer,
licensing charge on the commissioning agents who mediate between the buyer and farmer, and host of
other charges from related parties. Such high taxation, at the first-level purchase of the commodity, has a
cascading effect on the prices.
-Further, these rates are different in each state, and the APMC charge is collected as a state tax, rather
than as a charge for the market services rendered. This is a major impediment in the development of a
national market for agricultural produce.
-Monopoly of APMC restrains farmers to sell their produce directly to end-consumers.
-Cartelization of traders.
-Traders dont feel comfortable with the entrance of new buyers, therefore stringent conditions like owning
of a shop requirement, high license fees and other difficult norms has implemented.
-APMC play dual role of regulator and market. Hence, conflict of interest always occurs.

Model APMC act:-


Recognizing such negatives, Central govt. has recommended model APMC which freed farmers to sell
their crops at their discretion.
-This Act also increases the competitiveness by allowing for a common registration of market
intermediaries.
-Some states implemented it through establishment kisan markets, also known as Apni Mandi, rythu
bazaar, haats etc.
-Other provisions of new act are- no delay in payments to farmers, quality control mandatory
infrastructure and e-information about prices and transactions.
With implementation of part of the policies of the act, there has been some improvement which is evident
from the implantation of it in Karnataka with a single Licensing system.

Politically powerful lobbies hold over APMC committees and older acts good revenue generation
capacity for state governments holds back respective governments to approve and implement new act.

The model APMC act, designed to help the states reform their APMC acts, also has its own set of flaws.
Flaws in Model APMC:-
-It retains the APMC charge as a state tax.
-Permission by some states for high license fees.
-While it allows for private participation, it mandates that private players should also collect the APMC
charge, on behalf of the govt. in addition to their charges for providing warehouses, loading, etc. Private
players are subjected to Mandi tax and cess.
- The model APMC Act retains the mandatory requirement of the buyers having to pay APMC charges
even when the produce is sold directly outside the APMC area.
-These provisions still doesnt even provide for creation of a national level or state level common market
-Even though the model APMC act was adopted by most states, they have stalled the implementation,
suggesting opposition from the political agents.

Agricultural and internal trade being state subjects, reform in the APMC act at a national level is difficult to
achieve. Very few states like Karnataka have improved the act using modern technology.
-While the door is open to impose Lawson the state through Concurrent subjects that can control trade in
most of the commodities, this would be major step back from the present policy of cooperative federalism.
-The problem of middlemen, private players, APMC committee members having nexus with politicians
and bureaucrats is the prime reason for losses to farmers. Lobbying by middlemen, corruption, lack of
funds etc are reasons which are making the reforms implementation difficult one.

Conclusion:-
Transparency in prices, MSPs, Election of committee members are needed for better marketing system.
Political will is needed for proper implementation of reforms.
-In such scenario, it is needed to introduce PPP model for infrastructure generation for sorting, grading
and quality improvement similar to ministry of food processing projects. Contract farming, cooperative
warehousing and export zones are other areas which should be looked in relation with APMC acts to
make Indian agriculture an honorable profession.
-The 2014 budget recognizes the need for setting up a national market and stated that the central
government will work closely with the state governments to reorient their respective APMC Acts to provide
for the establishment of private market yards/private markets.
The Economic Survey 2015, Volume 1, Chapter 8
Topic: Environmental pollution and degradation; resource mobilization

8)What do you understand by Carbon Tax? It is said that India has imposed the Carbon Tax
implicitly on its citizens. Examine how and in what form.

Intro:-

A carbon tax is usually defined as a tax based on greenhouse gas emissions (GHG) generated from
burning fuels. It puts a price on each tonne of GHG emitted, sending a price signal that will, over time,
elicit a powerful market response across the entire economy, resulting in reduced emissions.
- levied on the carbon content of fuels, a form of carbon pricing.
- Carbon taxes offer a potentially cost-effective means of reducing greenhouse gas emissions.
- From an economic perspective, carbon taxes are a type of Pigovian tax.
- They help to address the problem of emitters of greenhouse gases not facing the full (social) costs of
their actions.
- Carbon taxes can be a regressive tax, in that they may directly or indirectly affect low-income groups
disproportionately. The regressive impact of carbon taxes could be addressed by using tax revenues to
favor low-income groups.

How it is imposed implicitly and in what form:-


1) On July 1, 2010 India introduced a nationwide carbon tax of 50 rupees per metric tonne of coal both
produced and imported into India (Coal Cess). In a budget speech on 2015, the finance Minister
increased the Price to 200 rupees per metric tonne.
-In India coal is used to power more than half of the country's electricity generation.
-The clean energy tax will help to finance a National Clean Energy Fund (NCEF)
2) Earlier government deregulated of prices (eliminating subsidy) of diesel using the decline in its prices.
3) Simultaneously there is a rise in excise duty and cess for diesel and petrol as an implicit carbon tax.
This will serve as a carbon tax and also to price other externalities (like congestion, noise and local air
pollution) associated with burning of petrol and diesel. This also helps in raising substantial revenues for
social redistribution.
-Move from subsidization to taxation of fossil fuels, related to revenue and macro-economic consideration
but they are also consequential in their climate change impact.
-In the light of recent falling global coal prices and contribution of coal to both local and global pollution,
there may be room for further rationalization of coal pricing.
-Any rationalization of coal pricing must take account of the implications for power prices and hence
access to energy for the poorest in India which is a fundamental objective of policy.
-This, in addition to providing access and empowering people through renewable sources of energy, an
area of high priority for the Government of India. This will be especially important for serving remote areas
with limited access to grid.

Conclusion:-
On the whole, the move to substantial carbon taxation combined with Indias ambitious solar power
program and renewable energy targets suggest that India can make a substantial contribution to the
forthcoming Paris negotiations on Climate Change.
The Economic Survey 2015, Volume 1, Chapter 9

Topic: Indian Economy issues


9) Why many economists advocate increasing the use of plastic money in India? Examine its
advantages and disadvantages.

Intro:-

Technology has changed the way we deal with money giving us more convenience and easy access to
funds from anywhere. Economists have been advocating the use of plastic money (credit cards, debit
cards, ATM /cash cards etc) due to their feasibility and multiplicity of advantages.
-Economists advocate the use of plastic money against paper money, due to its qualities of durability,
safety, security, cost effectiveness and environment friendly nature, which are missing in paper money,
which suffers of being soaked, burnt and the inherent cost of paper, which requires deforestation.

Advantages of Plastic Money:-


- Eliminates the need for carrying huge cash
- Risk of Loss or Theft minimized
- Anytime/Anywhere Access Using cards
-Credit Facility for emergencies
- Online Payments can be done easily
- Secures data regarding transactions like sender and receiver and so no anonymity like in cash
transaction. This helps in tracing origin and can be used against Hawala cases, black money.
- The risk of clerical error is reduced and there is an electronic record of transactions being carried out for
customer ease.
- Great option for financial inclusion for the customers that are geographically far from convenient
physical banking.
- Curbs the fake currency circulation in markets.
However, in the Indian case, we have more than these reasons for promoting plastic money
-can allow the poor direct access to their money transferred via DBT/subsidy transfers
-will help in checking black money by establishing identity of the customer and data of transactions
-avoiding tax evasion
-will save cost of printing and environmental cost of deforestation

Some of the drawbacks or risks related:-


-Non-Acceptance at Small Retail Outlet hence cannot be used for all daily needs
-Loss & Misuse
-Low Value Transactions not always feasible
-Service Charges are applicable in many cases
-Impulsive Purchases and promotion to consumerism
-Cyber-crimes, phishing etc.
-The Indian scene hasnt embraced the credit-card debt scenario to this extent as customers still exercise
frugality and sensibility in buying. However, the trend of EMI payments for example is on the rise as
customers that require certain products would rather be in interest debt than wait. This changing trend in
customer preferences could spell trouble for people without financial planning.
-With time, the magnetic strip of the card gets worn out and the stop may stop working at some crucial
time.
-Indian scene not yet mature enough for acceptance as small retailers do not have connects devices and
customers prefer carrying cash.

Conclusion:-
For India, plastic money will help in curbing the menace of anonymous transactions, black money flow in
cash, keep vigil over all transactions like drug trafficking and terrorist financing, bringing more activities
under the tax net and so increase revenue collection and move towards possible tax reforms.
-Also domestic payment gateway RuPay will get a boost with more usage of cards and we will move
towards financial inclusion.
-Introduction of polymer notes can help curb counterfeit currency menace like in Australia, Canada, etc.
-However, currently India has a high ratio of cash in circulation to GDP unlike the developed nations
where this ratio is small. A lot needs to be done in this direction.
Though they cannot completely replace cash in the current scenario, they can be used as complementary
to cash. Moreover with their faster penetration, many linked advantages can be extended with plastic
money.
Livemint

Topic: Security challenges and their management in border areas

10) In your opinion, how has India fared in countering infiltration and cross-border terror?
Critically comment.

Intro:-

India's success in countering infiltration is visible by the fact that anti-social elements still trying to enter
the country from Pakistan and are still failing.
-Countering infiltration is as important as securing borders or territory. Cross border terrorism has risen
especially when there is globalized world where economic interdependence is more pronounced.
-Indian response over the years to handle these problems is defensive, with success in the form of
reduced cross border activities.

Causes for infiltration and cross border terrorist activities:-

Owing to the porosity of its long international borders - both land and maritime, India has witnessed cross
border terror and infiltration across the boundaries of Pakistan, Bangladesh, China, and Nepal over the
years.
-This can be attributed to ongoing border disputes with these countries, absence of fool proof physical
fencing across the borders, difficult mountainous terrain that makes manual surveillance difficult,
increased support provided by the locals to such infiltrates and terror groups.
-The Parliamentary attacks, 26/11 Mumbai terror attacks, the recent Burdwan Blast and Gurdaspur attack
(Punjab).

Efforts to counter:-

India's counter infiltration measures can be highlighted by the examples of patrolling army at Siachen
glacier in adverse conditions, Camel troops in Rajasthan in scorching heat, along LoC with high altitudes.
-Fencing & flood lighting on LoC and international borders.
- More no. of border posts; more manpower to man such posts.
- enhanced use of land mines all along borders identified as too porous.
- Armed security agencies with special powers such as AFSPA to deal with ground situation.
- developing intelligence networks in border villages.
- Technological equipment like radars
- CCTV camera's, fencing, Use of 'Drone' tech, Intelligence network should develop, Better coordination
between various forces should be promoted.
- Maritime infiltration has been curbed, good job by coast guard (Action against armed loaded boat in
Arabian Sea)
- Use of village locals for intelligence activities in affected areas and
- making efforts for early resolution of border disputes as seen in the case of Land Border Agreement with
Bangladesh have certainly paved positive results.

However, with the withdrawal of US troops from Afghanistan and emerging threat of ISIS, India needs to
upgrade its efforts to contain such threats by using technology like pressure sensors, hand held thermal
imagers and equipment to detect any breach of wire should be used along with complete fencing of
borders, clear demarcation of its boundaries, satellite surveillance particularly in risky terrains should be
made possible along with increased confidence building measures amongst the localities to aid
intelligence operations, combined efforts as seen in the case of Operation All Clear with Bhutan should be
made possible with neighboring countries.

Other steps needed:-

-Fencing along international boarder


- cease fire agreement along boarder which was partial success for some time etc.
- resume peace talks with Pakistan
- Settle land boarder agreements with china without delay (recently signed with Bangladesh)
- Increase in use of drones, information technology, space technology etc.
- develop counter intelligence networks
- Diplomatically keep talks open; drive home seriousness of issue on regional forums.
- Improving infrastructure to reach the harsh geographic locations along borders is also needed for
effective counter terrorism.

Conclusion:-

The paradigm for combating terrorism now involves the application of all elements of national power and
influence. Not only it is to employ military power, use diplomatic, financial, intelligence, and law
enforcement activities to protect the Homeland and extend defenses, disrupt terrorist operations, and
deprive enemies of what they need to operate and survive.
-Thus, with a political consensus formulate a clear-cut policy, with clear-cut statement of objectives,
defining the strategy to achieve the objective to end terror
The Hindu

Topic: Awareness in the fields of IT, computers and issues relating to intellectual property rights.

11) The US Federal Communications Commission (FCC) recently decreed that the internet is a
public utility just like natural gas, or water where charges arent distinguished by end-use.
Critically examine the implications of this decree for end-users and telecom companies. (200
Words)

Introduction:

Advent of internet has drastically transformed the way world communicates, accesses information and
conducts business. Remotest parts of world are connected and smallest piece of information accessible
at the touch of finger. Small entrepreneurs are riding on the services of internet to launch new startups
and businesses due to provision of internet that is neutral to all the end users and these businesses at the
same time

The FCCs decree is in favor of net neutrality which ensures a level playing field in terms of access and
speed to all content on the internet.

Implications on end-users:-

This has positive implications for end users as they can continue to access free applications like
Whatsapp, Facebook, and Youtube etc with no additional charges for these apps.

It should definitely be the sole right of the user to choose from the existing choices to what to surf and
where to spend most of its data that he legitimately has obtained after opting for the service providers.

Net Neutrality will ensure that the user is protected from any kind of data and bandwidth tinkering by the
service providers to the data flowing through their networks.

Implications on ISPs:-

The main apprehension and point of concern for the ISPs is that they spend huge sums for spectrums
and in return from the end users they are only getting a fixed sum, whereas sites like Facebook, Skype,
and Twitter etc are benefitting the most.

With the advent of IMs like WhatsApp and VoIps like Skype , they have eaten into their Messaging and
video calling revenue. Telecom companies argue that these application providers ride free on their
networks denting their SMS or call revenues. The implications of net neutrality on the telecom companies
would be to let go of an additional way to monetize the access they can provide by charging app
providers for access.

The skepticism and concerns of the ISPs are not fully valid as they still get benefitted from the data usage
by the end-user while surfing the above mentioned websites , tough indirectly.

However, in the absence of any net neutrality legislation in India, telecom companies have devised ways
to go around it. Reliance communication and Airtel Zero have tied up with partner apps to give
preferential access to their website or application and in turn generate revenue from them by charging a
data usage fee to be paid by the app provider. While these are free to download they may restrain the
end users first choice of preference and free access thus violating the concept of net neutrality

Topic: Conservation; Development

12) In your opinion, how should government balance its development imperative and its
responsibility to conserve environment? Critically comment. (200 Words)
Introduction:

As Gandhiji said, We have to make balance between the Need and Greed. Development and
Environment are the two side of the same
coin , Balance is necessary.

Green growth aiming to achieve a harmony between economic growth and environmental sustainability is
just what the world needs to obtain long-term and all rounded human development and maximize the
synergies between economic development and environmental protection.

Steps:

New mines shouldnt be allocated for excavation unless older mines are fully used.

River diversion, Land filling should be done after careful impact assessment.

Poaching should be locally regulated because often the local jobless people engage in poaching
activities as is clear in the case of Red Sander smuggler. Providing employment to local people in other
areas may prevent it.

Usage of plastic and harmful poisons/insecticide should be regulated in forest adjoining areas. As in the
case where more than 50 peacocks/blackbuck were found dead alongside field eating the poisoned bait
for pests.

Digging borewell alongside forest should also be regulated, because when groundwater goes down,
lakes/ponds in forest also go dry making wild animal to venture in villages for water which leads to man-
animal conflict.

Having stringent clearances regarding infrastructure projects, especially near ecologically sensitive areas
like national parks.

If linear projects already cut through national


parks, the government should strictly impose and reduce vehicular traffic
during the night hours, in order to prevent road kill.

Sign board at short interval should be put. Usage of high fence at places is also effective in the areas of
high rate of animal crossing on roads.

It can adopt best practices from countries like the Netherlands, which has built a series of camouflaged
bridges over the highways as wildlife crossings.

Integrated planning of the industry and Cooperation :- Number of power plants located in the region like
the UP on bank of Yamuna , Vidrabha etc. which lead to High level of the pollutant.

Need to set the priority to Use the Natural resources :- Today there are number of the water Parks in the
vidrabha where Farmer do suicides due to lack of water. Similarly Dam projects develop for the hydro
power while the Ecological balance of the Downstream alters.

Promote the Sustainable Development :- By Exporting Iron Ore we can earn the forex but it not
sustainable as we have to spend many more rupee behind the health care of the people living that region.
Similarly the deep sea fishing helps to earn the Forex but disturb the ecology.
Industry and Environment: - There shall be norms that Industry has to spend the 3% of her profit for the
Social forestry.

Impact assessment :- Need to carry the Survey of not only environmental assessment but also social and
Health assessment before the establishing industry and take the parallel and Preventative steps.

Environmental impact assessment of proposed development activity, economic balance should be made
before proceeding further

Think of available alternatives and evaluate them environmentally and economically and former should be
given preference in case of conflict

Strengthen NGT,NWB etc.. and consider their suggestions in the decision making.

Topic: changes in industrial policy and their effects on industrial growth.

13) Critically analyze what have been the major objectives and achievements of Industrial Policies
since 1991 in India.

INTRODUCTION

The New Industrial Policy,1991 seeks to liberate the industry from the shackles of licensing system
Drastically reduce the role of public sector and encourage foreign participation in Indias industrial
development.

MAJOR OBJECTIVE OF INDUSTRIAL POLICIES SINCE 1991

The broad objectives of New Industrial Policy are as follows:

1. Liberalizing the industry from the regulatory devices such as licenses and controls.

2. Enhancing support to the small scale sector.

3. Increasing competitiveness of industries for the benefit of the common man.

4. Ensuring running of public enterprises on business lines and thus cutting their losses.

5. Providing more incentives for industrialization of the backward areas, and

6. Ensuring rapid industrial development in a competitive environment.

ACHIEVEMENTS OF INDUSTRIAL POLICIES SINCE 1991

The New Industrial Policy 1991 aims to unshackle Indians industrial economy from the cobwebs
of unnecessary bureaucratic control.
According to this policy the rate of the government should change from that of only exercising
control over industries to that of helping it to grow rapidly by cutting down delays.
Removal of entry barriers and bringing about transparency in procedures. This policy therefore
also at virtually ending the License-Permit Raj which has hampered private initiative and
industrial development.
The new policy therefore throws almost the entire field of industry wide upon for the private
sector.

POSITIVE IMPACT OF NIP 1991:

1. Increase in production

2. Removal of bureaucratic hurdles

3. Increase in competition

4. Increase in efficiency of public sector.

5. Increase in foreign investment

6. Increase in exports

7. Balanced regional development

8. Less economic burden of government

NEGATIVE IMPACT OF NIP 1991:

1. concentration of economic power


2. increase in unemployment
3. jobless growth
4. ignore social objectives
5. Distortion in production structure: growth of capital goods industries declined.
6. adverse effect of small scale industries
7. .increase in regional imbalances
8. personalized relationship and corrupt practices still continue

CONCLUSION

Best outcome of the NIP 1991 was acceleration in the growth of GDP, which was under 4.0per cent
during 1951-52, increased to 7.7 percent during 2000-10.

Topic: Employment and related issues


14) Which elements constitutes Labor Reforms in India? Critically comment on the laws and
institutions that need to be reformed to bring meaningful Labor Reforms in India.

INTRODUCTION

Labour policy reforms in India are due for a long time, as the context in which they were framed
has changed drastically. Most of the 10-12 million youth joining labour force every year are forced
to join informal economy, where the working conditions are pathetic and earnings are also
abysmal.
The Labour scenario in India is far from satisfactory. Unless the systemic defects are addressed,
there is no long term, sustainable solution in sight.

ELEMENTS CONSTITUTES LABOUR REFORMS IN INDIA

Labour reforms include providing job security to the nations labour population along with financial
independence which can enable them to raise their standard of living
allowing the labourers a share in the decision making authorities of the companies
Ensuring equality of pay for men and women and safeguarding the rights of migrant workers and
other vulnerable groups.
Labour contributed to the increasing productivity of the industry in the long run.
Increase the percentage of workers in organised sector.

LAWS AND INSTITUTIONS THAT NEED TO BE REFORMED

1. Labour should be shifted from concurrent to state list, so that each state can better cater to their
labourers needs. This can be done from the increased quota of funds to be allocated to states
now.
2. The numerous laws complicate their usage. So, they should be consolidated category wise,
regarding provision of insurance, wage revision, dispute redressal mechanism, etc.
3. The Make in India initiative should also be able to diversify the production capacities, and
simultaneously provide job security to workers.
4. The maintenance of records of workers in the companies should be computerized, to avoid
tempering later and easy access.
5. There are plethora of laws governing labour today from the Industrial Disputes act, Factories
act, Contract Labour Act (1970), Trade Union Acts and other legislation. Some of there were
designed at a moment where they were meant to protect labour but some of them have become
obsolete and require revision according to the changes circumstances.

CONCLUSION

By effectively integrating the labour force of the country in the mainstream of the nations development,
govt will be able to gain their trust and also stop the unrest in the manufacturing sector, increasing its
productivity simultaneously.
Topic: issues relating to intellectual property rights.

15) Critically examine the laws and institutions governing the Intellectual Property Rights regime
in India. Also examine what are the objectives of Indias IPR regime.

INTRODUCTION

Intellectual property rights as a collective term includes the following independent IP


rights which can be collectively used for protecting different aspects of an inventive work for
multiple protection

LAWS AND INSTITUTIONS GOVERNING THE INTELLECTUAL PROPERTY RIGHTS

1. Patent - the first Indian Patent Law came in 1856. It was further amended in 2000, 2003 and
2005 in compliance with the TRIPS provision. India is a member of the Paris Convention, Patent
Cooperation Treaty, and Budapest Treaty and recently India signed the "Madrid Protocol".
Ministry administering the IPR-

a. Department of Industrial Policy and Promotion

b.Ministry of Commerce and Industry

2. Copyrights - The Copyright Act, 1957, as recently amended in 1994 and 1999, governs the
copyright protection in India. The total term of protection for literary work is the author's life plus
60 years. Ministry administering the IPR-

a. Ministry of Human Resource Development

3. Geographical Indications (GI) - it is the exclusionary rights for the indicator that identify the
goods originated within the member nation territories, or area or region of that territory, where the
reputation or other attributes of the goods is essentially related to the geographic origin of the
place. In India, the GIs regime is regulated by the Geographical Indications of Goods
(Registration and Protection) Act, 1999 and the Geographical Indication of Goods (Regulation
and Protection) Rules, 2002.eg: Basmati Rice, Kanjeepuram sarees and Darjeeling tea.
Ministry administering the IPR-

a. .Department of Industrial Policy and Promotion


b. Ministry of Commerce and Industry

OBJECTIVES OF INDIAS IPR REGIME

The objectives of India IPR's regime is:

Efficient processing of IP applications by inducting additional manpower, augment IT facilities and


automation in Intellectual Property Offices.
Strengthening public delivery of IP services.
Highest levels of transparency and user-friendliness.
Government trying to promote the awareness of IPR in such remote areas.

Topic: Various Security forces and agencies and their mandate

16) Write a critical note on the structure and mandate of following enforcement agencies in not
more than 100 Words each

ASSAM RIFLES

it was formed under the British in 1835 called Cachar Levy because the threat posed by the tribal
to British industries and plantations in Assam, so British decided to make separate body under
civil government.

The Assam Rifles is the oldest paramilitary force of India. now its under control Ministry of Home
Affairs

Assam Rifles and its units have served in a number of roles, conflicts and theatres including
World War I where they served in Europe and the Middle East, and World War II where they
served mainly in Burma.

they perform many roles including the provision of internal security under the control of the army
conduct of counter insurgency and border security operations; it has been guarding the India-
Myanmar. unit is under criticism because of the human rights violation in the Indian north-eastern
state of Manipur. under ( Armed Forces Special Powers Act)

NATIONAL SECURITY GUARD

The National Security Guard (NSG) was set up in 1984 as a Federal Contingency Deployment
Force to tackle all facets of terrorism in the country. It is specialized counter terrorism force,
intended for use only in exceptional situations.
the two components of NSG the Special Action Group (SAG), which consists entirely of Indian
Army personnel.
the Special Ranger Groups (SRG), which comprises personnel drawn from Central Armed Police
Forces and State Police Forces. The NSG is under the authority Ministry of Home Affairs.
major roles in operations:
1.1984 Operation Blue Star
2.1991: NSG employed along with SIT in search and strike missions after the assassination of
Rajiv Gandhi
3. Operation Ashwamedh,
4. 2008 Mumbai attacks Operation Black Tornado, response to criticism of the time taken by
NSG units to arrive in November 2008 Mumbai attacks from their base in Manesar. The
government should require deploying more number of NSG contingents in major cities across
India for quick response to emergency situation.

SASHASTRA SEEMA BAL

Sashastra Seema Bal is one of India's Central Armed Police Forces. It is often loosely referred to
as a paramilitary force; SSB is border guarding force Indo-Nepal and Indo-Bhutan borders. It
comes into existence under name of Special Service Bureau was set up in early 1963 in the wake
of the Sino-Indian War.
SSB is also engaged in Counter Insurgency operations in Jammu and Kashmir and Anti Naxal
operations in Jharkhand and Bihar. It is also performing internal security duties, Election duties
and law and order duties in different parts of India.
The primary task of the force was to provide armed support for RAW. And secondary task was to
inculcate feelings of national belonging in the border population, along with national awareness
and security consciousness, organize and prepare border population to resist enemy.
Nepal and Bhutan are totally different borders. As these borders are open and there is no
restriction of movement for people...Hence it is difficult to identify anti-social elements, more
technology, funds and more academy are required.

LINKS

http://www.insightsonindia.com/2015/04/10/insights-secure-2015-question-from-static-portion-9-10-april-
2015/

Topic: Effects of liberalization on the economy

17) Outcomes of the patterns of growth induced by neoliberal economic reforms have increased
the disproportionality between agricultural and non-agricultural growth, and with costs rising and
prices not keeping pace, agriculture is becoming increasingly unviable. Elaborate.

INTRODUCTION

Though share of agriculture to overall GDP in India has come down to 14 per cent, still 66.2 per cent of
rural males and 81.6 per cent of rural females are engaged in agriculture as cultivators or labourers.

Problems in agriculture even after economic reforms:

1. The farmers started growing those crops that fetch them a better price; problem is that price
depends on international agri-products. Currently international agricultural prices are below profit
level. So, exports have decreased and domestic markets sells dont provide enough revenue
which can counter bank loans on seeds and fertilizers.

2. There is no governmental framework, at state level, to guide farmers what to grow and in how
much quantity in relation to demand in domestic market and international markets.

3. Uneven seasons and crude prices is making inflation less predictable, this makes government
conservative while investing in agriculture and food industries.

4. Private companies least interest in growth of agriculture sector reasons:


(a) Less profit

(b) Less number of labourers at cheaper rate if agriculture becomes profitable

(c) No Government support through PPP model in agriculture industry

5. Lack of Marketing tools - National Commission on Agriculture, defined agriculture marketing


as a process which starts with the decision to produce a sale-able farm commodity and it involves
all aspects of market structure of system, both functional and economics considerations and
includes pre and post-harvest operations, assembling, grading, storage, transportation and
distribution.

6. contract farming

The main disadvantages faced by contract farming developers are:

land availability constraints;

social and cultural constraints;

farmer discontent;

extra-contractual marketing; and

input diversion.

7. Middle men: the system has created several layers of intermediaries, lengthening the supply
chain and increasing the opportunity for cartels to form, which in turn drive prices down for
farmers and up for consumers.

CONCLUSION

So, it is necessary to change methods of investing capital into agriculture sector. Domestic
growth of seeds, inputs and fertilizers will provide farmers more profits by which they can earn
more and compete with non-agricultural sectors.

Topic: Agriculture issues


18) These days, farmer suicides are reported even from prosperous regions of Punjab. Critically
examine the causes.

INTRODUCTION

Punjab is the granary of India. The green revolution made its maximum impact here and farming
became a very profitable business in the 70s and 80s. But that time is past now and even Punjab
reels under the pressure agricultural distress.
In 2012, the National Crime Records Bureau of India reported 13,754 farmer suicides. The
farmers suicide rate in India ranges from 1.4 to 1.8 per 100,000 total population, over a 10-year
period through 2005 Farmer suicides account for 11.2% of all suicides in India.

REASON FOR SUICIDE IN PUNJAB

1. raising Input costs; With inflation raising, the cost of input materials like seeds, mechanized
sowing and harvesting machineries, fertilizers etc has been increasing
2. Withdrawal of many input subsidies by the govt. These factors have led to increased cost of
production.
3. Fragmented lands and the over exploited lands have started to yield less crops per acre.
the over use of chemical fertilizers, salination of soils due to excessive irrigation further alleviate
the problems.
4. Problems of debt traps. In this Punjab region the agents called Artiyas who act as money
lenders are exploiting the small farmers into debt trap and consequently size their lands.
5. Role of agents in selling crops further reduce the gains of the farmers. There is no free
market mechanism and the so called Arthiyas demand huge commissions to sell the yield of
small farmers.
6. Involvement of Agri-companies who encourage farmers to grow cash-crops which beyond the
initial seasons do not sell for the high prices they were promised.
7. Punjab area reverse-tenancy is happening wherein; small farmers are renting their land to
bigger farmer and agro-companies because they cant do farming in profitable way. Many a
times, small and weak farmer lose their land to money lender or bigger farmers to whom they
rented. This makes them not to leave farming even when they are making losses.
8. Farmers often come into influence of agro-companies who are volatile in nature, for ex. 2
years back when Guar gum was in high demand; agro-companies reap benefits by growing it. But
when farmers in adjoining areas also followed the suite they made heavy losses. Agro-companies
stopped sowing it, based on international price fluctuations.

9. About 38 per cent of these persons were in the prime of their life (20 to 30 years old) while 60 per
cent had unpaid debts. In all, about 47 per cent of the 3,000 persons who committed suicide were
illiterate. Two districts of Punjab - Bathinda and Sangrur shows maximum trend .

WAY AHEAD

The solution to the problem lies in sustainable agriculture practices backed by integrated
watershed management which is cantered around rainwater harvesting.
Diverse crop combination, ensured irrigation facilities, insurance cover, access to formal credit
facilities, Procurement through MSP, backward and forward linkage are few important areas
among many others to work upon by govt., and supported by society, NGOs, activists and
everyone us.
loan restructuring in case of crop failure, agri insurance, Inflation adjusted MSP, soil health cards
and corresponding use of fertilizers, crop rotation, use of bio fertilizers, mitigation of soil erosion
etc
Hence, Farmer in such situation should diversify their income in cattle rearing, poultry, sowing
some portion of cash crops. This will make economically resilience.

Topic: changes in industrial policy and their effects on industrial growth.

19) Critically discuss major problems plaguing the micro, small and medium enterprises (MSMEs)
sector in India. Do you think governments policies to boost this sector adequately address these
problems? Examine.

INTRODUCTION

Micro, small and medium enterprises (MSMEs) play a pivotal role in the overall industrial
development of the country.
Inspite of this, the MSMEs continue to face several problems in their day-to-day operations, that
is, in production and marketing of their products.
Micro, Small and Medium enterprises give employment to huge populations and are around
5.2 crores in number. They are labour intensive sector and hence govt must do every bit to
provide financial and market access to them.
MSME sector accounts 45% of the manufacturing output and 40% of total exports of the
country.

MAJOR PROBLEM OF MSME SECTOR

1. High interest rates charged and collateral requirements by banks are a major issue when
MSMEs want to raise capital.
2. Equity capital also has only limited access as most investors do not risk investing in
unknown/small companies.
3. Market access for MSMEs is still very limited as due to lack of capital and various other
reasons they cannot tap lucrative export markets.
4. Revival mechanism for sick companies is virtually non-existent in the MSME sector,
therefore a company does not get a chance to reinvent itself and become profitable.
5. They are highly fragmented and no unions and organization speaking for their interest so
they often go unrepresented in many policies change.

6. They come under Priority sector lending along with other areas like Education, Export,
housing, Agriculture. Under RBI rule any domestic or private bank with more must lend 40% of
their lending funds to PSL list while foreign banks with less than 20 branches must offer 32% of
their funding amount to PSL areas. Hence most of the funds diverted to other areas as Education,
housing agriculture are more organized and often get waive off from govt.

7. Problems with government procurement due to requirement of high turnover and


experience requirement.
8. Lack of awareness about marketing and branding.
9. Lack of availability of skilled manpower.
10. Low technology levels and lack of access to modern technology.
11. Inefficient raw material procurement due to smaller scale and local procurement.

12. But more than 94% of MSMEs are unregistered, sub-optimal scale of operation, technological
obsolescence, supply chain inefficiencies, increasing domestic and global competition, fund
shortages, change in manufacturing strategies and turbulent and uncertain market scenario are
some roadblocks.

13. CGFT and SIDBI were set up to facilitate credit flow into this sector without the need for
collateral/third party guarantees. Despite this, banks continue to insist on the same.

GOVERNMENT POLICIES TO BOOST

1. In order to protect, support and promote small enterprises as also to help them become self-
supporting, a number of protective and promotional measures have been undertaken by the
Government.
2. The recently launched MUDRA initiative is not enough alone to uplift the MSMEs from their plight,
as MSMEs need tax reforms in the form of GST roll out as it will eliminate the cascading effect of
central and local taxes. They should also get a share of the Make in India
scheme, with their enterprises included actively in it.
3. The promotional measures cover industrial extension services, institutional support in respect of
credit facilities, provision of developed sites for construction of sheds and assistance for domestic
marketing as well as exports.
4. In general, there is need for tax provisions and laws that are not only labour-friendly but also
entrepreneur-friendly. More importantly, there is need for skill formation and continuous upgrade
both for labour and entrepreneurs through ITIs and management schools.
5. Other schemes like compulsory procurement from MSMEs have also been criticized for
being just paper tigers.
6. Many MFI started/supported by Govt takes 30-60 days to offer the loan which is long time as per
MSME working style.

WAY AHEAD

Govt should impose separate sub-limit of 7.5 per cent has been created within the priority sector
lending norms for micro enterprises.
Also, there should be little lower interest rates as compared to other loans offered for MSME. This
will give a kick start to MUDRA bank esteemed plan of financial inclusion.
Therefore, the sector is in dire need of reform. In addition to correcting abovementioned flaws
with its policies, the government could take steps such as compulsory procurement of material
by PSUs and government departments from MSMEs, setting up of R&D centres; ensuring access
to alternative sources of capital as private equity/venture capital and strengthening skill
development programmes.
By introducing all such reforms, the central govt will be able to help these unsung heroes of the
national development and promote their expansion.

Topic: Conservation
20) In a bid to reduce man-animal conflict in Assam, bio-fences are proposed to be set up
replacing electric fences, to ward off straying elephants. What are these bio-fences and how do
they work? Also suggest any other measures that can help prevent or reduce man animal
conflicts.

INTRODUCTION

Man encroaching forests in the name of development has led to growing instances of man-animal
conflict; often resulting in loss of human lives & precious wildlife.
To avoid wild animals from entering villages, fields and such other human settlements, the
previous stress on fencing or electric fencing has proved to be too hazardous for wildlife; animals
get entangled in barbed wires & eventually die or lose limbs;

WHAT ARE BIOFENCE

Bio-fences are fencing made by dense natural vegetation for warding off wild animals. It is
safe and sustainable. For ex. thorny bamboos when planted very close can grow to dense
layer of bamboos fence which any big animal cant cross.
It is safe because it will serve the purpose of warding off animal without harming them and
sustainable because it doesnt have maintenance cost like electrical fences which are hazardous,
high maintenance and useless when electricity is not there.

Hence, bio-fences which were used earlier in Brazil are now in trial period in Assams tea
plantation, they are growing bamboos sapling in nursery to plant it later across plantation.

HOW DO THEY WORK HUMAN ANIMAL CONFLICT

Involves planting dense thorny vegetation on the forest fringes to prevent wild animals from
entering human habitations and destroying crops.

Plant species such as "Agave Americana" commonly known as century plant and thorny
bamboos are typically used for bio-fencing.

OTHER MEASURES TO PREVENT

Making animal corridor reserved and not allowed for agriculture and forestry which is used
by animal to pass between forests. If its possible, fencing is better.
Making people around the forest area aware that, wild animal also fear from them and in any
case of man-animal conflict do not take situation in their hand, instead focus on Calling forest
official & saving themselves rather taking mob attack on animal.

Making forest official accountable for any death of animal in their area of supervision. Time to
control animals safely should be criteria of their promotion and bonus.

CONCLUSION

Animals are not rational being but humans are, hence we must make certain provision to save/ward-off
them rather than punishing them for being irrational and crossing their assigned territory.
Topic: Role of external state and non-state actors in creating challenges to internal security.

21) If the government is serious about bringing lasting peace in the North-East, the first step is to hold
ceasefire signatory groups accountable to ceasefire ground rules. Who are these ceasefire signatory
groups and what are the ground rules of ceasefire? Elaborate and comment on the statement. (200
Words)
The Hindu

Historical facts:

1. In 1917 and 1935 act, Nagaland was given slightly different status than remaining Indian
Territory.

2. Based on this, Nagas demanded their separation and insurgency began in 1947 with the
formation of Naga National Council (NNC).

3. Nagaland was given separate statehood in 1963.

4. Formation of NSCN revived the demand for independence.

5. NSCN was formed by coming together of several tribes, which soon led to formation of different
factions.

6. NSCN was divided into NSCN (IM) and NSCN (K).

7. NSCN (K): Khaplang faction: Khaplang belong to Myanmars tribe.

8. In 1997, ceasefire was signed with NSCN (IM). NSCN (K) was kept out of this.

9. In 2001, similar ceasefire agreement was signed with NSCN (K).

10. Cease-Fire Supervisory Board (CFSB): CFSB was formed with five representatives each of GOI
and NSCN and a Chairman to be nominated by the GOI,

a. Its function is to supervise ceasefire agreement.

Ground Rules-

1. The NSCN would not undertake offensive operations like ambush, raid, sniping and attack
leading to death/injury/damage or loss of property against anybody

2. There would be no offensive operations like ambush, raid and attack leading to
death/injury/damage or loss of property against the NSCN by the Indian Army, Paramilitary
Forces and the Police

3. NSCN will notify to the CFSB, the list of all their camps.

4. The CFSB would, after due scrutiny, finalise the list of the "designated camps" where all the
armed cadres of the NSCN would be located within three months.

5. The NSCN will refrain from extending any form of support or assistance to other militant groups.

6. The NSCN would refrain from blockade of roads and communications and from any activity which
would disturb the functioning of local and, State Governments and of economic or developmental
activities as well as essential services.
7. Protection of convoys and patrolling of roads would continue to be undertaken by the Indian
Army, Para Military Forces and Police.

8. During the course of the cease-fire, the NSCN will refrain from acquiring any additional
arms/ammunition military equipment.

9. The NSCN will refrain from forcible recruitment of armed cadres.

The Government of India reserves its right to continue operations against all other militant
groups who are not a party to the 'Cease-fire'

Ceasefire signatory groups: NSCN (IM), NSCN (K)

Development can only happen in the region if peace is prevalent. For peace to come there is need to
avoid violence and solve any issue via the mechanism established.

To hold true to ceasefire agreement, there is need to every ceasefire signatory group to
uphold ground rules of ceasefire.

Any grievance should be settled through the mechanism established by ceasefire agreement.

Importance of northeast for Act East policy of India in which Manipur and Nagaland were
envisaged as gateway for east.

Conclusion:

Development is the only way to ensure peace and prosperity.

InvestmentIndustriesJobsway to stop youths from joining such insurgent groups.

Increase connectivity in northeast region.

Ensure cooperation with Myanmar to solve insurgency problem.

Topic: Awareness in the fields of IT & Computers,

22) Recently an international newspaper reported that China is using a powerful internet tool known as
the Great Cannon to control web traffic flowing into its territory. Examine how this Great Cannon is
believed to function and its purported capabilities. (200 Words)

The New York Times

Background:

1. Openness is limited in Chinese internet.

2. China had very strict policy to prevent entry of outside traffic to Chinese websites.

3. Chinese Great Firewall: The firewall is used to block sites like Facebook and Twitter as well as
several media outlets.
Chinas stand on internet:

Chinas Lu Wei (internet czar) has called on internet community to respect Chinas internet
policy.

China can even sacrifice other goals/ economic interest for censorship control.

Chinas internet plus policy: Chinese companies are trying to extend their influence and
customer base abroad.

On the issue to governance of internet, China has favoured strict control of govt over internet.

Great Cannon:

It can be used to intercept foreign web traffic.

Could be used to spy on anyone who happens to fetch content hosted on Chinese
computer.

The Great Cannon is not simply an extension of the Great Firewall, but rather a distinct
tool that hijacks traffic to individual IP addresses.

Possible functioning of Great Cannon:

It intercepts any computer trying to access any data on Chinese website.

It can then divert the data to a different location.

From there, by use of malwares and all, Great cannon may even be used to spy on
that computer.

Thus, it is more powerful tool than Great Firewall which just blocks access.

Topic: Environmental pollution

23) A new study has found that the dispersant used as a remedial measure to clean the ocean basin after
the 2010 Deepwater Horizon oil spill in the Gulf of Mexico is more toxic to corals than the spilled oil.
Examine why and what are its adverse effects? (200 Words)
Down to Earth

Introduction:

Oil Spills:

An oil spill is the release of a liquid petroleum hydrocarbon into the environment, especially
marine areas, due to human activity, and is a form of pollution.
Oil spills may be due to releases of crude oil from tankers, offshore platforms, drilling
rigs and wells, as well as spills of refined petroleum products.

Oil spills penetrate into the structure of the plumage of birds and the fur of mammals, reducing its
insulating ability, and making them more vulnerable to temperature fluctuations and much
less buoyant in the water.

An oil spill represents an immediate fire hazard.

Spilled oil can also contaminate drinking water supplies.

Contamination can have an economic impact on tourism and marine resource extraction
industries.

Birds may ingest the oil coating their feathers, irritating the digestive tract, altering liver function,
and causing kidney damage.

The Deepwater horizon spill of 2010 was one of the largest incident of oil spill with more than 5
million gallons of oil was spilled with long reaching effects to human life and environment.

The basic approaches for removing the oil from the water were:

burning the oil,

filtering offshore

collecting for later processing

use of dispersant

Dispersant used: Corexit

How dispersant works?

Mechanism of action of dispersants used:

Emulsification of oil: On contact with the dispersant, oil that would otherwise float on the surface
of the water is emulsified into tiny droplets and sinks or remains suspended in the water.

Bioremediation: allows the oil to be more rapidly degraded by bacteria and prevents it from
accumulating on beaches and in marshes.

Dispersant is used to make oil mix well in water so that:

Oils concentration decreases.

Microbes will be able to digest it properly in lower concentration and soluble form.

How use of dispersant may have worsened the situation:

1. Oil plume:
Because the dispersants were applied deep under the sea, much of the oil never rose to the
surface. Thus, they are accumulated as plume under the surface.
These plumes are broken down at very slow pace, in the cold, making it a long-lasting threat to
vulnerable marine life.
2. Harmful effects of dispersant: a recent study found the dispersant to be

more toxic than the oil itself where the effect of various concentrations of oil and
dispersant were studied

it is toxic to the corals at lower concentrations than the oil

Dispersant spread fast in water. It increases its area of damage.

Because of harm to coral other fishes are also contaminated and dyeing prematurely.

Eggs of pelican are shown high percentage of dispersant.

Dolphins and Turtle infants are dying 6 times more than usual figures in areas where
dispersants were used.

Conclusion:
Need to reanalyze cleaning technique.
Strict penalty on oil spillers
The importance of these findings lies in the fact that a full enquiry into the effects of the oil spill
and the clean up techniques needs to be done. This could reduce the threats to marine flora and
fauna. Develop better techniques for such future occurrences.
Though use of dispersant is useful, but its long term side effects was not tested at that time,
because it was immediate requirement to take precautionary measures in 2010.
Adverse effect can be decreased by further usage of specifically cultured bacteria to eat away oil
traces near to shore.

Topic: Infrastructure Energy

24) Examine the challenges and problems that are faced while trying to use
renewable sources for large scale energy production and distribution
with less hours of outages

Introduction:

With threat of climate changing staring in face of the world, harnessing renewable sources to meet
energy demand has become a global priority. Germany is the best example in this regard which is
shifting its energy production sources from Non-Renewable Energy Resources to RER and currently
25% of its total production comes from RER
Renewable energy constitutes less than 30 percent of total India's energy production and is in its
nascent stage. However using renewable sources for large scale energy production and distribution
comes with its set of challenges.

Challenges:
1. Both solar and wind sources are weather dependent; many times weather is not favourable such as
overcast sky, storms etc; ensuring steady power supplies in such cases is difficult. This hampers
enterprises, industries. Adverse weather with no power generation may lead
to outage of power system
2. Our grid system is designed as Edison system which requires continuous and large Input but in RER
inputs are small and variable power flows. Thus keeping constant supply is biggest challenge. India
started Green Energy corridor scheme for this problem. Also for this small grid should be promoted.
3. The capacity to absorb the power generated from renewable resources by the grid is
limited. Also to harmonize different renewable sources over the grid is difficult
4. Land acquisition: sources such as wind & solar both require large land area to ensure large scale
generation; difficult to obtain in agrarian countries like India.
5. Proposal to utilize batteries to meet peaks loads during evening & mornings is marred by poor battery
efficiencies.
6. High initial cost of investment, High cost of per unit of production
7. Technology expertise in harnessing sources such as wind, tidal, geo-thermal etc is unavailable with
most of the countries.
8. Storage of such a huge quantity of energy is an issue. Batteries which are presently available run out
of capacity and are expensive which raises the cost of electricity. Storage problem to balance the
requirement in lean & peak periods
9. Long gestation period on returns on investment has dissuaded industry.
10. Poor quality of electricity delivery
11. Even with reduction in prices of solar and wind technology it is still capital intensive after including
operational and maintenance cost.
12. Funding crisis - Lack of developed long-term capital market in many countries. Government is looking
for foreign investment in this sector. India started national clean energy fund
13. Change of mindset required - it is believed that for energy production from RER 100% back up from
non renewable energy resources is required but it is not true as proved in Germany.
14. The sector requires high skills which are not available with most countries. Manufacturing capability of
many nations is weak in renewable energy equipments leading to higher imports.
15. Harmful environmental effects of large scale projects. Hydro-electric projects in hilly areas which are
fragile & seismic regions can cause Earthquakes, Uttarakhand flood like situation.

Conclusion
India has lofty target of 20 GW from solar energy by 2022 and total energy production from RER to 55
GW by 2017. For this many initiatives like solar energy corporation of India, offshore wind policy etc
have been started by the govt
Though the challenges remains the government with better policies, thrust on innovation & best
practices around the world can go a long way in harnessing this eco-friendly, sustainable & cost
efficient source of energy to feed its energy crisis sectors. This will save a huge import bill and move
to a regime of self-sufficiency. harnessing renewable is not a matter of choice but necessity

Business Standard

Topic: indigenization of technology and developing new technology.

25) Recently India agreed to purchase 36 Rafale fighters from France through Government to
Government route. Critically comment how this deal would affect Make in India campaign, defence
indigenisation process and security of the country

Introduction:
According SIPRI, India is the largest importer of defence equipment in the world. Recently GoI has
decided to directly purchase 36 Rafale fighter aircrafts from France through a government-to-government
deal due to critical operational necessities of Indian air force & because Rafale was the lowest bidder..

Impact:

The deal's affect on various spheres will be:

1. Make in India: with India buying jets in a fly away condition, France is no longer required to supply
proprietary technology or intellectual property or establish a part of production in India. Thus no
transfer of technology or job generation for India happens.

Deal is a setback to the ambitious project of boosting manufacturing in India 'Make in India'. Its
main motive is to reorient the India's manufacturing sector through Export oriented way so as to
have a double benefit of domestic demand and creating demand for exports. India with large
population and favourable demography will benefit from Make in India initiative via employment,
raise in income, economic development

2. Defence indigenization: no technology benefit derived by India from the deal; IAF will remain
dependent on Dassault for maintenance and supply of spares; our future modernization and
upgradation of this platform will be costly and become depended on France.

3. Security of country: with IAF down to 34 squadron against authorized 42 squadrons, deal will add two
squadron boosting prowess of IAF; but deal is unclear as to how supply will be made within two
years; as such supply will require Dassault to upgrade its manufacturing capabilities which will take
time; supply can be expected to take 3-4 years.

4. The deal would cost 4000-6000 crore annually to IAF capital budget. This could have been utilized in
R&D of defence sector

5. No generation of jobs or defence infra within India

Critical Note:

National security is of paramount importance especially with growing threat from China, Pakistan. Lack of
latest artillery and equipment for the defence is highlighted. The buying of Rafale fighters is an urgent
requirement and the country cannot compromise on the national security.

The deal was being negotiated since long & was on verge of collapse. Moreover keeping in mind IAF's
emergent requirement, deal has been saved and in future Dassault may be invited to partner with an
Indian private entity and manufacture Rafale in India.

Rafale deal had become a road block for many other deals like nuclear, defence and space cooperation.
Buying Rafale fighters is a win-win situation for both the countries to venture into other areas of
cooperation and also for India to meet its urgent requirement.
India is making efforts to move beyond a buyer seller relationship in defence equipment. Co-production,
Co-manufacturer and technology transfer are being preferred. With the government increasing FDI in
defence to 49% India is expecting to boost domestic manufacturer. Also the "Make in India" campaign
aims to promote local manufacture.

Conclusion:

Defence indigenisation process is a long term aspect but the current requirements cannot be
compromised given the inability of the Indian companies like HAL to manufacture within short span. It
will take at least 6 years to get the aircrafts delivered in flyaway condition if manufactured
indigenously.
The move of government is a right way of balancing the national security with 'Make in India' as
evident from the buying only 36 out of expected 100 plus fighters and inviting 'AirBus' manufacturers
for make in India.

Business Standard

The Hindu

Topic: Agriculture issues

26)It is said that Indian agriculture is the least reformed sector of the Indian economy. In your
opinion, what systemic reforms are needed to revive Indian agriculture and make it sustainable
occupation for farmers? Critically discuss.

Introduction:

Agriculture sector provides employment to 57% of the population directly or indirectly. Governments
since independence took several steps to reform agriculture, but agriculture, though a Prime Moving
Force for Indian economy, presents a grim picture & remains a unproductive & unviable sector

Reforms needed:

1. Lack of public investment on agri infrastructure is hurting the industry. Improvement of irrigation
facilities, 'Mandi' modernization, cold storages, and separate power feeders for agriculture should be
prioritized.
2. Marketing Reforms so that farmers are not forced by traders to sell their produce at farms door at
distressed prices:
Not all states embraced model APMC acts.
Rationalization of taxes on agri produce (Some states like AP taxing 6% for transactions in
'Mandis')
Forward linkages should be established with food processing and Pharma industries.
Community forming should be encouraged which shares both profit and risk.
APMC
e-choupals
3. Rationalization of Subsidies- Subsidies on fertilizers and power are distorting market prices. MSP for
paddy crop is not only pressurizing water resources but also discouraging non-MSP crops. Instead of
subsidies, Govt should concentrate on providing capital infrastructure
4. Integrated farm management (Crop rotation, Supplementation of farm income with poultry, dairy,
cattle rearing, fodder crops etc.)]
5. Revival of Mandis and commodity and futures trading for farm produce.
6. Timely availability of quality seeds, fertilizers and power.
7. Improving the soil quality and fertility using sustainable agriculture practices such as organic fertilizers
and pesticides.
8. Propagating agri-extension services and Kisan call centres etc.
9. Easy crop loans and focusing on other substitute options such as Kisan credit cards etc.
10. Kisan call centre, soil testing laboratory,
11. Enough revenues shall be generated by farmers so that he wont have to work under MGNREGA, as
construction worker which are seasonal in nature.
12. crop insurance
13. Consolidation of land holdings. - fragmented units need to be made productive (more than 80%
farmers are small & marginal) through
farm mechanization
cooperative farming
availability of credit ( institutional players)
14. Also extension services reforms like information sharing to adopt profitable & sustainable farm
practices & to adapt to climate change
15. Feminization of agriculture- as India is witnessing a male dominated migration therefore needs to
make agriculture more female friendly.

Conclusion:
On the eve of independence Indian agriculture was totally stagnant. It was only a subsistence occupation
for millions of farmers. Since then it has come a long way but still a lot of ground has to be covered. With
implementation of above mentioned reforms required dynamism and competitiveness will be infused in
the agriculture sector and make it a vibrant sector of the Indian economy.

Business Standard

Topic: Indian economy development; environment protection

27) It is often pointed out by environmentalists and other social activists that while improving
the ease of business is highly desirable, it cannot be the sole criterion for recasting the
legal and regulatory regime governing the environment and forest sectors. In the light of
governments recent move to bring a single statute by unifying forest and environmental
laws to ensure smooth development, critically comment what should be governments
priority to ensure ease of doing business or protecting environment and biodiversity?
And explain why.

Introduction:
Environment vs. dev has been a long standing debate. For few years, the decreasing rate of growth
of Indian economy is being attributed to policy paralysis, which arose due to excessive bureaucratic
framework, red-tapism and many incoherent and complex laws.

In a bid to boost the ease of doing business and eventually attract more investment, GoI is:
a) easing the rules for corporate sector
b) decreasing the time taken for any approval
c) tweaking the entire regulatory regime to prevent abuse of environment and rights of forest based
communities by:
Self certification based regulatory mechanism in place of certification from pollution control
boards.
Administrative tribunal instead of judicial NGT to review clearance on appeal.
Fast track clearance for power, mining and lining projects.
Doing away with veto power of gram sabha over forest lands.
Confine "No Go Area" to only very dense forests having canopy cover of 70%.
Changes in Act like Wildlife ,Biodiversity, mooted; TSR panel recommendations
Repealing various regulatory laws viz labour
consolidate all the prevailing environmental, biodiversity laws in one law , so that red-tapism can
be avoided and a clarity regarding rules to follow can be brought

Concerns:

This move is seen by many environmental activists as pro-corporate stance of government which can
produce enormous damage to environment.

1. Self certification has a wide scope for misuse by industries.


2. Diluting NGT will only loosen enforcement of environmental laws;
3. By turning a blind eye to environmental aspects will invite disasters; as seen Uttrakhand tragedy.
4. Limiting the veto power of Gram sabha and removing public Consensus is against letter and spirit of
73/74th amendment & grass root democracy
5. Encourage Naxalite like movements in concerned regions.
6. Reducing "No go areas" to 70% canopy cover will lead to further degradation of already shrinking
forest covers.
7. Wildlife act and biodiversity act were for communities living there. such interference in their life may
not be welcomed, Ex Niyamgiri Orissa
8. Different govt at centre and state has led to a collision course on the environmental laws
9. Defining No go area as with 70% canopy will greatly restrict the development activity

Debate:

Country's growth should not come at the cost of environment and indigenous communities; it is
essential to balance the development and environment concerns; any development oriented
regulatory regime should satisfy environmental mandate of present times.
Development and not just Growth should be the goal to be pursued. Also delicate balance between
the negative Externalities caused by a project should be taken into account. At present Government
seems to be focussed on the ease of doing business parameters only.
Panel Recommendation and discussion with civil society must be considered with involvement of
local inhabitants before taking any further Steps.
For ease of doing business, it is not required to weaken the environmental requirements. What is
necessary is to reduce the delay caused by different sets of law. A consolidated law will be helpful in
that regard.
It will have a clear objective, procedure to be followed and penalty in case of non-adherence. The
requirements can be as stringent as in existing laws and it will give less space to corporate to
manoeuvre the law and exploit the existing loopholes across different laws. It will be easier to bring
changes to one law to adapt to changing requirements.
Ruling out any improvement in existing framework is not an option, instead environmentalists should
argue for an effective consolidated law.
The existing conservation laws marred by many limitations like being obsolete, obstructionists, less-
coherent and lacking synergy with other laws needs to be re-examined, re-oriented, fine tuned and
synergized with other environmental laws in tune with the need of the time.
Both the imperatives of development and environment conservation should not be viewed as
conflicting but in integrative manner.

Conclusion:

Ease of doing business on one hand and protecting Biodiversity on other are both equally important
and Propensity towards any one of them may affect the other adversely. The need of the hour is to
maximize the synergy between economic development and environment which results in long-term
sustainable economic growth.

Business Standard

Topic: Environment pollution, Conservation

28) In the West, some scientists are trying out a new idea called Human Composting to
produce compost to plant trees. Examine the science behind composting, advantages and
issues arising out of Human Composting.

Introduction:

Human composting concept is based on the principle that Human bodies post death can be used for
giving new Life as Human body is rich in essential minerals needed by plants. Idea of Human
Composting was inspired by "nurse logs" in the forest where fallen trees provide nutrients to saplings
The idea of human composting is to use dead bodies of humans and turn them into compost using
scientific methods. It has originated in the US and is called the Urban Death Project

Process:

In it, Human body [Nitrogen rich] along with wooden chips [carbon rich] are placed together along with
some catalyst like Nitrogen compost etc to allow bacterial action over time in a monitored way. This
becomes bio compost and can be used for vegetation ,plants, improving Humus quality, soil
improvement as it will be rich in Nitrogen, Phosphorous, Potassium, Calcium and Carbon

Merits :
For New urban landscape, where land is scarce it can solve the issue of area required for burials
cremation by recycling human body back to soil without allocating land
Cost effective; It saves compost forming cost;
Unlike conventional methods of disposal such as burials and cremations, where either the body takes
ages to decompose or the incineration releases green house gases into the atmosphere causing air
pollution; Body Composting helps create bio compost suitable for being used as organic fertilizers to
improve soil quality
Culturally can be accepted as Death can be linked to nature and new life
For nations relying on Urea, this could be an option. Its a natural method to meet demands of
fertilizers and reduce dependence on synthetic fertilizers and corresponding decrease in fertilizer
subsidy
Also use of natural things rather than formaldehyde[ carcinogenic] + metal boxes
Land for Burials can be now orchids, and areas of vegetation.
cremated sites are less appealing Vis a Vis to blooming flowers
Environment friendly as burial may cause groundwater pollution, funeral produces ash which enters
rivers etc.

Concerns :
Human body may contain traces of heavy metals like tooth fillings, bone repair rods and other
implants prosthetics. Such metals can contaminate soil. Hence, prior to composting such metals need
to be removed or else the compost shouldnt be used for soil growing human consuming food items
Legislative framework is missing on such new methods.
People died from certain disease which even after death remains can be spread to those eating
plants from that soil Bioaccumulation;
In India its not viable because majority of Indians prefer to burn the body rather than bury it. So, land
is not an issue here. Also, human composting cost is more than what Indian style burial costs. Hence
it is viable in US and other developed nation
Religious and cultural sentiments -. Orthodox and Family emotions may act deterrence to it as they
may not allow the body exposed and not with proper ritual bidding. Certain nations require bodies to
be buried, entombed, cremated
Ethical problems over people actually accepting this method. It would behove one to use their final
remains for a noble cause as providing for another life even after death. Utilitarian ethics would
approve it; however personal ethics and value systems would consider this an aberration of a certain
taught principle according to the tenets of the religion, sect of the concerned person. For
e.g. for Hindus, cremating the body is believed to free the soul of its earthly limitations. An idea of
composting the body may be taken as an insult by many families.
Vegetarians protests use of human compost on lands meant for growing food
In cold countries, low temperature will not allow composting; controlling temperatures will add to cost.

Conclusion:
Research should be promoted to open new avenues for cost effective and animal composting.
Composting is a noble attempt, if the positive effects are well conveyed to the general population.
Like Organ donation, Consent can be taken by individual for human compost. With growing human
population, rising concerns of climate change and reducing spaces for burial grounds, this method
needs to be adopted

The New York Times


Topic: Science and Technology- developments and their applications and effects in everyday life

29) Digital revolution is fundamental to inclusive growth of the country and sponsored data or
zero rating is the pathway to realising the Digital India dream. What do you understand by
Sponsored Data or Zero Rating offering? Examine how it benefits economy. (200 Words)

Livemint

Intro:

1. Sponsored Data or Zero Rating offering is a plan provided by internet service providers
(ISPs) under which the ISP does not charge anything for a pre-defined bouquet websites
and apps. The idea behind Sponsored Data is that ISPs get paid by the big internet companies,
the internet companies (websites, apps etc.) get to increase the user database and the user gets
to access some websites free of cost. E.g. Internet.org of facebook.

2. "Sponsored Data or Zero Rating Offering" is a practice wherein data charges resulting from
certain type of usage say access to specific website, apps or web content are billed directly to the
sponsoring company (which is website or app itself) and not to end customer.

Benefits of Sponsored Data:

1. Expands the digital reach, especially in developing countries like India, where data charges are
still high for its universal reach given the economic status of majority of the population.

2. Increasing the scope of e-governance & m-governance: By collaborating with ISPs for
Sponsored Data, Govt services can be accessible to public free of cost.

3. In developing countries like India where only a minority (about 20%) of the population has access
to internet, Sponsored Data could help in increasing internet access among the populace.

4. Increased revenues for ISPs: Necessary because they spend considerable amount of capital
on building infrastructure for providing internet. Thus, incentive for them to innovate and invest.

5. E-Commerce markets nearer to people thus helping in increased economic activity and at the
same time, more revenues for the Govt due to higher sales.

6. Increased penetration will increase demand of services leading to improved business


opportunities for people. This will spur growth and job opportunities.

7. Augments Digital Literacy.

8. These apps also helps in furthering the social goals of government in sectors like health,
education, agriculture by incremental usage of m-education, m-health etc.

Concerns:

1. Violates the principle of Net Neutrality.

2. Sponsored Data makes ISPs the gatekeepers of the internet traffic thus deciding which data
the users should use or not.
3. Discriminatory and restrictive nature of the arrangement. A few companies can potentially
acquire monopoly over Internet. The consumers do not get to choose the list of websites that are
offered free.

4. Discourages the spirit of Innovation and Entrepreneurship which has made the internet what
it is today. But for the net neutrality today we would not have the Googles & the Facebooks.

Inclusive Growth:

1. Digital inclusion is a sine-qua-non in the present day knowledge economy for us to realize
our dream of inclusive growth.

2. Sponsored Data is a very effective tool to bring in Inclusiveness, which is the key to
balanced growth as more and more benefits can be reaped with transparency & integrity thus
improving the overall governance.

Conclusion:

1. Sponsored Data services are a double edged sword and hence the implications of allowing
them should be carefully studied and debated before a considered decision is taken. The
government should adopt such policies that deliver connectivity to the masses at affordable
prices and reliable speed.

2. Given the focus of Govt on Digital India and use of electronic services, such kind of sponsored
data may be very beneficial for the masses regarding their outreach with the Govt and similarly,
Govt can also easily contact its citizens thereby reducing the Digital Divide to a large extent.

3. At the same time, it is essential for the government to ensure that net neutrality is not
compromised. It should be ensured that some big corporate ISPs and established companies
dont monopolize the new market effectively pushing the small and new entrepreneurs out of the
market. Therefore it is necessary to have a balanced approach for Sponsored Data which does
not violate the spirit of net neutrality.

Topic: indigenization of technology and developing new technology.

30) What is the difference between Amplitude Modulation, Frequency Modulation, Medium Waves
and Digital Radio? Examine which one is better to send clear signals to receivers and why. (200
Words)

The Hindu
Reference

Radio (2 types) 1. Analog radio

2. Digital radio

Analog Radio works on two modulations:-

1. Amplitude Modulation (Modulation of amplitude, frequency unaffected)


2. Frequency Modulation (Modulation of frequency, amplitude uniform)

Frequency modulation: In FM new artificial waves are created in such a way that the program
signal causes fluctuations in the carrier's frequency but will have the same amplitude. Because of uniform
amplitude the radio voice quality is better in FM.

Amplitude modulation: In AM an existing wave is superimposed with different amplitudes from


the source transmitter. In such modulation the receiver does not receive same signal strength all the time
because of different amplitudes. This is the oldest method of radio transmission.

Medium waves: Medium wave (MW) is the part of the medium frequency (MF) radio band
usually between 500 kHz to 1600 kHz and is used mainly for AM radio broadcasting.

Digital radio: Digital radio, as it's called, sends speech and songs through the air as
strings of numbers. It is the most efficient and new technology used in radio transmission. Interference
from the surroundings is minimum via this medium because of its discontinuous nature and wide band
width, and therefore voice quality is best in the radio.

Which one is better to send clear signals to receivers and why:

1. The trouble with AM and FM is that the program signal becomes part of the wave that carries it.
So, if something happens to the wave en-route, part of the signal is likely to get lost. And if it gets
lost, there's no way to get it back again.

2. Digital radio can help to solve that problem by sending radio broadcasts in a coded, numeric
format so that interference doesn't disrupt the signal in the same way.

Topic: Agriculture issues; Issues related to direct and indirect farm subsidies

31) Critically analyse the trend of institutional subsidised agriculture and its effect on agriculture
and farmers in India during last two decades. (200 Words)

Business Standard

Intro: In a country like India, more than 50% of the population is engaged in the agriculture and majority
of them are economically weak living in hand to mouth situations. As a result they need government
support in the form of subsidies.

Institutional subsidized agriculture:

1. Input Support: It can be of great benefit to the farming community and agriculture if is optimally
targeted and utilized. But over the past two decades, its functioning has shown some anomalies.

a. Firstly government provide subsidized urea to help the agriculture sector but excessive
use of it over the years have imbalanced NPK ratio (In Punjab, N:P:K ratio was 42:13:1
as compared to standard 4:2:1 in 2012) resulting in adverse impacts on environment and
ecology of the area and consequent impact on soil fertility, ground water availability,
polluted water etc.
b. Excessive use of agro-chemical inputs like insecticides, pesticides have resulted in
issues like land degradation, soil erosion, decrease in productivity etc.

c. Further government provides subsidies in irrigation which is also being used


excessively. Moreover, the decrease in crop productivity has made farmers use more
water in order to compensate for it resulting in problems of salinity-alkalinity along
with depleting ground water table. Cases of states like Punjab and Haryana are
testimony to this.

d. Free/ Concessional power for agriculture led to more power utilization coupled with
reduction of water table.

2. Credit support (Concessional credit facilities, Kisan Credit Card)

a. Not properly utilized or mis-utilization by wrong beneficiaries.

b. Banks reluctant to give loans to small farmers fearing of NPAs.

c. Non-availability of credit at right time results in delays in sowing, harvesting etc and suicides
in the extreme cases.

Conclusion:

1. It is high time that government gives utmost concern to the sector with immediate effect as the
sector not only provides food to the country but also involves half of the country's population
mostly living on margins.

2. There is a need for better targeting to subsidies along with raising awareness among the
beneficiaries as how to reap its benefits efficiently without impacting the agriculture.

3. Subsidy linked to organic farming and other capacity building activities may be incorporated in
Institutional support mechanism.

4. In a democratic country like India, it is not politically feasible to stop Institutional support to
agriculture. Hence, it would be better to reduce them slowly in time bound, phased manner but
supported by well framed, timely and well executed policies framework strengthening the farmers
and enabled them to do self-sustaining sustainable agriculture practices.

Topic: Indian economy development; environment protection

32) If self-reliance can be achieved in the strategic fields of space and nuclear technology,
through dogged pursuit and by creating institutions of excellence with political support, there is
no reason why it cannot be done in the equally strategic area of military hardware. In your
opinion, what needs to be done to improve and increase defence indigenisation and achieve same
success as achieved in Space sector in the defence sector in terms of developing own
technology? Critically discuss. (200 Words)

The Hindu

Intro: Indias status as the largest importer of arms shows its large scale dependence on other
countries to fulfil the needs of the armed forces. The increasing bill of defence goods imported every year,
calls for a new strategy to be formulated regarding indigenisation of defence goods production to attain
self sufficiency in future in defence sector.
Measures required:

1. Improve and drastically change the management and organization of defence research
institutions especially DRDO, through better budget allocation and recruiting more quality
scientists to lead the initiatives.
2. Need to break down the components and set up institutions separately similar to what is
done in ISRO, where there are many branches which work on different parts like parachute,
engine and others.
3. Need to have high degree of coordination between research institutes of space and defence. This
will leverage inter institutional development.
4. Incentivize Private companies to get involved in the initiative and to encourage time bound
projects, as opposed to TEJAS project which took more than 30 years, to be introduced in the Air
force.
5. The manufacturing license attainment procedure should be simplified, to allow more
designs to proceed in lesser time.
6. Political incentives to institutions like ISRO have led to the completion of MOM project. Similar
enthusiasm can be shown in defence production, to motivate the concerned institutes to develop
innovative designs which have the same or better efficiency when compared to imported
products, as opposed to popularising FDI only.
7. Also, there is a need to convince the defence exporter countries to transfer technology too,
so as to make the upgradation process of the present equipment easier.
8. Skill India program can be used to encourage the youth to join the initiative to make new
designs for equipments for the armed forces, which will also simultaneously help in creating more
jobs.
9. Due to our urgent needs, we always dilute the offset policy. Such tendency needs to be controlled
in future.
10. Encourage low cost alternative technologies in current manufacturing and improving new
processes to re-orient towards better implementation.
11. Policy stabilization is very essential otherwise our ultimate goal of Increasing FDI for technology
transfer will not be achieved.

Steps taken:

1. New Defence Procurement Policy and following offset policy in defence procurement.
2. Allowing FDI in defence sector.

Way Forward:

1. We have already made helicopters and HAL is now able to upgrade Sukhoi as well. So we
already have some expertise. We now just need to use our diplomacy with US, Russia, France
etc. to share their technology more and more while doing any deals so that theme become not
only Make In India but also Made by India.
2. It is through increased efforts from both private and public sectors and with coordinated policy
actions from Govt, the indigenisation of the defence sector along the lines of nuclear and space
technology will be possible for a self reliant India.

Topic: Conservation

33) Discuss the objectives, design and performance of Integrated Development of Wildlife
Habitats (IDWH) scheme of the Union government. Suggest if any improvements needed in the
scheme. (200 Words)

The Indian Express

Intro:
1. "Integrated Development of Wildlife Habitats (IDWH)" scheme, rechristened in 2009, is a
revamped version of the earlier "Assistance for Development of National Parks and
Sanctuaries programme" of Govt. of India. While the former version focused on wildlife
conservation efforts within Protected Areas (National Parks, Wildlife Sanctuaries, Conservation
Reserves and Community Reserves), the latter version backed the efforts outside the Protected
Areas too.

2. The goal is also to recover critically endangered species and habitats outside PAs and areas of
high biodiversity value as well as to protect corridors and areas adjoining PAs.

3. So far the scheme has been centrally sponsored. Funds have been used mainly for activities
such as habitat-improvement, reducing human-wildlife conflict, wildlife veterinary care, capacity
building, etc.

4. Shift in Govt approach from Protected Area to landscape centric approach.

Issues:

1. With all these efforts, while there has been a significant improvement in the habitats and species
population, at times, they proved to be counter-productive.

For instance, the conversion of grasslands to plantations or the development of water resources
is not in tune with the ecological need of the species. Therefore, a scientific and ecological
assessment of the spending activities is the need of the hour.

2. Very little response for Govts call for private funding of the IDWH scheme.

3. With growing the man-animal conflicts due to infrastructure & agriculture development, the animal
space has been consistently depleting.

Some changes required in the Scheme:

1. Better monitoring facility.


2. Learning best practices across the world.
3. Recruiting compassionate human resource for the job.
4. Clear national policy on the subject.
5. Skilling & involving local & tribal communities for the conservation to make the scheme
sustainable.
6. Routing the Funds under Corporate Social Responsibility from Health and Education to
Wildlife.
7. Encouraging the Private Individuals/Entities and Philanthropists to spend on the
development of Habitats on Public Land or by donating land and/or developing it for Wildlife
Habitation.
8. Encouraging the Participation of local population in the protection of the area through prevention
of Poachers, Plantation in the area etc.
9. Removing the fences or the products from such Areas which may prove lethal for the Animals.

Conservation efforts needed outside protected areas because


1. Some wildlife species such as sarus crane, endangered snow leopard are predominantly found
outside protected areas.
2. Habitats essential to wild species such as grasslands, river systems are often situated within
human dominated landscapes.

Way forward (Measures which may be taken) for wildlife protection and conservation:

1. Appointing philanthropist entrepreneurs as ambassadors could be of some help.


2. Civil societies must also be involved to persuade business people for funding the scheme.
3. Drafting and implementing different conservation strategies for different species.
4. Private parties should be allowed to take up area-specific or species-specific conservation
responsibility.
5. Taking special measures for the safeguard of front line staff.
Conclusion:

1. Wildlife conservation may not give anyone direct economic benefits, but it gives large-scale
intangible benefits for the entire society. Therefore, it should be regarded as a social
responsibility of everyone.

2. Constitution under Article 48 A, DPSP provides for the wildlife conservation. In wildlife
conservation programmes, the means could be diverse but the end is always identical.

Topic: Economic development; Conservation

34) The concept of ecological modernism, which sees technology and economic development as
the key to solving big environmental problems, is gaining adherents (eco-modernists) and getting
a lot of attention these days. Elaborate this concept and examine how is it different from
sustainable development approach. (200 Words)

The New York Times


Reference

Intro:
1. Nature if left alone can continue to support life for millions and billions year. The single most
unstable and potentially disruptive element in environment is Human and his development spree
at cost of environmental damage.

Eco-Modernism vs. Sustainable development


1. Eco-Modernists: With rise of modern technology and efficient methodology of human activities
there is hope that we can do better in lesser environment damage and leave nature in more part
of earth undisturbed. EM advocates the confining of human activity in a smaller part of the planet
through smarter, greener and more efficient technologies thereby allowing nature to rest.

2. Sustainable development means economic development without depleting natural


resources in long term to keep development sustainable with demand of humans and supply
from nature, while Ecological modernism means to use modern technology and efficient
method to obtain human needs from less harming the nature and returning the natural
resources unclaimed.

For ex. Pollution control as per Sustainable development contain: Public Transport, lesser usage
of vehicles, efficient engines, solar energy usage etc. while Ecological modernism will take path
of Carbon sequestration, highly efficient engines, usage of bio-diesels, hybrid cars etc.

3. Environmentalists in favour of Sustainable Development argue to de-intensify human


activities & approaches such as reduced use of chemical fertilizers for cultivation, no mega dams
etc.

While proponents of eco-modernism propose that approach to solve our environmental


problems should always involve technology (innovation). Solution lies in intensifying human
activities such as cultivation, energy extraction and settlement through innovative technologies so
that they use less land and interfere less with natural world.

4. Sustainable development wants to ensure that needs of future generation are not
compromised; it couples human welfare with environmental impacts while eco-modernism
de-couples human welfare from environmental impacts through technological interventions.

5. Sustainable development advocates usage of resources below certain intensity while eco-
modernism advocates using resources at high intensity so that needs of all people are fulfilled
and society transits into developed one.

Examples of Ecological modernisation are:

1. Carbon Sequestration for CO2 capturing.

2. GMOs.

3. Raising yields of agriculture and leaving rest of land as forest or dense green cover.

4. Making industries, building more environment friendly by making its green architecture to lessen
energy requirement.

5. Keeping storehouse in opposite side of sun, covered with trees, using Coolers for low temp
difference while AC for high temp difference, segregating fruits from crops and cooling in
categories.

6. Better logistics in transport and road construction. Repellent material in road covers to
increase efficiencies of vehicles. Straight and freeways to increase mileage of heavy transport in
long distance.

7. pro-nuclear, pro-mega dams, pro-urbanization and pro-geo engineering of the planet

Criticism of Eco-Modernism:
Seen as a ploy to push unaffordable patented green technology by the developed countries in the
garb of protecting nature.

Lacks Social and political compass.

Conclusion: Ecological modernization definitely holds promising future and it is one concept to be
proposed in Paris meet for environment discussion this year. But the need of the hour is to wait and
watch and proceed cautiously.

Topic: Challenges to internal security

35) Critically analyze the strategy used by Naxalites in India to attack state forces and expand
their base. (200 Words)

Introduction:

Since their inception in 1967, Naxalites, the extremist left wing, have been making relentless efforts to
expand their base and achieve their stated goal of creating a Red Corridor, stretching from the border of
Nepal to Karnataka.

As per the writings of Mao Zedong, their strategy seems to be


1. Establishing regional bases in areas situated in isolated and difficult terrain.
2. Progressive expansion by means of attacks on state forces, elimination of persons with alternative view
points, etc.

Their strategies:

They have been changing their approach from time to time in the implementation of their strategies.

- In the beginning, they used to wage guerrilla wars and inflict surprise attacks. This approach has
two-fold strategy. One, to make the enemy weak, and two, to make common people in those
areas believe that the state is not all that mighty.
- Later on, when they realised that this strategy takes decades to achieve their objective, they
preferred to silently strengthen their network and build capacity. Unfortunately, in this process,
they did not hesitate to make ties with any anti-India force, including terrorists, separatists, crime
mafia, human/animal traffickers, etc., to mobilise money and modern weapons.
- There strategy also focus on enemy, in their case this enemy is forces which are there to fight
them police forces and CRPF. So they try to attack so that when enemy advances we retreated.
- They try to harass the enemy and when enemy is not ready to fight they attack.
- Maoists recruit the people from tribes and people who become rebels due to area's poverty, caste
based ill treatment, issues of tribal dignity and convenient forest cover. They train these people
and also make them better rebels. They get their weapons from north -east India.
- They also try to look for alternative areas for their base so that in bad times they survive and
sustain.
- They plan the attacks on forces very well day and night and terrain for attack and defence. They
have very good contacts to get the information of force movements and their networks are very
strong which are used and this game continues.
- Sometimes, they take up social and economic causes against the govt. without being identified as
naxalites and provoke innocent people. In this way they try to obstruct every development project.
- They also operate their own medical and education services in the areas controlled by them. But,
similar confidence building exercises by state are not tolerated by them.
- They also keep changing their strategies while attacking the state forces. For example, attacking
in huge numbers of not less than a hundred, destroying security camps with improvised explosive
devices, etc. There are twin benefits with such attacks - enemy becomes weak and arms and
ammunition can be acquired.

Government Steps:

- So far, central and various state governments have done a commendable job in restricting their
expansion. Still, there is a lot to be done. Govt. should take every measure to reduce regional
imbalance. It is in those areas where there is administrative and political vacuum that the naxals
establish their regional bases. Sharing the experience of successful states, like Andhra Pradesh,
to troubled states too can certainly help.

Topic: Employment; Land reforms

36) India is one of the countries which is home to largest number of landless people, especially in
rural regions. It is said that the new land acquisition act would provide employment to such
landless people. In this backdrop, critically examine the factors that lead to landlessness and the
employment opportunities available to such people. (200 Words)

Introduction:

- The reasoning behind the push for the LARR bill is to support the alleged foundation of industrial
corridors in resource rich sectors.

- The LARR essentially is a resource mobilization process through which, resource invested
in land would turnover jobs for the landless, and make the financial beneficiaries in this project.

- As per NSSO survey more than 30 crore of Indians are landless. A more grave fact is that majority of
these landless are in rural regions where opportunity of employments are limited. The new land
acquisition act is said to cater to the employment need of these people.

Major reasons for landlessness:

1. Since independence large portion of land has been taken away by the state in the name of
development needs of the country be it dams, power plants etc. This was to a large extent without any
compensation or rehabilitation.
2.Enchrochment in the forest region for extraction of minerals resources and infrastructure has caused
land alienation to large number of tribal
3.Indians has traditionally occupied small farms since independence and the partition of land as the family
grew led to even smaller fragmentation of land.
4. In India there has been sporadic cases of communal violence where the victimized community often
leave their parent land in preference to safety
5. During partition of India when people migrated from Pakistan to India leaving behind their land, many
people did not get suitably rehabilitated.

Employment opportunities of such landless people are often limited:


most of them work in farms of other peasants
they migrate to cities and work there in factories, kilns etc
some them get employment opportunities in govt. schemes like MNREGS
Some of them get employment opportunity in the factories established by taking over the land.

Pros:

Agricultural outcomes are getting low returns, with respect to Investment hence People shows
apathy toward Agriculture and it is also reflected in the recent survey of CSDS where 60% Land
Owners does not want to continue with Agriculture.
The another Problem is of water. As lots Government Projects related to Water Linking are still
pending, it is difficult for them to sustain.
Climate condition is the major player now. Crop Insurance is not much popular among our
Agriculturalists, and hence they suffer.
There is very small number of farmers who shows apathy due to Fringe benefits i.e.; marriage
and education of child, etc.
Recent Land Acquisition Bill is said to be the boon this Land Owners as well as Landless people.
Having a clause of compensation between 1 to 2, it is said to that it will ensure the compensation
part and industrialization of the same land will provide permanent employment to the landless
people. Like, making of Industrial Corridors and SEZ etc.

Cons:

However, the reasoning for this industrial push as is flawed as is clear by the number of demonstrations
that the ordinance has faced recently. The reasoning involved would accommodate and further aggravate
the conditions faced by tribal communities in India in the past:

o The governments claim is that the agricultural sector has faced a decline
since 2000 where approximately the total employment share in the population has
gone down from 60 to almost 50%. If that is the case, then asking people with
land to sign it over the government for the sake of industrialization goes
against the figures the government raised. If landlessness is at 30 crore, then
signing off land for projects would further widen this gap.

o The social trend of land division amongst brothers continues to mar the
level of development. With a further increase in fragmenting, net land holding
decreases per generation. According to Prof. Sens thesis, this level of
unsurity puts the poorer population at a considerable risk, and keeps the poverty
cycle going for those that are BPL.

o Rather than cash-incentive industrialization, enabling the farmer population


would be a better option. Employment elasticity in the agricultural sector has
gone down precisely due to random variables such as weather patterns, lack of
technical and social infrastructure. If the farmer is further educated trends
such as land-fragmentation could be combated. As far as lack of infrastructure is
concerned, better investment in these sectors in respect to cold storages is highly
required. But a lack of these leads to consistent net returns that are a loss.
o The argument that employment would be generated would mostly be available to
skilled sectors. Unless each industry is responsible with a CSR wing that is
focused on the local population, hiring departments would prefer a skilled
workforce which is sadly not available in the rural sector.

Way Forward:

The government needs a properly formulated plan in order to address the rural
sector so that they can truly avail of the ongoing industrialization in the
Indian job sector. Signing off land wouldn't aid them, but be a regressive
factor in their financial security.
In recent Budget, there is sharp cut in MNREGS, so Govt. should increase the fund and have to
ensure drop in leakages via Jan Dhan Yojana.
As govt. did under Jan Dhan Yojana, govt. must ensure crop insurance mandatory.
As Industry, sometimes take decades to build, in such a case, Govt. must give land, back to the
owners.
The compensation is between a scale of 1 to 2, which must be fixed at 2, so states will bound to
cooperate with the same.

Conclusion:

While Ensuring a Growth, as stated by World Bank and others, we should not forget our roots. Poverty is
our big challenge and if measures are not taken in concrete way that it will get worst.

Topic: Issues relating to development and management of Social Sector/Services relating to Education,
Human Resources.

37) Recently the University Grants Commission issues directives to Indian universities that
requires all universities to follow a Choice Based Credit System (CBCS). What do you understand
by CBCS? Critically examine its merits and demerits. (200 Words)

Introduction:

- The absence of any Indian Universities in the top 200 of global rankings of higher education
institutions points to the dismal state of the higher education in India. One of the recent initiatives
on reforming higher education has been the recent directive of UGC for all Indian Universities to
follow CBCS.

- CBCS stands for the choice based credit system, or a cafeteria like system is the solution for a
type of transformation from the traditional teacher oriented education to a student-centred
education.

- Taking responsibility for their own education in this way, students can benefit the most from all
the available resources. Academic commissions and committees such as UGC, TANSCHE and
NAAC recommend CBCS for higher education.

The Choice Based Credit System (CBCS) has the following salient features:
* Each degree has a prescribed number of credits for acquiring it
* Each course is assigned with a fixed number of credits based on the contents to be learned.
* Student has a choice in selecting courses out of those offered by
various departments. There are compulsory courses and elective courses.

Merits:

CBCS is globally being used by all the top Universities and Indian institutes
like IITs etc.

Gives autonomy to students in terms of choosing subjects as per their own interests
and preferences

Facilitates inter-University comparisons easy because of uniform syllabus and method of


evaluation

Facilitates transfer of credits earned in different Departments universities or institutions of higher


education in India and abroad.

Since syllabus is same students exchange between universities will be easy.

Promotes interdisciplinary teaching and learning across all faculties

The CBCS imminently fits into the emerging socioeconomic milieu, and could effectively respond
to the educational and occupational aspirations of the upcoming generations.

Aided by modern communication and information technology, CBCS has a high probability to be
operationalised efficiently and effectively.

Would help in elevating students, institutions and higher education system in the country; taking it
to newer heights by allowing children to follow the course of their choice, from the college of their
choice.

Will encourage all institutions to come on par with each other, in terms of quality education, so as
to attract students to the courses they are offering.

Demerits:

Implementation is main concern for CBCS. There is no fixed thought out plan and this may result
in harming the education system even more.

CBCS consider all Universities as equal in status which is far from true.

CBCS needs to be preceded by increased capacity building, adequate infrastructure and faculty.
These aspects are missing in Indian scenario.

One more concern is that institutions will lose academic autonomy and that they would have no
say in the education structure or policy making.
Conclusion:

- CBCS has huge potential for reforming the higher education in India. However, the time is not
ripe for implementing it. There is a need to bring the institution on similar levels in terms of quality
and infrastructure before implementing it.
- In addition, UGC also needs to emphasize on Infrastructure, curriculum upgradation, research,
industry-academia collaboration etc. for complete reformation of higher education.
- As is often said and witnessed, a well intentioned policy can be a disaster if implemented without
due-diligence or adequate preparations.

Topic: Effects of liberalization on the economy

38) What do you understand by Capital Account Convertibility (CAC)? There is a debate going on
whether India should go for full CAC or for partial CAC. Examine the advantages and disadvantages of
both full and partial CAC. Based on this examination, suggest whether India should go for full CAC or not.
(200 Words)

Definition
Capital account convertibility is a feature of a nation's financial regime that centres on the ability to
conduct transactions of local financial assets into foreign financial assets freely and at country determined
exchange rates. It is also referred to as capital asset liberation or CAC. Such convertibility may be
completely unrestricted or may be partially regulated. Capital Account Convertibility is argued to have a
direct co-relation with economic development of the country.

In Indian context, it refers to the ability or freedom or ease of converting the rupees to other nation
currencies for capital creation, investment abroad and for international borrowing and FDI, FII etc.

Advantages
It will help India deepen its market and help India attain the status of a leading global economy.
Improved access to international financial markets and reduction in cost of capital.
This will open Indian financial markets to global competition and would in the long run contribute
to the development.
This will ensure the availability of large funds to supplement domestic resources and thus
promote economic growth.
Certain class of financial institutions and later NBFCs can access global financial market.
Banks and financial institutions can trade in Gold globally and issue loans.
Fully convertibility will help address issues of current account deficit and trade imbalances on real
time basis.
Full CAC would provide individuals and businesses to freely invest in businesses abroad
providing them opportunities for diversification of investments.
Businesses would be allowed higher External Commercial Borrowing without RBI or government
approvals.

Disadvantages
Full Capital Account Convertibility can lead to erratic flow of foreign capital, which can lead to
financial instability.
In today's globalized world, such close integration can make India vulnerable to global economic
shocks like the 2008 financial crisis.
Market determined exchange rates being higher than officially fixed exchange rates can raise
import prices and cause Cost-push inflation.
Improper management of CAC can lead to currency depreciation and affect trade and capital
flows.
Speculative activity can lead to capital flight from country as in case of some South East Asian
economies during 1997-98.
Volatile nature of capital has the potential to bring instability in financial system as flow will be
unregulated.
Weak institutional and regulatory framework will add to black money cause only and illegal gains
will be hard to prevent.
It has the potential to create another subprime crisis if companies are allowed to have full CAC.
The advantages have been found to be short lived as per studies and also International financial
institutions are sceptical about CAC post 2008 crisis.

Way Forward

While there are advantages of full Capital account convertibility, the Indian Financial System is relatively
underdeveloped to face volatility risks. Consequently, adoption of partial CAC looks more prudent. This
approach will allow the economy to enjoy benefits of CAC in calmer times in addition to controlling the
volatility through better regulations.

Also, considering Indias need of huge investment in manufacturing, infrastructure, full CAC will help
garner foreign investment. However World economy is not doing that good and could be detrimental for
Indias growth. So, a cautious approach, as suggested by Tarapore committee, needs to be adopted and
better risk mitigation and stabilization controls should be in place before opening up capital convertibility
fully.

39) Recently, the Indian government issued notices to foreign institutional investors (FIIs) to extract Rs
40,000 crore from them under a law called minimum alternate tax (MAT). Critically examine why this law
was enacted and what are its objectives. Also examine why some economists regard this law as bad and
unwanted. (200 Words)

Introduction

Paying tax is a fundamental duty that must be adhered to. In the past, many companies, despite having
profits, managed to escape paying any tax on the account of exemptions and deductions available. Many
companies have been seen distributing dividends but showing nil or negative taxable income, thus being
called Zero Tax Companies. The government realizing the mistake enacted Minimum Alternative Tax
(MAT) to bring such company in the tax net.

Definition
The Minimum Alternate Tax (MAT) is a tax levied under India's Income Tax Act of 1961 that targets
companies that show profits on their books and declare dividends, but pay minimal or no tax. Minimum
Alternate Tax is an indirect tax of 18.5% of book profits introduced in 1995 under IT act to tax domestic
companies. The basic cause was that companies keep their books under Companies Act while they file
taxes under IT act.

Objectives

To bring "zero tax companies" into tax net which despite earning profits and paying dividends to
its stakeholders were not paying any tax.
To rationalize the expenditure of the government.
To reduce the fiscal deficit of the country as government.
To create a level playing field among all companies.

Why is it considered bad?

Notices to foreign investors seeking MAT arrears will reopen past transactions. This is bound to
create confusion besides lowering the investor confidence.
The timing of these notices is unfortunate given that various sectors of the Indian economy are in
dire need of foreign investment. This move is bound to scare off foreign investors from making
long-terms investments.
In the short term, it may induce panic sells in debt markets leading to a weakening of the rupee
and higher interest rates.
Many feel it as tax terrorism towards some companies creating a rigid environment for doing
business.
The idea of giving income tax exemptions and incentives to a company and then applying MAT
to get tax revenues from the same company is flawed in itself.
Generally, the decision on application and quantum of MAT is done arbitrary and hence provide
grounds for corruption.
India is already among the bottom most countries in ease of doing business. This will further
impact Indias position.
Besides, this will seriously affect the new upcoming start-ups in India as they will not be able to
convince foreign investors to fund their ventures in the absence of any support from the
government further hindering our economic growth.

Way Forward

There must be objective evaluation of the case. MAT application must be restricted to few areas where
there is absolute distinction between other companies in tax paying like FIIs who invest and run business
through P Notes etc. Thus, government needs to clear the air over the issue as quick as possible to
maintain investors' confidence.

Conclusion
While the objective of MAT is not bad per se, the manner in which the imposition of tax on foreign
companies has been dealt with is questionable. There is a need to revisit the decision. Also, India is in a
developing stage as a country which will need enormous capital. Having a sustainable and predictable tax
regime will do good to attract investors.

40) Indian government is putting in place a new pricing scheme (NPS) for urea manufacturing units.
Discuss why there is special pricing scheme for Urea manufacturing and how different will be new
scheme compared to previous ones. Also comment what it means to end consumer. (200 Words)

Introduction

Urea forms the most important element of fertilizer regime in Indian agriculture. The use of fertilizers is
necessary for increasing the yield of crops. To ensure availability of urea to farmers at an affordable price,
the government has subsidized it by fixing the maximum retail (MRP) price, which is lower than the
production cost.

Importance of Urea

Urea is the only fertilizer that remains under full price control. Government reimburses the difference
between production cost and MRP to urea firms in the form of subsidy. The rate of subsidy is currently
fixed by the New Pricing Scheme-III

But subsidy pattern was highly distorted which leads to over consumption compared to domestic
production and India ended up becoming importer. As of today, India imports more than quarter of its
urea consumption, which leads to high Fiscal deficit also.

Need for special pricing scheme

To ensure availability of Urea at reasonable cost to the Farmers


To incentivize domestic production of Urea and thereby reduce import dependence.
To ensure the subsidy burden is within limits of financial prudence.
To promote balanced use of fertilizer.
Excessive use of urea as fertilizer in relation to Phosphate and Potash fertilizers due to urea
subsidy is adversely affecting fertility of lands. The government wants to rectify this anomaly.
To encourage investment in urea sector in the aegis of 'Make in India'.

Differences between the two

New scheme will give incentives to units which produce urea at more than 100% of their capacity.
New Urea Investment Policy of 2008 had dis-incentivized such units.
Fixed cost not paid under previous urea regime, New Pricing Scheme will rectify the same.

Benefits to end customer


Urea will be available at a cheaper price.

As most of the landholdings are very small, rationalized fertilizers may result in land pooling,
cooperative farming etc.

Redirecting subsidies to improve mechanization will improve farm productivity benefiting the
Mega Food parks.

Provisions to create an Insurance pool against weather shocks.

Policy ensures optimum use of fertilizers, coupled with soil health cards beneficial for farmer and
the nation.

Consumers will have a better choice option between the urea and other fertilizers.

Conclusion

Policy announcement is only the first phase of reforming fertilizer regime in India. The key lies in its
implementation in spirit, otherwise this policy may also have the same fate as has been the case with
earlier policies. The policy also needs to be supplemented with increased investment, adequate supply of
Gas at reasonable cost and reforming subsidy regime to encourage balanced use of fertilizer.

Topic: e-technology in the aid of farmers

41) In the past several years, Indian agriculture has become the victim of unpredictable weather and
climate events. To protect crops, against such an events, what technological measures have been taken
or being proposed? Examine. (200 Words)

The freakish climate events due to increased temperatures are predicted to cause an increased rainfall in
the next decade, after which there could be extreme dry spells. The nature of the change is not certain as
the upper atmospheric circulations can change in the event of melting ice caps.
Preventive Measures
~PHS or Pre- harvest sprouting can be useful where Heat strokes, wet season during harvest are
prolonged.
~ Genome project which sequenced for Wheat for which India partnered can now reap dividends
~ Consensus with Environmentalist & trials for same can achieve Climate resilient agriculture
~In house farming and technologies like Soil free farming and Aerophoenic farming can avert the risk
of such weather calamities due to their Artificial environment for crops . Entrepreneurs can be
encouraged for same

Curative Measures
~ Post harvest losses can be technologically analysed and compensated with Accrual instrument like FIIS
scheme of Gujarat which insures the loss of farmer between the real income and income reduced due to
rains, storms etc
~Post loss, Information dissemination through Green Sim and Green fabhlet can help prevent Farmers
from suicide etc
Technology is a strong step, if not panacea, to all ills. Our farmers will benefit from technology and be
able to contribute more to the nation
Topic: Infrastructure- Housing

42) Recently the Union Cabinet cleared the Real Estate (Regulation and Development) Bill. Critically
comment on the important features of this bill. Examine how these features will have bearing on the
consumers. (200 Words)

Real Estate (Regulation and Development) Bill passed by Union cabinet recently has come as a big relief
for the buyers as well as for overall picture of Indian real estate market.

Features:
1) Independent Regulatory Authority: Real estate regulatory authorities (RERAs) will be established
in every state and be paired with real estate appellate tribunals (REATs) to consider appeals against
orders of RERAs. All commercial and residential projects now need to be compulsorily registered with
RERA.
2)Prevention of diversion of funds: The Bill now provides for a compulsory deposit of 50 per cent of the
total amount realized from buyers into a monitorable account in a scheduled bank to be used only for the
construction of the designated project. This will help in less diversion of funds but it is lesser than initially
speculated 70% so some funds may still be diversified to chit funds.
3) Mandating consumer protection measures:

a) Specification of project cannot be altered without consent of two-thirds of buyers


b) Prevention from accepting more than 10% advance fees without written agreement.
c) Stringent penalties in case of violation and right to reclaim refund in case of promoter failure of delivery.
d) Promoter needs to declare carpet area

4) Mandatory Registration with RERA: Only projects registered with RERA can be bought or sold.
Developers will have to mandatorily disclose all details on the regulator's website.

Advantages to consumers:
1) Consumer will have redressal mechanism
2) They will have more idea about project and they cannot be betrayed
3) Consumer money will be safe and will be refunded in case of issues.
Disadvantages:
1) Existing projects might get delayed.
2) Costs can go up.
Overall this bill will be beneficial for consumers and will definitely benefit India's real estate market in
longer run. It will also help in curbing black money in real estate.

Topic: Infrastructure energy

43) Do you think Indias commercial, diplomatic and military weight are in tune with its energy needs in
the coming decades? In the light of Chinas growing energy needs and its assertive energy diplomacy,
critically comment how India is managing its energy requirements. (200 Words)
India has big dreams and for realizing that India has to secure her energy needs by clean and stable way.
So, far India relied heavily on Persian and Middle East nations for Crude oil while refineries in India.
But with unstable geo-political situation in Persian Gulf and Middle east region is a concern for India.

Commercial Viability of Indias energy need is good, because India is growing fast and with rise in
Industries and local refineries which saves refining cost offshore and also provides local jobs. But, India
needs to focus more on green and clean domestic energy to save her from Paris summit and long
term Kyoto Protocol subjugation.

Diplomatically, India is west strong while eastern weak, West strong because India has good relations
with Iran, Iraq, Saudi Arabia and Russia while in East we have ONGC Videsh limited projects hanging in
Vietnam PhuKhanh basin in South China sea.

Military might is less important for energy security unless we face navy confrontation in Indian Ocean,
Malacca Strait because of diplomatic protest laid by China to venture in PhuKhanh basin. Also, military
aid in Africa, Venezuela, and Persian Gulf region led to strong foot hold for Indias future energy security.

China is assertive in dominative way as we can see her moves in Mauritius, Maldives, Sri Lanka and
Africa where they exploit natural resource in lieu of funds and mining rights. Recently, China took move to
have 400 billion $ Gas deal openly to help Russia at time economic distress against US will. Also, with
maritime silk route and Land silk route belt project, South China sea domination etc it is clear that, China
is directly confronting US with ADB and ADIZ issue.

In comparison to this, India is looking for stable and long term energy security with strong diplomatic ties
with nations. It can be easily demonstrated by ONGC Videsh limited investment in Venezuelas River
belt for oil exploration. Also in African nations, India invested in oil exploration along with human
capacity development. In south china sea, India took bold step to sign agreement with Vietnam the
same day Xi Jingping reached India. Such steps assures long term energy security for India, though it
looks slow.

Topic: Agriculture issues

44) Critically comment on the challenges and policy recommendations proposed by the Economic
Survey 2015 for the revival of Indian agriculture.

Source: Economic Survey 2015, Volume 2, Chapter 1

Introduction:

In India agriculture sector employs almost half of the labour force and have almost 16%
contribution to GDP.
Given the slow-growth rate and low productivity, it is time to reform this sector.
Economic Survey, 2014-15 presents many challenges and recommends various policy measures
to counter these challenges to make the agriculture sector more productive and for inclusive
development.
Challenges of Agricultural Sector:

Small and fragmented land-holdings


Good quality seeds are out of reach of the majority of farmers
Manures, Fertilizers and Biocides are not available to majority of farmers.
Only one-third of the cropped area is under irrigation.
Lack of mechanisation
Agricultural marketing still continues to be in a bad shape in rural India.
Inadequate storage facilities for agriculture produce.

Recommendations by Eco Survey:

Irrigation and Fertilizers:


o The irrigation facilities cover only 40% of the area devoted to agriculture. To expand the
irrigation facilities Pradhan Mantri Sichai Yojana is started and drip irrigation is given
impetus. The suspended irrigation projects are need to be revived and states are
provided with assistance for development of watershed projects.
o Given the dependability on monsoon, ES asks for diversifying the activities in allied
sector and research in drought-resistant varieties.
o The higher dependability on urea and skewed ration of N, P, K fertilizer, which has
resulted in degradation of land need to be corrected according to ES.
Research and Education:
o Given the low productivity and very low yields of crop, compared to world and inter-state,
ES recommends more research in high-yielding varieties and making them available to
all farmers, especially to poor and small farmers.
Marketing:
o APMC Act which was legislated to reduce exploitation of farmers has resulted in
exploitation of both farmers and consumers by intermediaries. The ES asks for
amendment in present APMC Act by states and in lone run providing more a national
market.
o Shanta Kumar committee recommendations which stressed on common agri market
require Consensus building among states.
Structural Changes:
o Increasing production will be challenge in itself given Decreasing /stable Land under
cultivation, Low Size and Farm mechanisation which also requires Bank credit, which
restricts credit flow Post waiver scheme and high NPA of banks. Also encourages wilful
default.
o Advance pricing and hedging price risk by farmer and strengthening FMC is underway
but will require time and efforts
Finance and Subsidies:
o Financially, lack of credit and march-run of banks harm the small farmers more. ES wants
the availability of credit whole year and on the basis of warehouse deposit-slips, so that
farmers can start for next season. Rationalisation of subsidies and better targeting to
reach to small farmers.
Technology:
o Leveraging modern technologies extension services to be provided and information about
commodity exchange market to be provided and farmers are encouraged to use these
facilities.
Conclusion:

Agriculture sector is overburdened with about 50% of the labour force dependent on it and is
largely unremunerative, need is for developing rural nonfarm sectors such as food processing
and at the same time adoption of technology in agriculture
Although recommendations are in line with Problems faced by Agriculture it requires Political will
to meet the challenges in total.
Only by making agriculture more productive, India can attain more inclusive and sustainable
development given the large dependence on agriculture sector.

Topic: Environmental pollution; S&T daily applications

45) What do you understand by Refuse Derived Fuel (RDF)? Recently National Green Tribunal
permitted states to produce RDF, but there are concerns expressed regarding the production of
RDF. Examine what are these concerns and suggest if there are any alternatives to RDF.

Source: http://www.thehindu.com/todays-paper/green-tribunals-thumbs-up-for-wastetoenergy-
projects/article7127827.ece

Introduction:

Refuse-derived fuel (RDF) or solid recovered fuel/ specified recovered fuel (SRF) is a fuel
produced by shredding and dehydrating solid waste (MSW) with a Waste converter technology.
RDF consists largely of combustible components of municipal waste such as plastics and
biodegradable waste.
RDF can be used in a variety of ways to produce electricity.
RDF is capable of being combusted cleanly or in compliance with the Kyoto Protocol.

Advantages of RDF:

It can be used to produce electricity, alongside the traditional source of fuel like coal.
It can used for co-incineration in cement kilns
Co-gasification with coal and biomass

Concerns of NGT regarding RDF:

Cost effectiveness of the process for power generation.


No segregation between recyclable and non recyclable waste, thus instead of doing good to the
environment it will harm the environment as while producing such recyclable commodities again
lot of fossil fuel or RDF itself will be wasted creating a vicious cycle.
Non inclusion of liquid waste in RDF.
Toxic and polluting gases emitted by this process of power generation.
Major concerns regarding use of RDF are with respect to the emissions of such plants, which in
turn creates another constraint in locating such facilities.
There is a dire need for technology upgradation in RDF; otherwise it becomes similar to poor
people burning waste to cook food, hurting themselves more in the process.
Concern of wet waste: used for feeding animals like veggies to cow sheds areas.

Alternatives to RDF:

RDF can be used as alternative fuel in Thermal power plant, cement factory, brick kiln to make
them cost effective and also Municipal will get rid of such wastes.
Under Kyoto Protocol it can be sold under Carbon credit exchange and hence India can earn
money and make it cost effective.
Mechanised separators and other methods like Shredding, size screening, magnetic screening
etc can be employed.
Co-incineration of MSW with fossil fuels has showed remarkable ecological advantages when
compared to combustion of MSW alone.
RDF facility needs can be placed near green Belts, and with technology upgradation the nature
and environmental impact of emissions can be kept under check.
Biological reprocessing for organic materials like biogas plants in rural areas
Plastic can also be recycled for Jeans, other products etc
Plasma arc gasification organic material is directly converted to synthetic gas, which can be
burnt cleanly to produce electricity.

Topic: Disaster Management

46) In the light of recent earthquake that crippled Nepals economy and caused huge loss of lives of
people, critically examine what policy measures needed to manage earthquake disasters. Also examine,
if Japans experience in managing these disasters can be emulated in India and Nepal. (200 Words)

Earthquakes are vibration of the earth due to plate-tectonic movement. Its risk is defined in magnitude,
probability and social fragility. Mankind have no control over 1st two but preparedness can reduce the
fragility.
- Recent, 7.9 & 6.8 Richter magnitude of was very high on social fragility.

Policy measures required to overcome impact:

1). SAARC seismological agency needs to be set up for integrated planning of preparedness for mountain
region.
2) Habitable zones should forewarned and also made responsible for protecting themselves.
3). Sites for all large dams and nuclear installations in the region need to be reevaluated.
4). Role of media in creating awareness about disaster preparedness. Role of social media to be
enhanced to make the society aware regarding the protective measures during Earthquake
5). Public& private sector involved in construction are also stakeholders in preparedness.
6). Legislation for Building Standard Law, tax-incentive for low cost anti-earthquake construction.
7). Special Earthquake force which is swift into action with proper training via drills as initial 72 hours are
critical in saving life.

8) Making the children aware regarding such natural disasters, by including disaster management in their
syllabus, along with regular drill practice in schools too.
Japans experience History is replete with the effectiveness of Japan disaster management
techniques to mitigate earthquake causalities. Some of the takeaways are
---Scientific approach to defining the earthquake risk and inculcating in their buildings.
--- Rapid response force and synergy between various bodies. There is surveillance by self-defence
forces, a disaster countermeasure preparation office starts working, and a medical assistance team
is on standby which is activated by a real time observation system.
--- Their continuous efforts to merge technology with disaster preparedness, involving the
community through training programmes enabling a rapid response, has been one of the constant
methodologies for saving lives.
---Japan's Real time Earthquake Early Warning (EEW) measure probability quickly as the
earthquake wave in sea travel at 8 km/sec which is less than an electric signal can be used in
coastal regions.
---Japans experience can be used in promoting research, public policy and investment, building
codes, awareness among masses including children at school, early warning system etc are areas
where India could draw lessons. However, as far as the technology transfer is concerned, India
cannot afford Japanese cost.

The Hindu

The Indian Express

The Indian Express

Topic: Inclusive growth and issues arising from it

47) It is said that to make the Pradhan Mantri Jan Dhan Yojana (PMJDY) a meaningful exercise in
achieving the goal of financial inclusion, the key driver will be the Direct Benefits Transfers (DBT)
payment flows from government to banks to customer accounts. Elaborate and explain why DBT is crucial
in this regard. (200 Words)

Financial inclusion or inclusive financing is the delivery of financial services at affordable costs to sections
of disadvantaged and low-income segments of society.

To make PMJDY a meaningful exercise for financial inclusion, at this stage, DBT can act as a key driver
in following ways:-

1) DBT will act as incentive for hitherto reluctant people to open bank accounts and realize its
importance as cash handouts will be directly linked to it
2) Transaction margins will lead to establishment of a network of bank branches and BC in
marginalized or excluded areas specially in country side thus bridging the integration gap in
financial inclusion
3) With time, ensuing competition and reducing DBT margins will force banks& BCs to adopt new
strategies to retain existing account holders by ways of new offers such as insurance, credit
concession, cheap financial products to win customer loyalty and gain edge over rivals--all
adding to financial inclusion
4) DBT will help in removal of dubious role of intermediaries, cutting of red tape in financial
services as it will enhance interaction level and customers will have more choices in terms of
account portability
5) DBT will usher in a new era of financial literacy by removal of traditional hesitation or
unreliability about financial methods and ICT tools and frequenting to branches and BCs will
make people more conscious about money movements, its earning potential and benefits of
financial instruments
6) A nationwide network of Bank-BC-end user will lead to a cash free digital economy which will
be an asset for business, private sector to streamline financial transactions, remittances, reduce
time therein and improve financial outreach-all adding to inclusion
-

In addition to all the above benefits DBT would give a positive push to the economy, enhance bank
transactions; bring every entitlement on easy to access electronic records etc. Thus, DBT is indeed a
landmark in India which can transform the society if challenges like ensuring adequate connectivity,
micro-ATMs; PoS counters etc. are overcome in time

Business Standard

April Paper 4

Topic: Essence, determinants and consequences of Ethics in human actions;

1) Do you think capital punishment is ethical, even if the convict is sentenced for a gruesome crime
belonging to rarest of rare cases? Also examine opinion of the Supreme Court of India regarding death
sentences. (200 Words)

The 262th Law commission report has recommended abolition of death penalty except in case of terror
and waging war against the state. The report suggests deterrence is a myth.

Points against ethnicity of death penalty -


* these are on the right ethical track, since retribution must not descend to vengeance. Society shall not
seek justice in the form of vengeance, which has been the case under 'capital punishment'. Seeking
vengeance terms the society 'blood thirsty'.

* Punishment by State, which involves killing of a human being, can never be justified.

* India has historically been a peace loving place. It is a part and parcel of our ethical and moral values. It
is high time that such barbaric penalties be removed from statute books.

* The correct way of punishment is return to restorative and reformative justice.

* Indias criminal justice system is not foolproof .Trial errors exist and if any innocent is hanged on basis of
framed evidences, it will gross failure of justice system.

*Majority of people with limited means are on death row, hinting toward lack of judicial capacity of
defendant in such cases. It would it be unfair to execute such persons

Points in Favor of Death penalty-


* it is unethical to provide capital punishment to criminal who understands what wrong he/she has done,
but it is totally ethical for convicts who are murderer by their own wish and don't bother committing murder
again and again.

*instead of leaving culprit in name of faulty criminal justice system, reform should be brought so that
innocent are not convicted.

*President has been provided with mercy petition to review that death penalty must not be unfair.

Supreme Court's on death penalty:

Capital punishment should be given only in rarest of rare cases like in honor killings and fake encounter
killings.
Murderer who murders of people from vulnerable social groups should be considered for capital
punishment.
Multiple murders where many members of a family or group are killed also merit the death penalty.
The murder of a wife in order to marry another woman with whom a man is infatuated can incur capital
punishment

SC in Shatrughan Chauhan vs. Union of India Case has concluded that inordinate delay in the rejection of
mercy petitions of death row convicts amounted to torture and that it is a sufficient basis, in and of itself,
to commute a sentence of death to life imprisonment.

SC says while considering the rejection of the clemency petition by the President, under Article 32 read
with Article 21 of the Constitution, cannot excuse the agonizing delay caused to the convict only on the
basis of the gravity of the crime. It creates new forms of the accountability of the executive at all stages of
the pardon process, right from the level of the Ministry of Home Affairs all the way up to the office of the
President of India.
Justice demands that courts should impose punishment befitting the crime so that the courts reflect
public abhorrence of the crime.

Capital punishment must be "observed" upon the atrocity of the crime, the conduct of the criminal
and the defenseless and unprotected state of the victim, and not just on ethical dimensions.

Topic: ethics in private and public relationships

2) What do you understand by conflict of interest? Do you think if a Professor who is guiding ten
students in their doctoral thesis has an affair with one of his students, will represent a case of
conflict of interest? Justify your answer giving reasons. (150 Words)

The Indian Express

Conflict of Interest is when an individual or entity is involved in an activity and there is also personal stake
of the individual/entity in the activity which drives towards self interest rather the holistic purpose. For
example, recent BCCI president Srinivasan as its head who makes rules of the IPL also has stake in IPL
as he or his relatives owning a team in IPL. This could drive person towards making rules in their own
favor.

A professor ought to be neutral to students in rewarding for their efforts in their academic endeavor. It will
ensure a level playing field for all and enthuse, hard work and competitiveness in students. Any personal
favors shown to one particular student will change the dynamics for the whole student community. The
deserving one will not get the due recognition they deserve and one less deserving might come at top.

a. The professors teaching reputation can easily be compromised and in the field of academics this
would be much more destructive as ostracizing from academic circles would mean the end of a career. As
the academic field prefers minimal emotional thinking for completing academic projects, having such a
reputation would automatically disqualify one from getting contracted for these positions.

b. The relationship would also result in a partial assessment and understanding of the students. The
students would also be demotivated and may potentially ruin their careers by under-working for their
respective theses.

c. The manner of academic research being conducted would be undermined, and would also be wastage
of academic and institution resources.

Such kind of relationship should be avoided as far as possible or should be pursued with extra diligence
and maturity so as to diminish or eliminate the effect it will have on outcomes of a result.

Topic: Role of family, society and educational institutions in inculcating values.

3) These days crimes against women are on the rise especially crimes such as rape. A part of
society blames the victim for inviting rape by wearing short dress or going out late at night. What
is your opinion on this issue? Do you think those who blame the victim have merit in their
arguments? Justify your answer. (200 Words)

Man is like many creatures is a Social Animal but what makes a man set apart from them is his ability to
use Brain unlike other animals which just succumb to their natural instincts of pleasures or any
responses.
This led to the evolution of so called Principles and Moral thinking which one should follow when we live
in a Society not to cause any disturbance or infringement of others rights.

Violence against women is increasing alarmingly in India specially the 'rape' and the perpetuating 'rape
culture' creates a cloud over the safety of women. Rape and atrocities are not a new phenomenon; rather
it's the positive effects of globalization, urbanization, social awareness that encourages a victim today to
report such incidents. So the so called modernization, liberal dressing trends, late night outing cannot be
blamed as the reason for 'inviting rape' -as argued by some parts of the society. National Crime Records
Bureau recorded 33,707 cases of rape and 70,739 cases of assault on women throughout India in 2013.

Such irrational arguments lack empirical validity and reflect stereotypical mindset. It reflects submissive
role of women accorded, owing to patriarchal nature of society. If inviting is the reason then how come a
village girl, dresses in salwar suit is being raped in mid noon while returning from exam, how children are
getting raped at schools, marital rape, gang rape by the instructions of village Khap Panchayats, raping of
middle aged women are done.

The clash between old India (Bharat) and modern India is currently taking place. In the developing
world, women are in special jeopardy. Their embrace of autonomy and mobility risks putting them in
conflict with a law-enforcement establishment and media that still view women through a pre-feminist
lens: Good girls who stay at home should not be raped, while bad girls who stake a claim to public
space are fair game. Those who blame the victim have no merits in their arguments. There are some
myths about rape.

1. MYTH: Women who wear revealing clothing invite men to rape them. TRUTH: Clothing does not
determine who gets raped.

2. MYTH: Its not rape if a woman wears revealing clothing, because then she wants to have sex.

TRUTH: When a woman wears revealing clothes she is not actively consenting to sex she must still be
able to deny her consent to sex if she chooses to...

Recent survey reveals that most of the rapes are by persons known to the victims/their relatives where
dress can't be a stance. Everywhere, at work, at public places, at home, at schools, these kinds of
atrocities continuing. If one has to equate everything with dressing sense of women then that is a
misconception. Supreme Court of India once said "Women are safer on roads than at homes. In 98 per
cent of the cases, the victim knew the
alleged rapist stating the fact that most rapes are premeditated acts by
rapists. In a 1996 survey of judges in India, 68 per cent
said provocative clothing is an invitation to rape. A Khap Panchayat in Baghpat recently barred women
below 40 years from using mobile phones and from shopping.

The reason for rape is not the dress/lifestyle, but it lid somewhere else deep inside the patriarchal
mindset of our society that creates gender inequality and insecurity in sphere of life including our
culture, education, upbringing. Many men still believe that women are only for household unpaid works,
procreation to continue their family and their body is the tool for their recreation and when they find any
deviation, and then they try to teach the women a lesson by showing their masculine power. We do not
look into reasons like easy access to liquor, lack of policing and apathetic attitude of society towards rape
victims. We as a society praise and idolize a porn star while a raped woman does not get any help even
on humanitarian grounds.

scared/worried parents have two reasons:

(1.) Conservative mindset that cant face the fact that women is now equal to men and have freedom to
do everything without being told and guided. This reason is dangerous as it dictates someones life

(2.) Worried parents/guardian who doesnt want to invite any unwanted bad gazing to their daughter/sister
and any trouble, hence find it easy to dictate women for dos/dont

The connection between rape, male privilege and female sexual vilification was one of the key insights of
feminists in the 1970s an insight they thought had been successfully applied to cultural debate about
rape, and to law.
Asking females to wear modestly is right to an extent because India is in transition phase between old
cultural values and new generation who got much open exposure to western ideas/dressing/culture.
Some people find it easy and fashionable to wear what they want, but we must consider that, major part
of crowd is not open minded as us. Different people take same dress in different perspective... Hence,
prudence is to wear what is appropriate to the place and time. For a ball party skirt is fine but in midnight
time living in shady corner of city, wearing skirt is not prudent.

It is not to be forgotten that women cannot go late at night because 'night ' is not ' safe' for life and dignity
owing to such obstructive environment; 'unsafeness' is not because of her 'night' visit ; As some part of
society is blaming the women for wearing the short dresses and going out late at nights. But as a part of
the democratic society being an equal human women are having right to walk even at the night times.
These type of ideologies need to be changed and the patriarchal thinking need to be changed
thus, it is essential that societal attitudes are reformed and such prejudicial notions are discarded. So that
women can have access to equitable and dignified rights. Our educational curriculum, proper gender
sensitization and family norms should inculcate moral values of respecting womanhood.

So it's the mindset, culture, social environment, background that is to be blamed for this grave heinous
situation, not the women. As Gandhiji rightly said, When a girl walks alone at the midnight on the roads
of our country, then we can proudly celebrate our freedom.
Though this statement may become true by the globalization world which is providing jobs for women
even at night times, but it is becoming absolutely wrong in terms of her security. The family, peer group,
educational institutions, new educational curriculum, social media can only help to inculcate a sense of
respect for women into the society and gift the women a safe haven to breath.

Topic: ethics in private and public relationships; role of family in inculcating values

4) Raman cleared UPSC civil services exam in his very first attempt and secured
second rank. He was a son of poor landless laborer. His parents lived in a hut in
their village. He got posting in his home state. He was given government bungalow
and car. He married a beautiful, well-educated daughter of a rich politician. Raman
now wanted to bring his parents to his bungalow and make them live with him. But
his wife did not like this. She told him bluntly that he should choose between her
and his parents to live in his bungalow. She told him that she was not comfortable
with the idea of his parents living with them in the same house. She told him that
she needed privacy. He yielded to her threats and did not bring his parents to live
with him. It was one of his dreams to make his parents happy, live with them and
take care of them in their old age. However, he made arrangements in his village
so that his parents could live in a better house with good amenities.

a) Do you think what Raman did was correct? Justify giving suitable reasons.

b) Do you think Ramans wife was right in expressing her opinion boldly and opposing Ramans
proposition to bring his parents to live with him? Justify giving suitable reasons.

Answer 1:-
(a) This question is very common these days because of rising culture of nuclear family and new found
independence in personal lives by everyone.
Raman here has 3 responsibilities viz (1.) Being a Civil Servant his duty to deliver duty with rectitude (2.)
His Wife (because he married her with fully grown consent, his wife left her home for him and she is going
to be with him till death) (3.) His Parents
First Raman must try his best to convince her wife, if not then he must try for at least few weeks visit of
their parents so that they know each other better, may be this can alleviate the concern of her mind of
encroached privacy.
If this is not successful, then Raman must realize that, he married her with full consent and now its his
responsibility to maintain peace and democracy in his home. An unhappy wife in home can decrease his
mental peace and efficiency of work. He has dream of keeping his parents with him, but if they are at
ease and happy in native, that is better, rather than running to pursue his dream at the cost of disturbance
in life.
It is not that he yielded to her threats; it is prudential and wise decision. Occasional visit of parents can
also be middle path which will not be problem for his wife and fulfil Raman's wish to serve his parents.
(b): Yes, Ramans wife is right in expressing her view, but the way and course of action of expression is
wrong. In married life, both husband and wife have equal say in any decision. He should have given a fair
chance to test whether all her doubts and fear of restricted freedom will be true or just cooked story
shown by Television serials.
She should recognize her husbands wish also.
Hence, she is right that she spoke her mind clearly but at the same time she should have given a chance
to experience first-hand their parents visit few times.
Answer 2:-
A. Ramans decision is a balance between his responsibilities as a husband and a responsible son.
It is a prudent and wise decision considering the situation. The woman has an equal say in the
couple's life. Hence it the responsibility of the husband to fulfil it too. Also it seems that the
parents were well off in the village that seems to face them with good conditionalities. A frequent
visit by parents in the house is acceptable in these circumstances that will fulfil both purposes.
Using forced decision imposition on the wife may lead to uncanny disturbances in the couple's
lives which have just started. It may then affect the professional lives too.
B. Wife has an equal say in the marital matters and matters concerning the couple. It is easy to
consider only the male point of view while discarding female's version going by the way of our
societal functionings. But this will only lead to disturbances in their marital life. Situation might
ease of in the future if female gets allayed of her fears about the parents. Possibilities are open
for many events. But parents have the option of visiting the child. Freedom of the wife should be
respected.
The Hindu

Topic: ethics in private and public relationships; role of family in inculcating values

5) Many times it is reported in news that victims of rape are harassed by the police and courts in
India. If you become a Superintendent of Police, how will you treat rape and sexual molestation
victims? Suggest measures that need to be taken to protect victims from harassment. (200 Words)

If I become a Superintendent of Police, I will follow Standard Operating Procedure (SoP) for Investigation
of Rape Cases.

Registration of FIR-:

1) As soon as information about such offence is received, FIR shall be recorded in accordance with the
provisions of Section 154 Cr.PC.
2) Investigation of the Rape case shall be conducted by a woman officer.
Handling of victim:

1) The victim should be handled with utmost sympathy and sensitivity. The behavior towards women
victim should be courteous. No embarrassing or indecent questions should be put to her who is already
under shock/trauma.
2) The victim should be medically examined only by a lady doctor.

Inspection of the Scene of Crime:

1) It shall be the foremost duty of the I.O. to safeguard the scene of crime so that clues available at the
scene of crime may not be disturbed.

Rehabilitation of the Victim:

1) The female victim needs help in rehabilitation. The rehabilitation has to be in four folds, viz physical,
mental, psychological and social.
2) The victim is likely to suffer social stigma and possible alienation from the family. Necessary counseling
to her and her family members should be provided.

Handling of Media:

1) Sex offences attract wide attention and arouse more public indignation than any other type of crime.
While briefing media about sex crimes, identity of victim should not be disclosed, if need be, pseudo
name may be given to the victim. Victim should not be produced before media.

Trial:

1) Charge sheet should be filed in the court within the limitation period I e 90 days after the arrest of the
accused, as thereafter the accused may be let off on bail.
2) During trial safety and security of the victim and witnesses should be ensured.
3) Presence of witnesses should be ensured in the court as and when summoned.

Some suggestion to protect victims from harassment:

1) Women police cells in the police stations and exclusive women police stations should be set up as
needed.
2) Police officials charged with the responsibility of protecting women should be sensitized
adequately.
3) Police personnel should be trained adequately in special laws dealing with atrocities against women.
Enforcement aspect should be emphasized adequately so as to streamline
it.
4) Women police officials in the State Police Force should be recruited widely.
5) Close coordination between the police and the NGOs dealing with the interests of women may be
ensured.

Topic: ethics in private and public relationships; role of family in inculcating values

6) One of your close friends was caught by your father while stealing money from your room. He
was a regular visitor at your home. He has been your close friend since your school days. Recent
he was facing severe financial problems and was badly in need of money to repay old debt. He
had sought your help, but due to your other commitments you could not help him. Your friend
stole money from you as last resort when his creditor threatened him with grave consequences if
loan was not repaid within stipulated time. Your father, who did not like your friends circle, now
wants to lodge police complaint against your friend. Your friend pleads that he has been nice guy
throughout and should be pardoned by you and your parents. Your mother is now accusing him
of stealing all the past missing things in your home. He begs you and your father to not to lodge
police complaint. The money which he stole was substantial and was meant for your sisters
marriage. You were saving this money for many years.

A) Should your father forgive your friend because he was a good person all these years and
committed the crime due to pressing personal problems? Justify giving suitable reasons. (150
Words)

B) Will you continue to treat your friend who stole money as one of your close friends? Why?
Substantiate. (150 Words)

(Different people have different opinion. Mentioning some points)

A) Should your father forgive your friend because he was a good person all these years and
committed the crime due to pressing personal problems? Justify giving suitable reasons.

1. Economic problems could make a person to go to any extent and what my friend did is no
different. It is my father's as equal responsibility to correct my behaviour as also to correct my
friends behaviour who has committed a mistake. And knowing me and my behaviour my father
must not lodge police complaint as that will spoil the career and could push my friend into more
problems and will have its consequences on me and my friends circle. He must talk to my friend
and counsel him about his behaviour which could create more problems than solve his economic
problems. My father must get money back first which is of much importance to family now. Then
enquire what consequences made him to borrow money and why he is unable to repay and ask
him to work for solving them in a proper way like requesting the lender to give some more time to
find alternatives or earn money. Mere stealing would not be the solution to all economic problems
and my friend may get into the trap of committing bigger crimes in later period if not corrected at
this crucial junction of his life. My father has to show him the right path and suggest him to work
hard to fulfil his needs.

2. Thieves steal something even more valuable than money or goods. They also steal
trust. It's a double blow, to lose your hard-earned cash or property and to have such valuables
taken by someone you know. Both aspects of the crime are rotten. The challenge, as the victim,
is handling both forms of betrayal to keep the rot from taking hold. My father will not forgive my
friend easily since he has committed a breach of trust that my family had on him since many
years. As my friends economic condition was not good, in this situation if my father lodges police
complaint against my friend that will definitely ruin his career. As everyone deserves a second
chance. I would expect from my father must show me and my friend the right path in these
circumstances. The point is to move on from the event while learning from it, and to reclaim
confidence. It's not easy, and it often hurts like hell, but such is the work of healing.
3. Trust and friendship is built over years. They are tested in bad times for their shadiness. As a
head of the family, it is the responsibility of the father to uphold the morals and values to the
utmost level. He should not forgive the friend since he has committed a breach of trust (and a
crime) that the family had on him since years. Secondly, the father must show his son and his
friend the right path in all circumstances and should not shy away from punishing them for their
wrong deeds. Once we start reducing the threshold value below which we arent to stoop down,
there are always chances that we may compromise on our ethics and principles for trivial benefits
and a time can come when petty crimes can turn into bigger ones if left unchecked in the
beginning.

B) Will you continue to treat your friend who stole money as one of your close friends? Why?
Substantiate.

1. I will try to treat my friend in a positive way provided if he promises me not to repeat this again.
Friendship cannot be spoiled with such mistakes and the trust that built must not be degraded. It
is a good lesson for my friends circle as well to know about good and bad and how money can
create problems in relationships. A mistake happens only once and repetition of it is a crime. It is
in my interest to make him know how his single act spoiled the trust and make him realize what
he did for both of our benefit. I will give up my friendship if my friend repeats this again as that is
corroding my character and beliefs. I will try to treat my friend in a positive way provided if he
promises me not to repeat this again. After all everyone deserves a second chance .At the end of
the day, the whole miserable experience distills into a simple teaching on ethics, mindfulness and
compassion. There is right and wrong to this situation. The ethics are clear, yet the emotions can
seem murky. Maybe we feel sorry for ourselves, and perhaps for whatever it was that drove my
friend to betrayal. Compassion grows here, for all victims and all perpetrators -- accountable as
they are, they suffer, too. In the end, it's the compassion that saves the day, because it helps us
heal.

2. It will be difficult for me to forget what he has done to my family since the money was kept for my
sister's marriage. I would least expect a friend to cause me such a huge loss for his personal
gains. It is evident that he has shown priority to his needs rather than my problems. I shall not
regard him as a close friend now, but shall continue the friendship and as a good friend will try to
make him understand that what he has done is a crime and he should restraint himself from doing
anything of that sort in the future.

LINKS

http://www.insightsonindia.com/2015/04/10/insights-secure-2015-upsc-mains-questions-on-current-
events-10-april-2015/

Topic: Attitude: content, structure, function; its influence and relation with thought and behaviour;
7) Should happiness be the ultimate goal in ones life? Where do you find happiness in your daily life? In
your opinion, what constitutes real happiness for you? Explain. (200 Words)
The Hindu
Happiness and Sadness are continuous part of life.
Happiness and Sadness are relative. For some, Happiness is perfect bliss, but for others
happiness is good health and safe life, while for someone else happiness is just absence of
sadness. Hence, Happiness is relative and depends person to person.
Being happy and content is sole aim of all living beings action. Nobody likes to be unhappy. But
being a social being, our goal should be little more than personal happiness. It should be
happiness for all. Our wish should be happiness for all and action should be in direction of
happiness for all who can be affected or related to our action.
Happiness in doing what one likes which can be job or hobby
The materialistic desire leads to unethical move by the person and creates large number of
problems and unhappiness. The best way to find happiness in life is to do the things which we
love the most and which is ethical, moral and acceptable to all rational minded people.
Real happiness lies in fulfilling small targets and goals, may be personal achievement goals, or
may be related to relationships and response to the environment.
Simply materialistic world does not carry the keys to happiness. Happiness is when what you
think, what you say and what you do are in harmony
Happiness is the state of mind. It depends on what makes one feel achieve that state.
Happiness is the best ingredient of life and has a capacity to heal many ills both physical and
mental.
find happiness in daily aspects of talking to family and friends, talking positives of life with whom
one meets, treating problems as one that strengthens, learning good things from others and
above all talking to myself
The Real happiness to lies in service to people.
A genuinely happy person is one who brings happiness to those around them.

Topic: Contributions of moral thinkers and philosophers from India and world.

8) An eye for an eye only ends up making the whole world blind Mahatma
Gandhiji. Do you think the meaning and message of this famous quote by Gandhiji
is relevant today? Comment.
Intro
Gandhiji's famous quote is one of the foundational guiding principles, which holds its relevance since
eternity. This phrase is used to describe an approach of revenge. The statement points to destructive
attitude of answering violence by violence in which people retaliate and harm other person without
considering mediation and peace talk. this cycle goes on and leaves only miseries behind

Elaboration
Though the context, space and time may vary, but the underlined meaning remains the same, which
can be applied in different ways but centered on a single theme - Non-violence. Gandhi's advocacy
for Non-violence and his belief of ultimate prevalence of truth, are two dominating force behind the
idea.

Gandhiji followed the path of non- violence. if he would retaliated the violent suppression of British in
the same vein, then there would have been a huge turmoil and heavy loss of lives and every effort to
gain freedom would have been in vain. His belief of not hating the criminal, but the crime, liberated
India from the mighty colonial power

This has the potential to deal with several problems that Humanity is presently faced with. Climate
change, terrorism, poverty and many such similar problems can be dealt with different approach
guided by the quote. For example

To deal with the climate change, we need to fight with the source of climate change, change our
living habit, have a balance between materialism and spirituality and above all live with the nature
in harmony. Thus the fight is with the problem.
Similarly instead of fighting with terrorists, we need to fight with the very origin of terrorism which
is poverty, twisted interpretation of religion and negligence. Though a situation may arise where
lives of many people is at stake, like in terrorists hostage case and be dealt with use of force, but
beside such exceptional situation, the quote is time tested instrument of action.
In current geo-political scenario of nuclear weapon era when every nation is in a race to acquire
nuclear weapon to flex muscle shows application of this statement where if nations go against
this principle, there will be endless destruction.
Similarly in social scenario, rising number of crimes, violence, terrorist attack illustrates same
philosophy where in 2 parties, shia-sunni, religious extremists fights each other in an unending
and inconclusive war. there is endless revenge in Gaza strip , Yemen-Saudi Arabia , Syria crisis
etc
In Economic scenario, companies these days take path of destructive competitions like buying
customers, buying all other companies, providing free services and it becomes economic war
between firms to shame and malign each others reputation. These is clear from Ola cab bought
Taxi for sure, while Sukhoi hired bloggers to shame Dassault Rafale in internet so that India will
not buy it.

Conclusion
2 world wars have shown that how conflict of egos and easily solvable disputes can escalate to
unimaginable bloodshed. Tolerance lies at the very foundation for the smooth and peaceful co
existence of today's multicultural global village that the world has shrunken into.

At the individual level, hurting a person the same way he has hurt you will only result in two hurt
people, not justice. Besides, acting on a sense of revenge will invariably spark a never ending saga of
vengeance. Gandhi's words founded on non violence, tolerance and forgiveness are relevant today
and will remain relevant for as long as mankind exists.
Topic: Political attitudes

9) Do you think so called vote bank politics is ethical? Substantiate. (150 Words)
The Hindu

Intro:

1. The multifaceted word ethics derives its authority from human rationale and guided by the
principles of honesty, equality, justice, transparency, truth and similar acronyms. Whatever
be the context, its fits into the size and acquire the desired shape but is always guided by the
above basic fundamentals.
2. In the context of politics, the very basic ethical conduct warrants an equal level playing field for all
the contestants on one hand and informed voters on the other hand, to enable a free, fair and
transparent election. But often both of these morals are twisted to meet the vested interests of
power hungry politicians.

3. Vote bank politics is a divisive tactic (generally along the caste, religion or linguistic lines) used by
politicians and political parties as part of social engineering to win elections in First past the Post
System.

Repercussions of Vote Bank politics (Unethicality Reasons):

1. Against the Constitutional Ideals of Fraternity, Unity and Integrity of the nation and no
discrimination based on caste, class, religion etc.
2. Blot on secular democratic nature of country.
3. Damages social harmony Social tensions.
4. Dying institution of caste not only revived but also revitalized.
5. Lack of ethical commitment from top to bottom in bureaucracy.
6. unbalanced growth
7. rise in poverty
8. regional discrimination
9. Fractured political system.
10. Hampers the fair judgement of voters, who should vote on the basis of merit and integrity of
candidates, instead of choosing on the basis of caste and religion.
Some examples:

Babri Masjid demolition, exploitation of U.P.ites and Biharis in Mumbai, decision of releasing the killers of
Rajiv Gandhi or even the Gujarat riots, Muzaffarnagar riots.

Solution:

1. Banning caste based rallies as recently done in U.P. by Allahabad High Court.
2. Change the system of elections from First Past the Post to Proportional Representation.

Conclusion:

1. Jyoti Basu once said "morality is becoming a casualty in politics."

2. Gandhiji said that whenever there was difficulty in solving political problem; don't divide
causes but think of the poorest and depressed person in the country and how his problem can
be solved. But, Vote Bank politics turns this noble idea upside down.

Counter-View:

1. If we view the issue of Vote bank politics from a KAUTILYAN ANGLE we find that its correct and
ethics has no relation as far as capturing power is concerned. The same view is amplified when
we talk in terms of Machiavellis philosophy which offers a realist and pragmatist approach
towards politics and its means.
2. At the same time, it may also be argued that appealing only on the basis of identity of the voters
cannot guarantee winning of elections, because the voter is assumed to be rational and
pragmatic, while casting his vote.

3. Also, it may be claimed that vote bank politics is only an appeal and an appeasement; its not
coercive as such. Hence, it is not right to blame the political leaders because its ultimately the
voters who give legitimacy to such an appeal.

4. On the other hand, an Idealist approach would offer a contrary opinion. In this view, playing a
vote bank politics is unethical because it defames democracy and its soul, it ruins a healthy
competition between political parties and at the same time it disillusions the people with many
promises which seldom get fulfilled once power is captured.

Although, there exists diverse opinion but if vote bank politics is healthy then we may conclude that
everyone is free to some extent in doing whatever he can to capture power, it is the wisdom of the voter
which actually matters.!!

Topic: role of family, society and educational institutions in inculcating values.


10) Recently, a student belonging to one of Indias prestigious colleges teaching liberal arts was
suspended for carrying the principals interview in an online magazine. The principal suspended
him citing the reason that his interview was published before it could be edited and approved.

Do you think Principal was right in suspending the student from his classes? In your opinion,
what values are compromised by both the principal and the student? Discuss. (200 Words)

Introduction:

- Interview is an open ended discussion between interviewer and respondent.

- Here, if there was prior agreement between Principal and interviewer or already set protocol of
college such that, any material before put up for publication has to be approved. Then the student
is wrong as he broke the protocol. Principal holds rights to take action BUT with agreement of
other stakeholders like student(s), professors etc.

Issues:
- Both Interviewer and Respondent are free to ask and answer respectively. In this case Principal
gave interview and his interview was published before showing him draft version without his
approval this is failure of journalism ethics from student.
- But at the same time Principal taking such harsh and without mediation decision single-handedly
shows lack of democracy in college administration.
- Also, as it is Liberal Arts College, so such action by principal also doesn't showcase good
example for what college stands for.
- It is an error both in process of suspension and grounds of Suspension. Process was arbitrary,
single handed, one man Decision, No Single chance was given to the Student involved in the
Activities which in principal's views was divergence" "Not- according to Institutions standards.
- One must note that such happened in the Prestigious Liberal arts college where thoughts and
actions are allowed to fly.
- Also Grounds of Suspension are also not in line with the Punishment accorded. As per the
information, He [student] is suspended for posting interview of principal unedited and unapproved
on E magazine.
- Also there can be a possibility that he did this same thing in order to show his achievement
among his mates in the college, and he might not had known about such approvals and
consequences of his actions.

Values compromised By :

Principal- Transparency, Fairness in decision, No chance given to correct or apologise, Punishment


rather lesson and reason was awarded, freedom of expression
Student- He being student of college should have asked atleast or kept Principal in loop before doing the
same. Also Before posting he should have consulted with his /her friends for any corrections or principal
Himself. Also before going to Court He could have used Platform in College itself and taken down the
post from such magazine

Conclusion:
- So, here both student and principal shows lack of ethics for what they stand for which is
journalism and liberalness.
- As both of them belongs to a reputed Institute of liberal art they should act with more responsibly
and act wisely to be followed by others, not to be in highlights for wrong reasons and try to avoid
any such issues in future.
- So, clear cut guidelines should be made public by the Principal to avoid any such issues in future.

Topic: Utilization of public funds, challenges of corruption.

11) Public servants have, in fact, been treated as special category under Section 197 of the Code of
Criminal Procedure (CrPC). Discuss why. Do you think such special protection leads to corruption?
Examine. (200 Words)

Introduction

Public Servants role is extremely important in driving a nation forward.

In a vibrant democracy like India, Public Servants role is highly challenging as one is expected to do a
variety of jobs ranging from jobs which require technical competence to the jobs which are targeted to
provide mundane governance.

Special Treatment

Special protection to Public Servants and not treating them under the normal CrPC is to insulate them
from conflicts between politicians and bureaucrats so that the smooth running of administration is not
affected.

For this purpose Section 197 of the Code of Criminal Procedure disallows courts to take cognizance of
criminal charges against a public servant unless previous sanction to prosecute him is received from a
competent authority.

Reasons for Protection:-


It was added to provide freedom to public servants to work hassle-free without worrying about
disturbances caused by unsubstantiated accuses.
The section 197 is novel given the long time taken by our judiciary and the involvement of public
servants in many ventures.

Misuse of this provision:-

While it is important to provide them immunity, it is undeniable fact that this special treatment
have been and can be misused by corrupt officials or might be manipulated to meet the vested
political and other interests.

This provision has helped many corrupt officials to grow and yield the crop of self-benefit. Being
treated special has given such official a false sense that they can utilize the underlying capital of
their designation to fulfil their own interest.

The authority can use it to delay the sanction indefinitely even if prime facie evidences are
against the public servant, who in turn had to work according to the whims of politicians.

It creates nexus between public servants and politicians which is considered as the major cause
of corruption.

It also creates a different class which beyond the jurisdiction of courts and this special treatment
make it more opaque and unaccountable to public which goes against the principle of public
service.

Way Forward

As it has been rightly observed by the Supreme Court, Special protection should be provided only
on case by case basis depending on the merit and honesty of the officer. This will create a sense
of responsibility and accountability.

At every cost rule of law must prevail and along with it Justice must be served for the society
without corruption.

Interpretation of section 197 of Crpc must be done in right spirit so that irresponsible and
unhealthy allegations should be curbed by insulating Public servant from fake cases and at the
same time focus should also be on mitigating its misuse.

Conclusion

Protection of public servant is important for their fearless service. However, with increasing cases of
corruption among public servants, care must be taken that powers given to serve people are not
misused to serve oneself. Dishonest and convicted officials should be dealt with strict punishment to
create strong deterrence towards corruption.

Reference:http://www.thehindu.com/todays-paper/tp-national/not-a-shield-for-corrupt-
officials/article7120648.ece

Topic: Utilization of public funds, challenges of corruption.


12) a) In your opinion, in India, what barriers exist for women to showcase their talents in different fields?
Critically discuss (150 Words)
Women to achieve or show case their talents has to overcome various Impediments ,obstacle, which
Opposite sex do not have to do it in Various Sectors/fields across India . In This Sense Emphasis of
Father of Nation becomes important which was, to call a Women Weak is Libel, Its a Men injustice, to
women. Despite this, Various Sectors Ranging from Manufacturing, Operations, Field sales, Sports, Self-
employed and Services. Broad contours of Social restriction to This Sex are
~ Morality,
~Considering as Weaker sex
~ Delicate , something which needs to be protected By "men" from men
~ Certain fields requires dress, viz sports which is not approved by society Hence cultural restrictions.
Recent issue, which Post Sc ruling allowed Women to enrol as makeup artist was also founded on such
irrational contours described above

12) b) How will you react to your friends opinions and his advices? Justify. (150 Words)

Everyone has the right to his opinion in this free democratic country. I would agree to his statement to
the limit that she can take care of the household works efficiently. However his opinion that girl should
restrict themselves to that work only amounts to imposition of work violation right to freedom. Men and
women are equal and have the right to work their choices, certainly after marriage too. I would explain to
him that world has changed now. A husband and wife cooperating in professional lives have great
benefits. They can be their mascots and supporters and partners.

His opinions are shaped by his upbringing. So he is ignorant of those mothers that are highly active in
professional worlds. I would advise him to consider his position again and read and be aware about the
happenings in the world. Times have changed.

The women themselves need a change in mindset. They too believe with my friend's ideology. Probably it
has been ingrained due to repeated seeing similar situational stuffs and being ignorant about other
opportunities that exist. Her education health is utmost important. Imagine that if lady in the household is
well versed in every respect, her handling of elders and child rearing capabilities will be so much
increased. She can contribute to the economy of the household and be a motivator and supporter of the
whole family.

So women care should be the priority and for that change in the mindset is the first step. I would suggest
same to my friend.

Topic: laws, rules, regulations and conscience as sources of ethical guidance;

13) In your opinion, why is child marriage considered as illegal and unethical? If a 15 year old girl
wants to marry a person whom she loves, shouldnt she be allowed that choice? If denied, does
this mean one is denying her rights as human being? Critically comment.

Source: http://www.thehindu.com/todays-paper/timely-intervention-helps-stop-child-
marriage/article7128173.ece

Introduction:
The illegality of child marriages is set on the premise of Prohibition of Child Marriage Act. The
act itself was based on the scientific analysis and knowledge of the human body.
Perceived harms and future life when taken into consideration the law makers in India arrived at
certain age where it may be safe to allow marriages that secure the individual's life and make
society stable.
Teenagers/Children are mentally and physically not fully mature. They grow at the pace of
biological maturation and in the way what is exposed to them. Their growth depends on
environment, parents, facilities like books/schools/outside exposure, foods etc. In initial years
their mind and body is fast changing.
Hence, it takes time to grow as mentally stable, physically capable to sustain as a part of society.
After marriage one has to take bigger roles and wider decisions about family which a child cant
sustain economically and psychologically.
Early marriage of children, especially girls make them vulnerable to many complications related to
reproduction. It is the age of growth and body is going through many changes and require proper
nutrition and care. Pregnancy at tender age may increase the chances of mortality.
Another reason for prohibiting child marriage is at such a age chances are to have frequent
pregnancies which in turn affect its health, and contributing in population growth because at such
a young age lack of information for secure physical relation and lacking other related information.

Denying a 15 years old girl for not marrying at this age is not encroachment of her rights as
human because:

Constitutionally she is not allowed to marry at age of 15.


Biologically she is not prepared to conceive.
Economically she is not independent to take such decisions.
Psychologically she is not strong enough to bear stress if something goes wrong.

If a minor girl of age 15 wants to marry her beloved person, she should not be allowed to marry at this
stage but should be advised to wait until she attains the determined are to get married. She should be
informed politely about the consequences of such marriage, if the couple really love each other, they
must not have any problem regarding proposal.

Conclusion:

So, the denial of her rights to marriage does not come under human rights, as she is still a child.
On the other hand, this act is protection of her rights as a child.
Denying it does not mean denying her rights as human being because although marrying person
of choice may be right but marrying early is not because it is against her own interests as well as
those of society and a wrong act cannot be considered a right.
Although, she should have absolute choice of marrying her lover but the girl must be persuaded
to take more time and reach the age of 18 before taking a decision on her marriage.

Topic: challenges of corruption.

14) Do you think is it right to punish bribe-giver along with bribe taker? Justify. (150 Words)

Corruption is the abuse of entrusted power for personal gain. Of the different forms of corruption
cohesive and collusive is significantly rampant .In collusive two parties committing an immoral and illegal
act - the bribe giver and the bribe taker. The common perception is that it is because of bribe seeker
asking for the bribe in lieu doing the work, people are forced to pay bribes. In this context, the idea of
punishing both the bribe giver and bribe taker requires examination.

The bribe seeker compromises on his integrity, human values and ethics, and abuses his positional
power.

He is also violating the law of the land. On closer examination it is clear that a bribe seeker makes
compromises out of greed for personal gain, thus violating the trust posed in him by the society.

The bribe giver on the other hand may give a bribe out of his desire to gain undue benefits, or may be
forced to give bribe to get his due right out of situation compulsions. The need of the hour is for a law
which punishes the bribe seeker, and the bribe-giver which will act as a deterrent

The Hindu

Business Standard

Das könnte Ihnen auch gefallen